Current Affairs’ “Some Puzzles For Libertarians”, Treated As Writing Prompts For Short Stories

[Taken from here.]

I.

Deep in the forest, thousands of miles from civilization, there is an isolated village. It has not seen contact with any other humans for a long time. It is, however, a pleasant and flourishing community, which strongly values freedom and entrepreneurship. There is, however, one tiny quirk. In this village, there is a ritual. Every year, a boy who reaches 18 is cannibalized. It brings the rains, or something. But despite its taste for cannibalism, this village wishes to live in accordance with libertarian principles. Thus, they will only cannibalize the boy if he consents. In order to encourage this to happen, they will put tremendous social pressure on the boy. All through his youth, they will tell him they believe the future of the village depends on his consenting. His parents tell him that he would bring great shame on the household if he refused, which is true. The choice nevertheless rests with the boy, and whatever he chooses will be respected. The parents and villagers attempt to persuade him, but never lie to him, and make clear that they would never force his choice. However: if the boy refuses to be cannibalized, the village has a backup plan. The boy will be blacklisted. No shopkeeper will sell him food, no hotel will give him a room, no hospital will treat him, no employer will hire him. After all, under libertarian principles, nobody can be told how to use their property. The boy’s parents, ashamed of him, will turn him out of the house with no money. He may leave the village, but it is certain death, for thousands of miles of desolate wolf-infested wilderness stand between him and other humans and he has no food. (The wilderness is also privately-owned, and he cannot pay the admission fee.) He is shunned and despised, left to wander the streets in a futile search for shelter and sustenance. However, no force is exercised against him. He is never touched or arrested. He is treated as nonexistent, as the villagers await his demise. So the boy starves to death. The villagers then cannibalize his emaciated corpse, reasoning that they cannot be compelled to give him a dignified burial (plus he died on private property, collapsing in a flowerbed).

Is eating the boy’s corpse after he dies the only potential violation of libertarian principles in the village? Is every single other aspect of this completely permissible?

The setting sun shone its last few rays on Independence Hall. The delegates were tired, but a thrill of excitement filled the air. The wrangling and deal-making was almost done; nothing remained but a few technicalities.

As the last sunbeam went below the horizon, something stirred in the middle of the chamber. It grew into a wind, then a whirlwind, and then standing among the assembled Founding Fathers was a strange man dressed all in silver, wearing a pair of gold goggles.

“You’ve got to stop!” he shouted. “It’s all lies!”

George Washington had stayed calm through cavalry charges, artillery fire, and the assembled might of the United Kingdom. He flinched only a little here. “Who are you, sir? Where have you come from?

The traveler barely heard. “Listen! You think ‘democracy’ can solve all your problems. But – imagine there’s a village full of cannibals. They have a tradition of picking a child, and killing and eating him when he’s eighteen years old. Well, even if that village is a democracy, then 51% of the population can just vote to kill and eat him! Do you want a child to be killed and eaten? Because that’s what your ‘democracy’ inevitably leads to! Checkmate, liberals!”

The delegates were only less dazed by the man’s speech than by his sudden appearance. Finally, General Washington asked whether anyone wanted the floor. After a scramble of shouts and raised hands, the chair recognized James Madison, delegate from Virginia.

“Thank you,” said James Madison. “Our Traveler may not know this, but I am preparing a Bill of Rights to be added on to the end of this Constitution, severely limiting the powers which the government may exercise. I’m planning one on cruel and unusual punishment, which sounds like it ought to cover killing and eating someone, and there will also be various restrictions on seizure of persons. The Traveler is already wrong that we operate entirely on the basis of 51% of the populace – rather, there will be representatives, senators, and Supreme Court Justices. But even if all these people should agree to kill and eat someone, I am confident that the natural rights included in my bill will restrict such practices.”

“AHA!” said the Traveler. “You’ve fallen for my trap! Because even if the government is banned from assisting in killing and eating someone, it could still happen. Imagine a system where, if the victim refused to be killed and eaten, then everyone in the village refused to house him, or feed him, so that he starved to death. Then he’d be dead anyway, and your precious Bill of Rights wouldn’t be able to do anything about it!”

“Couldn’t the victim just move to a different village?” interjected John Jay.

“The village is in the middle of a giant forest stretching five thousands miles, teeming with dire wolves,” snarled the Traveler, annoyed at such a stupid objection.

“Couldn’t the victim just build his own house, and farm his own food?” asked John Adams.

“The dire wolves would tear up the house, and trample all over the farm!” said the traveler. “You’re splitting hairs here! Why won’t anyone answer my question in the spirit it was intended?!”

There were more shouts and another frenzy for attention. General Washington banged his gavel. “The chair recognizes Alexander Hamilton.”

“Yo,” said Hamilton. “The institutions of our Constitution, give a clear solution to this persecution. The Revolution…”

“The chair unrecognizes Representative Hamilton, and offers the floor to anyone who does not speak in rap.”

“Thank you,” said Benjamin Franklin. “My good Mysterious Traveler, perhaps you labor under the misapprehension that political philosophies are also moral philosophies, and so fail irredeemably if they ever recommend an immoral course of action. I do not believe democracy is always right. But I believe it is a wise way to govern. All that systems of government can do is take nations – with all of their conflicts, ideas, prejudices, and values – as input, and then magnify some impulses and suppress others. Start with a country where every single person is entirely set on doing as much evil as possible, and democracy alone cannot save it; they will simply vote to do as much evil as possible. But start with a country in which there are many different classes, agendas, and visions, and I believe that a democratic system is more likely to magnify those impulses that help the common people, and suppress those impulses that lead to tyranny, than any other system yet devised.”

“So you’re saying,” said the traveler, “that you don’t care that your precious democracy and even your so-called Bill of Rights aren’t good enough to save the life of a child in – ”

“You listen here,” said Benjamin Franklin. “I care plenty. In a village that didn’t have any form of government, as soon as anybody big and strong enough wants to eat you, they can form a mob and drag you away. In a village that operates as a direct democracy, it’s harder. You need 51% of the population to want to eat you before you end up as dinner. And in a village that subscribes to Mr. Madison’s notion of natural rights, it’s harder still. You have to have every single person in the village agree not to feed the victim, without a single kindly old lady leaving food out on her porch at night when it’s too dark out for anyone to see. We have gone from tyranny – a system where, as long as even one person wishes you ill, you perish – all the way to a system where as long as there is a single person who does not wish you ill, you endure. That seems to me to be the best we can do in this world.”

Suddenly the Traveler seemed to warp, or crackle, like a signal from far away was being disrupted. “I must go!” he said. “I’m being recalled to my home time!” he shouted. “Where I will tell people that they should form a government based on socialism, and that it will be great, and nothing can possibly go wrong!”

“Stop!” said Jay. “You must tell us about this ‘socialism’ of yours!”

“Say, before you’re lost to me, at very high velocity, what is this new philosophy, that can prevent atrocity?” begged Hamilton.

But it was Washington, ever the man of action, who jumped up from the chair and grabbed the Traveler by his silver arm, holding him against the winds of Time. “This ‘socialism’ of yours – ” asked Washington. “It can ensure that – even in a barbaric society where literally one hundred percent of the people are wholeheartedly dedicated to do so – nobody ever eats their fellow citizens?”

“Yes!” said the traveler. “Why, in true socialist countries, nobody ever eats anything at all!”

Then he broke free of Washington’s grasp and disappeared forever, just as the first rays of the moon cast their white light on Philadelphia.

II.

Is there a meaningful difference between coercion by the state and coercion by private entities?

The door caved in loudly and suddenly, like a thunderclap. My Golden Retriever ran up, tail wagging, to investigate. Another loud noise, and my dog lay dead, bleeding on the floor in front of me.

“PUT YOUR HANDS UP!” said a man in black body armor and a black helmet. There were five of them, all with guns. My five-year-old son started to cry. “HANDS UP!” he shouted, “I’M NOT GOING TO TELL YOU AGAIN!”

I put my hands up. My son, who was screaming, got the presence of mind to put his hands up also, though not before one of the armored men had put a gun to his head.

“What’s wrong?!” I asked. “Why are you doing this to us?”

One of the men put a gun to my head. “Admit it! You’re growing marijuana here!”

“I’m not!” I insisted.

“Is this 2051 Willow Street?”

“No, this is 2052 Willow Street. 2051 Willow Street is on the other side!”

“Oh. Well, sorry.”

“Sorry? That’s all you have to say? You killed my dog! You terrorized my five-year-old son!”

“Yeah, sorry. We’re McDonalds employees, and corporate headquarters must have given us bad directions.”

“You’re…McDonalds employees? Why are McDonalds employees doing no-knock raids in body armor looking for marijuana?”

“I…I don’t know.”

“Then why are you even here? You just think it’s okay to randomly go around, kill people’s pets, terrorize their families, when you don’t know why you’re doing it? How can you justify such a thing?”

“I heard there were a bunch of people who were okay with private coercion, and only objected to coercion when it was applied by the State.”

What? Where did you hear that?”

“I don’t know. Some socialist magazine, I think.”

III.

Can you construct a theory of property rights that does not suffer from internal incoherence or depend on specious natural law assumptions?

Professor Kryzenski sat down in her desk and booted up her computer. It was another quiet morning here at the Harvard Philosophy Department. She had won her position as Department Chair by discovering a complete theory of morality grounded in first principles with no internal incoherence or any specious assumptions, able to determine everything from the optimal number of minutes to spend speaking to your mother each week to how close you could come to beggars before you were obligated to give them money.

She had just finished checking her emails – mostly invitations to speak at various conferences and events – when something started to stir in the center of her office. It turned into a wind, then a whirlwind, and finally, a strange-looking man, dressed in silver with gold goggles.

“Professor!” said the Traveler. “Professor Kryzenski! Terrible news!”

The Professor, whose mind had plumbed the depths of ontology and ascended the heights of metaphysics, was a hard woman to perturb. “Yes?” she asked the man. “What is it?”

“Suppose there’s an evildoer who punishes all evildoers who do not punish themselves. Does the evildoer punish himself, or not?”

Professor Kryzenski realized the implications right away. “My God. It’s a paradox! My complete theory of morality grounded in first principles with no internal incoherence or specious assumptions, able to determine everything from the right amount to tip your waiter to the exact words you need to speak before a sexual act for it to qualify as consensual – lies in ruins!”

“And that means…” began the Traveler.

“That’s right,” said Professor Kryzenski. She and the Traveler spoke in unison: “Nothing is true and everything is permissible.”

“Come,” she said. “I’ve prepared for this day.” She took a key out of a potted plant on the windowsill, then used it to open a locked cupboard in her desk. Inside were two hatchets. She handed one to the Traveler.

“Where are we going?” he asked.

“To the daycare down the road, to hack the limbs off the babies,” she said. “Obviously.” The Traveler nodded his approval, and off they went, eyes red with bloodlust.

IV.

The Infinitely Rich Man is not infinitely rich. He is just very, very rich. Nobody knows quite how rich. One day, you happened to meet the Infinitely Rich Man in a bar. At first he was friendly, but soon you found yourselves in an argument about horses. You were for them, and he was against them. Or perhaps you were against them, and he was for them. You don’t actually remember how it went. As you parted ways, you expected never to see the Infinitely Rich Man again.

Little do you know: the Infinitely Rich Man now despises you. His sole desire on earth is to see you unhappy. This should hardly trouble you, though. After all, you have a good job at a castanet factory. You own your own home, which has a picturesque lake view. You have a wife, whom you love and who loves you. You also have a prized possession, your 1972 Pontiac Lemans. You don’t have much spare cash, but this never bothers you because of your stable job. The Infinitely Rich Man is also a strict Libertarian. He believes it is illegitimate for anyone to initiate force against another. And because you are fortunate enough to live in a Libertarian world, you are free to enjoy those things you treasure most in the world without being bothered by the state or the Infinitely Rich Man. The Infinitely Rich Man is not discouraged, however. He still believes he can ruin you. He will be a Count of Monte Cristo, but an extremely law-abiding one.

The first thing the Infinitely Rich Man does is buy the castanet factory where you work. He immediately fires you. He also makes sure that if any other employers inquire about you, the castanet factory will refuse to serve as a reference. Not that this matters, for he intends to bribe any other castanet company who hires you into firing you. (There are four castanet companies.) You therefore find yourself unemployed. Fortunately, you have a skill. You know how to make castanets! (Castanets are very popular.) So you scrape together what money you have, and you open a little drive-thru castanet stand out on Route 9. But the Infinitely Rich Man has a plan. He opens a stand next to yours. At his stand, castanets are free. He gives them away by the truckload. He sets the whole world clacking. You cannot compete. You are ruined.

At least you still have your wife, your friends, your lakeview home, your 1972 Pontiac Lemans. But the Infinitely Rich Man has a plan. First, he buys the lake. He fills it with concrete. No more lake view, and your property value diminishes by $100,000. Then, he buys every house around yours, flattens it, and turns it into a landfill. The smell doesn’t reach your home, but it turns the neighborhood unsightly and desolate. Your house becomes worthless. The Infinitely Rich Man buys the heating company and refuses to provide gas to your home at any price. (You try to talk other gas companies into competing, but they refuse; laying a new main for a single home would be absurd, they say.) But you have a wife! And friends! And you get to drive a 1972 Pontiac Lemans! The Infinitely Rich Man offers a bribe. Any of your friends who refuse to speak with you ever again will receive a salary of one million dollars per year. At first, many decline to take the bribe. But sooner or later, most of them have one or another sticky financial situation, and they give in. Goodbye, vast majority of your friends! At least your wife loves you.

But one day, she becomes ill. She finds out that she will die, unless she goes on a treatment regimen for the rest of her life. The regimen costs $100,000 a month. The Infinitely Rich man pops up, and offers to pay. The one condition is that she divorce you, cut contact, and never speak with you again. As soon as she breaks the agreement, he will cease to pay for the treatment. You love your wife, but you do not want her to die. You both agree that it is better that she should accept. At least you can drive your 1972 Pontiac Lemans. Oh, but wait. The Infinitely Rich Man invests heavily in electric energy. Slowly, he makes gasoline-powered transit obsolete. He buys the oil companies, burns the gasoline, and converts every gas pump to a charging station. You can only drive your Lemans short distances, using some of the last gallons of available petrol, which you ordered from the internet. (That is, if the Infinitely Rich Man didn’t outbid you!) They don’t make the Pontiac Lemans anymore. Parts therefore exist only in small quantities. The Infinitely Rich Man buys up all existing Lemans parts. The moment it breaks, you are out of luck. As you sit alone, broke, and starving in the garage of your unheated home, caressing your disabled Lemans, thinking about your long-gone wife, your lake view, and your job, you are thankful that you live in a world of freedom, where nobody can encroach upon the liberty of another.

Questions for Libertarians: Has the non-aggression principle been violated? Should the Infinitely Rich Man suffer any civil or criminal penalties for his actions?

“That,” said Mr. Thaddeus Nett-Worth III, Esq., “is the most benightedly offensive statement I have ever heard.”

“All I said,” I said, “was that horses were basically elongated cows.”

“They are a noble animal, an unparalleled paragon of mammalian perfection!”

“Right,” I said. “Like cows are. Only more elongated.”

“Dastard!” said Mr. Nett-Worth, pounding the table so hard his top hat and monocle almost fell off. “You’ve messed with the wrong captain of industry, believe you me. Let me tell you what I am going to do. You own a castanet factory? I am going to undercut you, undercut you bad. I will destroy your business. Then, I shall buy the lake by your home and fill it up with concrete. I will buy your heating company and refuse to sell you gas. I will bribe your friends never to speak to you again. I will wait until your wife develops a deadly disease, then offer to treat her if only she divorces you. I can do all of it, because we are in a perfectly libertarian society with no laws besides the non-aggression principle, where lawmakers have failed to pass commonsense legislation like ‘making it illegal to hurt someone by legal means’. And then – alone, friendless, shivering in the cold in your hopelessly ugly house – then you will rue the day you ever compared horses to – ” (he almost spits) ” – elongated cows.”

“But, I mean, think about it. Their faces are a little bit longer. Their bodies are a little bit longer. They’re pretty much just elongated cows. I’m sorry this is so hard for you.”

“What? You’re not backing down? You should be at my feet, begging me for forgiveness! Don’t you know all the things I can use my wealth to do to you in our perfectly libertarian society?”

“Yeah, well, about that. I know this guy named David Friedman, whose hobby is designing weird insurance systems for anarcho-capitalist utopias based on, like, the laws of medieval Iceland or something. Anyway, he sells ‘rich person gets a weird grudge against you’ insurance. I have loads of it. However much you try to bribe my friends not to talk to me, his company will pay more to bribe them to ignore you. However much you try to pay for my gas company, his company will pay more to keep my heat on. And if you try to offer my wife free health care to leave me, his company will offer her better health care to stay.”

“What? How did you even know to buy such an insurance?”

“Well, part of it was just a ‘why not?’ sort of thing. The odds of the situation ever happening are so astronomically low that the insurance was incredibly cheap – a few cents per year. Also, the mere existence of the insurance prevents rich people from starting bizarre revenge schemes against the people in it, so they can afford to assume they will rarely have to pay out. I guess the price was so low that it was a no-brainer.”

“But what about transaction costs? Why would you even think to look into such a product?”

“Well, this is going to sound weird, but – I was reading a history book a few months ago, and – you remember that time a time traveler appeared in the middle of the Constitutional Convention, making some kind of point about how democracy wouldn’t work in a village of evil cannibals? Don’t you think that was pretty weird?”

“I had thought it was just one of the many colorful, larger-than-life stories from the Revolution. Like how George Washington was unimpeachably honest, or Benjamin Franklin always had a witty saying ready, or how Alexander Hamilton always spoke in rap.”

“Yeah, I used to think that too. But then I was reading a political science book last month, and – well, isn’t it weird that we’re a perfectly libertarian society? All of the old political philosophers used to say that libertarianism was an ideal system that could only be approached, never reached, and that even the approach would take a dedicated and virtuous population to pull it off. And our population isn’t that virtuous – I mean, just the other day I heard on the news about an ethics professor who went on a violent rampage chopping the limbs off babies.”

“Oh yes,” said Mr. Nett-Worth. “I heard about that too. Terrible stuff, terrible!”

“But it really only clicked a few weeks ago, when these goons from McDonalds broke into my house on a no-knock drug raid and shot my dog, and then muttered something about how surely I couldn’t object to private coercion. And it just got me thinking – what if this whole world is just a thought experiment by a communist with a crappy understanding of political philosophy trying to weak-man libertarianism? And then I thought – frick, I better get some really good vengeful-rich-person insurance, like, right away.”

“I am so confused right now.”

“Well, most sources define libertarianism as a political philosophy emphasizing individual autonomy and skeptical of government intervention. Libertarians come to their position for a wide variety of reasons, including belief that bottom-up local knowledge makes better decisions than top-down absolutism, or that government intervention naturally favors the powerful, or that if you actually ask poor people what they want, it’s usually more money, not people taking choices away from them and treating them like children. A fraction of libertarians – I think a small fraction, though I can’t prove it – are also believers in a deontological theory of natural rights which emphasizes non-aggression as the fundamental moral principle. If you’re doing shoddy journalism aimed at inflaming people rather than enlightening them, you might try to tar all libertarians by identifying them with this subset.”

“How does that explain all the weird things going on?”

“Take the cannibal village. If for some reason you believe the Non-Aggression principle perfectly defines the moral outcome in every situation, it must be pretty devastating to learn it can lead to cannibalism. But if you’re a normal libertarian who just thinks of libertarianism as a political position, then it’s no worse than a supporter of representative democracy learning that representative democracy could sometimes lead to cannibalism – which of course it can. In fact, you should be happy to point out that a libertarian village is much more resistant to cannibalism than a direct democratic or monarchical one.”

“What about the ethics professor’s rampage?”

“If for some reason you insisted property rights were based on perfect axiomatized natural law, it might be pretty devastating to learn that moral philosophy can’t get that kind of precision. But you’re a normal libertarian who just thinks of libertarianism as a political position, then learning that you can’t perfectly axiomatize property rights is no more devastating than learning that you can’t perfectly axiomatize caring about the poor, or thinking torture is bad, or not hacking off babies’ limbs. You’re still allowed to care about these things for the usual reasons even if you can’t construct a perfect moral theory around them.”

“And what about the goons from McDonald’s?”

“If for some reason you believe that only the government can do anything bad, and private companies…look, I don’t even want to speculate on who exactly they’re trying to straw man here.”

“Not straw man. Weak man. There are some real libertarians who believe only the non-aggression principle matters.”

“Maybe, some of them. I think in general they believe there are moral values other than non-aggression – after all, many of them are Christian, and believe in all sorts of moral values – but they’re skeptical of the government enforcing them. Remember, it’s not always correct to insta-convert ethics into law. I think in general they believe both that it’s important for a society to be virtuous, and that the government compelling people to exhibit more virtue than they possess can only go terribly wrong.”

“But there has to be some subset who don’t believe in virtue at all, and think the Non-Aggression Principle is literally all there is! And all these weird thought experiments show they’re stupid, right?”

“I disagree with them but I’m hesitant to declare them stupid just based on a few experiments. I mean, I like to say ‘I’m against torture’, and I like to say this is a strong moral principle of mine and not just a maxim of convenience. But with enough effort, you could create a ridiculous ticking-time-bomb thought experiment in which being against torture led obviously and inexorably to horrible results. Have you proven that people who say they’re against torture are stupid? Or would you be willing to cut them some slack in this situation? And are you willing to cut the same slack to this tiny subset of fundamentalist Non-Aggression Principle libertarians? Thought experiments are a useful tool, but sometimes the best lesson to take from them is ‘things are complicated but principles still matter’.”

“But surely, somewhere, there are incredibly stupid libertarians who think morality consists of the Non-Aggression Principle and nothing else, don’t believe in any other kind of virtue, and aren’t just holding it as a sacred but non-final principle the way you hold not torturing people?”

“Okay. Maybe there are. But how does it help to focus on this tiny pathological subset of libertarians and desperately try to convince the world that every libertarian is like this? There are some pretty pathological socialists too – should we demand everyone accept them as the only possible representatives of socialism? Should political discussion just be relentless weak-manning of the other side, with whoever is more simplistic winning the victory?”

Before Mr. Nett-Worth could respond, the bar we were in started to shake. “What’s that?” he asked me. “What’s going on?”

“A disturbance in the Farce,” I said. “This world was created to provide stupid weak-man arguments against dumbed-down versions of libertarianism. I guess what I just said – it threatened the fabric of reality itself. Hold on to your seat. This could get pretty bad.”

The bartender suddenly stood up. “All blue-eyed people need to leave the bar now!” he said. “As a proud bigot, I refuse to serve blue-eyed people. I don’t care how much profit it costs me! And also, this is the only bar in this city – nay, in a five thousand mile radius! Now no blue-eyed person will be able to go to a bar ever again!”

Scarce had he finished speaking when a very-finely dressed woman stood up. “I am a billionaire,” she shouted. “And I will give poor people money to humiliate themselves. Anyone who goes to the farm and rolls around in pig excrement for an hour, I will give ten million dollars! And many of you have terminal diseases that require expensive treatments, so you’ll die if you refuse! Mwa ha ha! Roll in pig shit! ROLL, YOU PEASANTS!”

But her jubilation was interrupted by another man, in the other corner of the bar. “I am a factory owner, and I am off to go sexually harass all my employees. There are no laws against it, so nobody can stop me. And I own the only factory in the world, so my employees can’t leave. And dire wolves eat anyone who tries to start new factories. So there!”

“You fools,” said a wild-haired man near the window. “I will cleanse this city of scum like you. Since there were no laws against making atomic bombs, I have built a nuke in my basement. Soon I will set it off in a great purification. And there’s nothing you can do about it until it’s too late, because there’s no law against owning nukes. Nobody can stop me! NOBODY!”

I took a deep breath. “I can stop you,” I said. “I can stop all of you.”

Every face turned to look at me.

“This world runs on dumb weak-man objections to libertarianism. The only way to fight them is with even dumber weak-man objections to libertarianism. So that’s what I’ll do. Nobody, blue-eyed or not, is going to leave this bar.”

The bartender scowled.

“Nobody, rich or poor, is going to go to the farm and roll in pig excrement.”

The rich woman looked skeptical.

“You’re not going to go your factory and harass your employees.”

The factory owner frowned.

“And you aren’t going to go set off your nuclear bomb. None of you are going anywhere!”

This entry was posted in Uncategorized and tagged , , . Bookmark the permalink.

522 Responses to Current Affairs’ “Some Puzzles For Libertarians”, Treated As Writing Prompts For Short Stories

    • vakusdrake says:

      Man if you thought this was long then this blog is not for you.

      • Toby Bartels says:

        No, I meant that the tl;dr was for my own comment.

        I just wanted to say that I laughed. But it took me three letters to say so unambiguously, and I felt like I should really only use one. (It's such a simple pleasure, you see.) So having droned on for three whole letters, I offered a summary, which I hoped will be understood in context.

        • Nick says:

          That is actually exactly how I took it, but not before wondering for a minute whether you thought one of the characters was from Death Note.

  1. TomA says:

    Does exercising your brain have the same evolutionary benefit as exercising your muscles?

  2. blacktrance says:

    Rand actually loathed libertarians and forbade her followers to interact with them, calling libertarians ‘a monstrous, disgusting bunch of people’, ‘intellectual misfits and scum’, and saying that the worst politicians of her age ‘are giants compared to anybody who would attempt something as unphilosophical, low, and pragmatic as the Libertarian Party’.

    And to top it off, it sounds “too much like a made-up word”.

  3. humeanbeingblog says:

    I loved every second of this.

  4. blacktrance says:

    Bryan Caplan had a good post along similar lines to II:

    If you’re a libertarian, you face what-ifs like this all the time. The point, normally, is to make you say, “Tough luck” and look like a monster., What puzzles me, though, is why libertarians rarely ask analogous questions. Like… “What if the government conscripts you to fight in an unjust war, and you die a horrible death?”…
    Why the double standard? The root, I suspect, is status quo bias. Most people tolerate the unpleasant ramifications of the status quo because they’re used to them. You might get conscripted and die a horrible death? Oh well, that’s life. Most people won’t tolerate the unpleasant ramifications of libertarianism because they’re used to a world where government says, “We’ll never let that happen”. But what’s so great about that assurance, when it’s bundled with a long list of other evils that governments blithely tolerate – or actively commit on a grand scale every day?

    • themountaingoat says:

      The reply is extremely obvious. People are not advocating for government in general they are advocating for specific types of governments.

      A person who was actually in favour of governments of whatever type would be vulnerable to similar hypothetical suggestions.

      • Salem says:

        Phew! Good thing the real-world governments they advocate for never fight unjust wars – otherwise those people would look really foolish.

        • I think there is a more fundamental issue. People routinely advocate consequences rather than institutions. They thus evade the question of whether there exist institutions that produce their desired set of consequences.

          The most obvious example is income redistribution. It’s pretty easy to argue that the deserving poor should get a subsidy at the cost of rich people who have more money than they have any real use for. It’s much harder to argue that a political system in which taking from A to give to B is one of the things the government can do will produce that outcome and do it without producing other and undesirable outcomes.

          Part of the point of public choice theory is to analyze political institutions not in terms of what we want them to do but in terms of what we can expect them to do, what outcomes they will tend to produce.

        • themountaingoat says:

          People could easily argue that unjust wars happen because people don’t correctly follow the principles they should be following.

          On the other hand the examples of bad things happening under libertarianism happen even if we follow the principles being advocated for.

          • Notsocrazy 24 says:

            I feel like you’ve still missed the point.

            Arguing for NAP/libertarianism is analogous to arguing for another system of government, not a system of morals.

            So obviously we could come to cannibalism through libertarianism, but also through direct democracy (51% vote for it), autocracy (the autocrat says so), a system like ours (Congress, the President, and SCOTUS agree and no one votes them out I guess), or communism (someone’s ability amounts to being cannibalized, and everyone else’s need amounts to cannibalizing him). Everyone would still supposedly be following “the principles they should be following” (autocrats give orders, communists take from each according to their ability and give to each according to their need), but with horrible consequences (cannibalism).

            Actually, I don’t think it’s that easy to argue that, say, the U.S. fighting in unjust wars is because people aren’t following the principles that they should be, except in a way that allows for libertarians to argue the same violation of principles anytime something bad happens in Hypothetical Weak Man Libertarian Land.

      • Alex Zavoluk says:

        I was going to say something snarky about how government programs are more likely to produce unintended negative outcomes than libertarianism is to produce the behavior in the types of arguments above, but David Friedman said it better than I could have.

    • Yosarian2 says:

      I mean, you could make the exact same argument in reverse and claim that the only reason people tolerate the unpleasant and constant real-life ramifications of capitalism (like “what happens if your boss fired you for an unjust reason and as a result you end up homeless and in the streets?”) is because of status quo bias.

      Or, you could be a little more charitable and just assume that most people who support democracy and most people who support capitalism (and most people who support both) understand that there are both advantage and disadvantage to each system, and just think that the advantages significantly outweigh the disadvantages in the large majority of cases, and that dismembering either system would almost certainly do far more harm then good.

  5. Whitedeath says:

    Ayn Rand is/was pretty influential in the libertarian movement, even if she herself wasn’t a supporter.

  6. Tracy W says:

    Awesome! Except I can’t work out why the cartoon guy is wearing two pairs of sunglasses.

  7. GPPrior says:

    I’m sorry, but much as I usually love your writing, here you’ve totally missed the point of the article you’re attempting to parody. You’ve picked apart the fine details of each hypothetical case, while ignoring the broader point that each hypothetical is meant to illustrate.

    I. There is such a thing as effective coercion without any actual application of force.

    A real-world example of this would be that someone who is forced to work an extremely dangerous job because the alternative is starving to death might reasonably be considered a slave.

    II. The source of coercion is not relevant inof itself. What is relevant is the effectiveness of that coercion.

    Your own story made this point very well. It is not inherently important whether McDonalds or the government is attempting to coerce you, what matters is that the government can kill you. The government has greater coercive power, but government coercive power is not inherently privileged other than its scale.

    III. Property rights are not, as many libertarians claim, a natural law.

    I’m not even really sure what your parody is getting at here. You don’t seem to disagree with the point that the original hypothetical is clearly making.

    IV. Allowing too much power to accrue in the hands of one person is inherently dangerous even if the legal system is maintained, because that person has the potential to act in ways that are not normally possible.

    The Infinitely Rich Man is a fantastically unlikely edge case. He’s supposed to be an fantastically unlikely edge case. That’s the entire point — that any one edge case almost never happens, but edge cases happens every day, and if you allow a large amount of power to accrue in a small number of hands the severity of such incidents will increase.

    • Evan Þ says:

      I. That is correct, and that is why I am no longer a doctrinaire libertarian.

      II. Yes, but government coercion is still , by and large, greater and thus more dangerous than McDonalds coercion.

      III. Scott’s saying that their point doesn’t matter; we can still advocate libertarianism as much as we think good on a practical level even if it isn’t a theoretically all-encompassing philosophy.

      IV. Yes, edge cases happen. But Scott’s pointing out that we can buy insurance against them. In the same way, getting my new condo broken into is an edge case – but edge cases happen, so I just bought insurance against it.

      • vV_Vv says:

        Yes, but government coercion is still , by and large, greater and thus more dangerous than McDonalds coercion.

        This is only because the government uses coercion to mostly prevent McDonalds’ from using coercion.

        • TheWorst says:

          This seems to be the knockdown argument against libertarianism in general and Ayn Rand’s version in specific–What stops the Infinitely Rich Man from hiring someone to shoot you in the head?
          And in Libertarian World, in which the government can’t coerce the rich in any way, what stops him from doing it there? After all, it’s his money, he gets to spend it how he wants. And stopping either him or the assassin would be government coercion, which is the only thing not allowed here.

          But definitely tell me more about how terrible it is that the government (sometimes) still stops rich people from indulging their every impulse.

          I get that you’ve noticed that McDonald’s doesn’t have death squads. What I don’t get is that you don’t seem to have noticed why McDonald’s doesn’t have death squads. Especially since “in a country where nothing stops Coke and Nike from having death squads, do they in fact have death squads?” is a question that has very much been answered, and the answer is “yes.”

          Infinitely Rich Man will absolutely have you shot the second that this option starts seeming lower-cost to him than the option of just putting up with you having offended him. That is because he is a human.

          • Thegnskald says:

            Question:

            Is failing to understand other people’s political positions a point of pride to you?

            Because that might be a decent argument about some variants of anarchy, such as anarcho-capitalism, but completely fails to describe libertarianism at all.

          • John Schilling says:

            What stops the Infinitely Rich Man from hiring someone to shoot you in the head?

            The same thing that stops the Infinitely Powerful Tyrant or the Infinitely Charismatic Demagogue from having you shot in the head – the fact that there is no such person. Given any proposed political system or philiosophy, I can identify a quality which, if a person holds it in infinite quantity or degree, lets them kill with impunity. Now, shall we talk about protection against real, finite threats?

          • Toby Bartels says:

            And stopping either him or the assassin would be government coercion, which is the only thing not allowed here.

            That is not Ayn Rand's version of libertarianism. This might pass as a straw caricature of anarchocapitalism, or, suitably rephrased, as a legitimate question to ask of anarchocapitalists. But you're not talking about Rand at all.

          • SEE says:

            This seems to be the knockdown argument against libertarianism in general and Ayn Rand’s version in specific–What stops the Infinitely Rich Man from hiring someone to shoot you in the head?

            Er, in Ayn Rand’s version in specific? The government.

            See The Virtue of Selfishness, chapter 14 “The Nature of Government”, where she details the need for a civilized society to have a government, defined as “an institution that holds the exclusive power to enforce certain rules of conduct in a given geographical area”.

            And perhaps you should refrain from commenting on the beliefs of Ayn Rand in the future? At least until you, you know, have some vague idea about what they actually are?

          • A1987dM says:

            The same thing that stops the Infinitely Powerful Tyrant or the Infinitely Charismatic Demagogue from having you shot in the head – the fact that there is no such person.

            Tell that to e.g. Holocaust victims.

          • moonfirestorm says:

            Tell that to e.g. Holocaust victims.

            And the fact that Holocaust survivors exist should make clear the distinction between Tyrant and Infinitely Powerful Tyrant.

          • Cliff says:

            You are saying that Hitler’s Germany was a libertarian paradise?

          • moonfirestorm says:

            Me? Not at all.

            My point is that Hitler was, thankfully, not an Infinitely Powerful Tyrant, and was eventually stopped through use of more power than he had.

          • sharper13 says:

            What stops the Infinitely Rich Man from hiring someone to shoot you in the head?

            The standard and obvious answer for an anarcho-capitalist society (which is different from a libertarian one where there is still a monopoly government for some things) is that people prefer not to be shot in the head by anyone, rich or not, so they pay for a service to either protect them from being shot in the head, or else at least revenge themselves on someone who shoots them in the head (or hires someone else to do it), thus discouraging that sort of behavior. No one has decided you aren’t allowed to defend yourself, after all.

            In terms of size, either a consortium of such providers making voluntary agreements similar to how insurance companies do today is sufficient to defer rich man, or else he’s so rich that he can do anything he wants regardless of political system because he controls everything and everyone already. (Not suggesting that would ever actually happen, BTW)

            In other words, in ancap theory, your much more likely to be shot by someone working for a monopoly government somewhere (who behaves in many ways like the theoretical rich man does already), than by a private individual who has managed to get that rich and powerful based only on voluntary transactions with people, i.e. doing stuff to benefit them.

            Ancaps aren’t generally against government, nor provision of services governments tend to provide, they’re against monopoly government, preferring competition for their business.

          • TheWorst says:

            So if I understand John Schilling’s point, he’s claiming that it’s literally impossible for powerful people to have less-powerful people killed (for reasons unstated; I assume this is the normal “Free Market Jesus wouldn’t allow that” faith claim).

            This is the most John Schilling post I’ve seen since the time he spent a few days vigorously pretended to believe that a politician having an email server was history’s greatest crime, and asserting that he was not lying about his belief.

            Then Pence turned out to’ve done the same, to no complaints from Schilling, demonstrating that he is in fact a doctrinaire mindkilled Red Triber–and a liar to boot–and that one therefore loses nothing by ignoring him.

            @moonfirestorm

            My point is that Hitler was, thankfully, not an Infinitely Powerful Tyrant, and was eventually stopped through use of more power than he had.

            This is not a very effective argument supporting the libertarian position that we should make it so there is no one with more power than Rich Dude.

            @sharper13

            The standard and obvious answer for an anarcho-capitalist society (which is different from a libertarian one where there is still a monopoly government for some things) is that people prefer not to be shot in the head by anyone, rich or not, so they pay for a service to either protect them from being shot in the head, or else at least revenge themselves on someone who shoots them in the head (or hires someone else to do it), thus discouraging that sort of behavior. No one has decided you aren’t allowed to defend yourself, after all.

            This is called “war.” You may have heard of it. It is not something you want more of. It is also something the rich faction wins, which is why you’ve completely failed to provide an answer to “what stops Rich Guy from having you shot.”

            The rest of it is the standard “Butbutbut free market jesus wouldn’t allow that because he’s omnipotent and omnibeneficent!”
            Sorry, I don’t share your religion. Faith claims aren’t persuasive for atheists, and right now you’re arguing “The Holocaust couldn’t have happened because Catholic Jesus is friendly!”

            It did happen, it does happen, therefore your claims that it’s impossible are false.

            Since your “standard and obvious answer” is both false and a faith claim, do you understand why it is not a very good answer? It’s like saying “It’s in the bible” when asked to support a factual claim; it’s unconvincing to anyone who doesn’t share your faith.

          • John Schilling says:

            So if I understand John Schilling’s point, he’s claiming that it’s literally impossible for powerful people to have less-powerful people killed

            The Infintitely Rich Person of the original hypothetical also had the characteristic of being an absolutely scrupulous practitioner of deontological NAP libertarianism. It is impossible for powerful people who meet that definition to have less-powerful people killed, at least by simple means like hiring someone to shoot them in the head.

            We can extend this to powerful but scrupulous holders of other political or moral philosophies; almost all of them are going to find “hire someone to shoot that annoying guy” to be off-limits.

            If someone has changed the subject to powerful amoral murderers, then I’m sorry I missed the change but that’s now boring and banal and not worth discussing. Wherever you go there are e.g. criminal kingpins who are very powerful and very amoral and can have people shot. Duh.

          • It is impossible for powerful people who meet that definition to have less-powerful people killed, at least by simple means like hiring someone to shoot them in the head.

            And it’s still possible within the NAP to do so by subtle means, when they have nobody positive rights.

          • Scott Alexander says:

            TheWorst is banned indefinitely. Really I can’t believe I banned so many other, apparently better, people first.

        • Evan Þ says:

          This is only because the government uses coercion to mostly prevent McDonalds’ from using coercion.

          That’s a very good thing for the government to do; unlike David Friedman, I want the government to keep on doing it.

          Meanwhile, the government is doing other things that’re much less good; let’s stop it from doing those things.

          • TheWorst says:

            You’re right on both counts.

            Sadly, of everything on this page, the idea that David Friedman would do something to reduce the power of the rich rather than increase it is the least-plausible of any of these hypotheticals.

            I mean, I have a lot of respect for Friedman; he’s obviously brilliant, a brilliant writer, by all accounts an excellent friend, and a man of deep and fascinating insight in any conversation that doesn’t offer any opportunity to argue that the only way to improve the world is to give more power to everyone who already has too much of it.
            The problem is that, well, he’s brilliant, so he can almost always find a way to argue for his overarching belief structure. And he does it very well.

          • He is the son of a preacher man…

    • Tracy W says:

      A real-world example of this would be that someone who is forced to work an extremely dangerous job because the alternative is starving to death might reasonably be considered a slave.

      Seriously? I’ve read Frederick Douglas’s autobiographies and I’ve read memoirs from the men who survived Gallipoli and I don’t see how anyone could reasonably consider that.

      The government has greater coercive power, but government coercive power is not inherently privileged other than its scale.

      And in related news, the proverbal 800-pound gorrilla is not inherently privileged other than its scale.

      • 1soru1 says:

        > And in related news, the proverbal 800-pound gorrilla is not inherently privileged other than its scale.

        Discretionary spending in US Federal budget 2015 1.1 trillion$, with power to allocate is spread over perhaps 500 people (senators, representatives, supreme court judges, the cabinet, agency heads, etc).

        Combined wealth of the 540 billionaires in the US 2016: 2.3 trillion$.

        That’s wealth versus spending, but still, it didn’t used to be that way; the US government used to have the discretionary spending to put a man on the moon simply because the President said so, and the idea of a single man running a private space program to complete with that would have been considered outlandish.

        In 2018, even the President will struggle to spend 30 million on a whim (the estimated cost of Trump’s military parade). There are 540 individuals in the US who could hold a bigger and better parade with less effort.

        There isn’t a lot of scope for a smaller government without it being the case that a small group of billionaires would start to fancy their chances in a conflict against it. With Putin/Trump versus the FBI there is an arguable case this is already happening.

        • Toby Bartels says:

          In 2018, even the President will struggle to spend 30 million on a whim (the estimated cost of Trump’s military parade). There are 540 individuals in the US who could hold a bigger and better parade with less effort.

          In 2018, the President is himself one of those 540 billionaires, so maybe he's going about this the wrong way.

        • Tracy W says:

          Wow, you really had to stretch, didn’t you, to try to make this argument?

          US federal government total spending is over $3 trillion a year. Speculating that a small group of billionaires might start to fancy their chances in a conflict with the US government is insane. Unless by ‘conflict’ you mean a legal one, which does come down to the success of liberalism in constraining government power.

          • 1soru1 says:

            Yes, the current balance of power, as approximated by financial numbers, is such that when a Russian PMC recently went up against the official US armed forces is Syria, they got thoroughly beaten,

            And it seems like the FBI’s lawyers are better-resourced than Trump’s lawyers, so the most likely he won’t be able to get away with having clearly committed crimes, but being too expensive to successfully prosecute, bribing or threatening the judges, or whatever.

            The insane thing is to think that that is something that would remain the case independent of the actual balance of power. Even the tax cuts already passed have already radically changed the numbers everyone is quoting. And whatever the details of libertarian philosophy, I don’t think you get to count yourself as a libertarian unless you are in favor of further massive tax cuts.

    • Scott Alexander says:

      I think you’ve missed my broader point, which is that this is attacking a specific rare libertarian paradigm and claiming it represents all libertarians.

      I think nitpicking the definition of “coercion” is a sign that you’ve gotten in a stupid definitional debate which has nothing to do with what you really care about, and you should take a step back and look at the real world. Especially when you are so desperate to win the debate that you stretch words so that you can be “coercing” someone by strictly offering them more options than they had before.

      I think that no morality can fairly be described as natural law deriveable from first principles. So, for example, you can’t prove it’s a natural law that you should care about the poor, but you also shouldn’t do what the Professor does and stop caring about the poor when you learn that. Instead, you should have a better concept of morality than “the things that I can prove are natural law”. To point out that one particular thing, like property rights, isn’t natural law – is an isolated demand for rigor.

      I agree edge cases are worth thinking about. As far as I can tell, the insurance solution correctly solves the Rich Man edge case. The reason nobody in the real world offers this insurance is that everyone knows this edge case never happens in real life. The proof of this is that it is completely legal today, whether society is libertarian or not, but you never hear about it. Also, I think the narrator’s torture example is a fair one. There are edge cases where torture is good, and their rarity is precisely the reason we believe torture is bad.

      • ChelOfTheSea says:

        > As far as I can tell, the insurance solution correctly solves the Rich Man edge case. The reason nobody in the real world offers this insurance is that everyone knows this edge case never happens in real life.

        But it does! This hypothetical is basically Scientology personified as a single dickhead instead of an organization devoted to dickheadery.

        • Alethenous says:

          Isn’t the Scientologist modus operandi throwing lawyers at people and manipulating the apparatus of government?

          • TheWorst says:

            Nope! They’re quite happy to use non-legal methods, in every instance where they can get away with it.

            Wiser people would think this is a good reason to make it so there are fewer circumstances where they can get away with it, rather than more.

      • fion says:

        …stretch words so that you can be “coercing” someone by strictly offering them more options than they had before.

        In a world populated by people who aren’t idealised game-theory machines, this is not a stretch at all. You can easily coerce people by giving them more options than they had before. Just in case we’re worried about the definition of coercion causing problems, I will say this: you can make people’s lives worse by giving them more options than they had before.

        • Anaxagoras says:

          Also, it’s quite possible that adding options makes things worse for everyone. Granted, it’s a somewhat contrived example, but look at Braess’s Paradox.
          https://en.wikipedia.org/wiki/Braess%27s_paradox

          Arguably, this is the mechanism of Moloch — offering options that are better than the alternatives in the moment, but which eventually result in the destruction of values, and from which individual abstention is increasingly disincentivized.

        • Cliff says:

          “You can easily coerce people by giving them more options than they had before.”

          How?

        • negativez says:

          1) The definition of coercion includes the qualification ‘by use of force or threats’.
          2) Scott qualified with ‘strictly’.

          You can’t *strictly* give someone more options with coercion because coercion, by definition, means you include ‘or else…’. Sure, you might be adding more options, but it adds the ‘or else’ to every former option, lessening them, which makes the situation non-strict.

          Scott’s Infinitely Rich Man would specify his ‘or else’ as “I will buy the provider of whatever option you do choose and shut it down”. This is pretty clearly lessening the value of all other options to zero, so it’s definitely coercion.

          I think the question to ask is how often someone actually brings up a *strict* scenario in actual debate on Libertarianism and calls it coercion.

          More relevant to real policy is when everyone chooses the strictly new option and the old option dies out, so now the town only has one store: Wal-mart. They were never coerced but now they have no choice left. Moloch-force is the analogue of coercion on a society.

          • anonymousskimmer says:

            More relevant to real policy is when everyone chooses the strictly new option and the old option dies out, so now the town only has one store: Wal-mart. They were never coerced but now they have no choice left.

            Real world, 95% of the population chooses the new options and the other 5% are screwed when the old option fails (as the new option has a serious defect for those 5% that the 95% consider a positive – e.g. various additives in personal care products which sweep an entire product line that was the go-to product line for the 5%).

          • Toby Bartels says:

            Indeed, but it can be bad even if literally everyone prefers the new thing, if the new thing stops being so good once all of the competition has been driven off.

          • and the other 5% are screwed when the old option fails

            If you are talking about products, 5% is easily enough to keep a product on the market. There are lots of things much more specialized than that.

            I recently purchased (via Indiegoogo, have not yet received) a Gemini, a cell phone modeled on the old Psion 5 PDA. It’s strikingly different from standard cell phones in a way I had long been wanting, having been very fond of my old Psion–a miniature laptop with a keyboard you can actually type on. I don’t have actual sales figures, but the first production run was for about a thousand units and I’m guessing total orders of about ten thousand. That’s less than 1/100,000 of total smartphone annual sales.

            Obviously the point is less true when the relevant population is much less–if we’re talking about a town which had three stores, one of which went out of business because most of the inhabitants preferred the other two.

          • anonymousskimmer says:

            If you are talking about products, 5% is easily enough to keep a product on the market.

            5% is a handwaving figure substituting for a non-zero % of the purchasers of a product who cannot make the switch. It does not mean 5% of the total populace.

            5% of a product line isn’t enough to keep a manufacturer from switching products, which you’d know if you read product reviews of people who state that X product is the only product they could find to use and now X has been re-formulated by the company. The manufacturer is not going to spend the overhead to keep making a particular formulation at a particular price point which is affordable to the typical person who can’t find an alternative to that product. How do I know this? Because I have seen such product reviews begging the manufacturer to keep making a discontinued product or formulation.

            Bespoke and small batch alternatives available from, e.g., Etsy, are often more pricey and not comparable, though they do meet some people’s needs.

            Obviously the point is less true when the relevant population is much less–if we’re talking about a town which had three stores, one of which went out of business because most of the inhabitants preferred the other two.

            Thank you.

            P.S. Smartphones are particularly high-cost, high-margin items which are often made in small batches for the well-to-do. I am not the poorest person posting to SSC, but you, Mr. Friedman, are far richer than my household. It’s not surprising that your mileage varies.

        • Sniffnoy says:

          At the risk of getting into an argument over words: That seems like a very misleading and noncentral use of the word “coercion”. While it certainly may be possible to make people worse off by offering them more options, I would avoid calling such a thing “coercion”. That’s just going to confuse people. If you really think such a thing is similar enough to coercion to be discussed in the same context, I would suggest coming up with a new term for the broader category that includes both, such as, say, “soft coercion”, or “subcoercion”, or “generalized coercion”, or whatever. Just mashing them together as both “coercion”, however, is the route to confusion.

        • fion says:

          @Cliff @negativez @Sniffoy

          Thank you for your comments. I guess our difference might be that I consider aggressive bribery to be essentially a form of coercion. I accept that this doesn’t fit the dictionary definition that negativez quoted, but I do think it falls in the same region of thingspace.

          If I say “do this thing that you really don’t want to do and I will give you lots of money” then I am strictly giving you more options than you had before but I’m forcing you to make a choice that you are likely to regret whatever you pick. (@negativez, feel free to reword this as “do this thing *or else* I won’t give you lots of money”.) This becomes even more powerful if there’s some kind of “natural” force or threat, for example if you’re really poor and my money could put food on the table, keep the bailiffs from your door and pay for your medical bills for some time.

          My offer is equivalent to saying “do this thing you don’t want to do or I will make you hungry”. I am sure you agree that this is a threat and therefore coercion (even by a strict definition); I imagine what you disagree with is the equivalence?

          • My offer is equivalent to saying “do this thing you don’t want to do or I will make you hungry”.

            No. It is saying “or I won’t stop you from being hungry.” You were hungry already and if I didn’t make the offer would remain hungry.

            You might consider that, following out the logic of your “make you hungry,” you are personally responsible for making millions of people hungry and killing quite a lot of them. There are, after all, hungry people whom you could have helped and didn’t. That is the consequence of refusing to distinguish between the results of action and the results of inaction.

          • fion says:

            @DavidFriedman

            I agree with your logic. This is why I donate to charity.

          • @fion:

            This is why I donate to charity.

            But do you donate to charity as much of your income as would be implied by the view that you are as responsible for consequences of inaction as of action?

            I conjecture that if you were offered the opportunity to safely murder a random stranger and be paid ten thousand dollars to do so, you would decline. Does it follow that as long as you believe that charity saves at least one life per ten thousand dollars you will donate everything you earn beyond the amount needed to keep you alive and earning?

          • fion says:

            @DavidFriedman

            I think it’s important to make the distinction between “what I would do” and “what I think is the right thing to do”. I’m not perfect, so I don’t do the right thing a lot of the time.

            I also think it’s important to make the distinction between people’s feelings and their material situation.

            I’ll try to illustrate this. There is an amount of money for which I think a case could be made that it would be morally correct to kill somebody. The money gained could be used to do more good than the harm that was caused by their murder. I don’t know if it’s $10,000, and I’m not particularly interested in trying to figure out exactly how much it would be, but hypothetically, that number must exist.

            However, I would not do it because I have strong emotional biases against murder. (My morality might not distinguish between active and passive killing, but my emotions certainly do. They also distinguish between pressing a button knowing that it will cause somebody’s death and actually having to shoot them at point blank range myself, but my morality doesn’t.)

            Similarly, I know I could do more good by donating more money to charity, but that would be hard, and I’m not perfect.

            My second point is a little bit more subtle, but I think it’s at the heart of this. I think some things that look equivalent on paper feel very different, and I think feelings matter. To use your example of the hired murderer, I imagine that DeadBob’s grieving family would be more upset by DeadBob having been murdered than if he’d, say, died in a car accident. The feeling of somebody doing something to hurt you is an unpleasant one.

            Out of interest, if you think there is a difference between active and passive actions, would you abstain in the trolley problem thought experiment, letting five people die rather than actively killing one person? (Or perhaps the ratio of 5:1 is a bit large. Feel free to adjust it. I think the spirit of my question is clear.)

          • @fion:

            I’m an intuitionist–my moral theory is an attempt to give a consistent account of my emotional reactions. So the distinction you are making exists for me only in a weak sense–thinking through the moral logic might convince me to rethink my emotional response.

            So far as the trolley problem is concerned, my view is that violating rights is a bad thing to do but not infinitely bad, so if I can produce a large enough benefit in terms of other values by violating a right I ought to do it. For five people probably not, for a hundred thousand probably.

            But action and inaction do not feel morally equivalent to me or, apparently, to you, although I gather you believe they are morally equivalent.

            Suppose there is someone who, without your intervention, will die–as there probably is. Consider two alternatives:

            1. You do nothing. He dies.

            2. You offer to save him, provided he does something for you that is unpleasant, but much less unpleasant than dying. He agrees and lives.

            As I understand your position in case 1 you have not coerced him, in case 2 you have. In which case have you behaved worse, as judged by your moral system? As judged by your emotional reactions?

          • Sniffnoy says:

            Fion:

            (Sorry, this is a direct reply to your comment above that addresses me, not to your thread with David Friedman… curse these limited levels of nesting.)

            I think you are thinking about things in a pretty useless way; the important question is not “how should we group concepts”, but “what should we do”. Saying “this is situation is coercion, and therefore we should do what we do in cases of coercion” is just reasoning by categories (to link our host’s old blog 🙂 ). I think here it’s better to dissolve the question of whether it’s “coercion” (which is purely a question about words) and just directly address the question of what should we do — and the answer to that is quite dissimilar to the answer of what we should do in the case of coercion in the strict sense.

            Before I elaborate on the above, I want to point out that I also think you’re equivocating on your claim. Above, you stated:

            In a world populated by people who aren’t idealised game-theory machines, this is not a stretch at all. You can easily coerce people by giving them more options than they had before. Just in case we’re worried about the definition of coercion causing problems, I will say this: you can make people’s lives worse by giving them more options than they had before.

            However, your post above does not do anything to support this claim (nor, apparently, is it using the same notion of “coercion”, although once again that’s not something we should focus on). It contains examples of giving people more options that you would label “coercion”, but it contains no examples of giving people more options in a way that makes them worse off. (Unless you are going to make a claim such as, making such offers induces people to do the thing they don’t want even though in fact starving is the better option; that kind of strains credibility, but in any case, you have not even made such a claim.)

            Anyway, returning to the original point. The relevant question is not “Is this something we should call coercion?”, the relevant question is “What should we do about it?”

            Coercion in the strict sense is, for fairly obvious reasons, something we very much want to discourage. We want to prevent it or punish it.

            The example you have provided, however, is something we very much do not want to prevent or punish, because its occurrence makes everyone involved better off, or, at least, no worse off. (Again, unless you are going to make an argument that such offers induce people to choose wrongly, or something similar.)

            Now, you may say, ah, but the rich man should do more! He should be willing to give his money unconditionally! Well, perhaps he should. But if he isn’t, then it’s just not really a relevant consideration, is it? Given that making such offers — which, again, leave everyone involved no worse off — is the most he is willing to do, it’s important that we not prevent him from doing so.

            The old “Copenhagen Interpretation of Ethics” piece is relevant here. Think about the laws that this way of thinking tends to lead to. If you do X, you must also do Y. (If you pay someone to do something for you, you must pay them at least a certain amount. If you build a large building, you must include works of art.) Well, guess what? There’s more than one way that conditional can be satisfied.

            Punishing or preventing things that are helpful-but-not-maximally-so is simply a bad thing to do. You can call such things “coercion” if you really like, you can argue whether it’s the rich man who’s coercing or whether really it’s nature that’s coercing and the rich man is off the hook for coercion; but really none of that matters, because regardless of what you call it, the proper thing to do regarding such situations remains the same, which is to allow it to occur. (…and for that reason I wouldn’t call it “coercion”, since that would mislead people as to the appropriate action to take, but… 🙂 )

            Now, there is one other thing worth mentioning. We could also just forcibly take some amount of money from the rich man and give it to the beggar. I’m not a libertarian; I don’t consider that entirely unacceptable. But I want to make several important points regarding doing such a thing:

            1. Doing this is substantially different from preventing or punishing deals that you consider “coercive”. For one, it doesn’t have an antecedent that can be negated.
            2. If you’re doing this, it’s important that it not in any way be conditional on the rich man having interacted somehow with the beggar; you have to take from every rich man and give to every beggar. Because otherwise we’re back in the situation above, with the Copenhagen Interpretation of Ethics, where we’re just incentivizing the rich man to avoid beggars. (This is really just a repeat of the above point, but it’s worth repeating.)
            3. One additional advantage of direct redistribution over banning “coercive” deals is separation of concerns. Now, you didn’t make an argument of this form, so this isn’t an argument against anything you’ve said, but I do still want to mention this since people so often do — basically, people often propose such conditional-style laws as ways of meeting multiple goals at once, and the result is that it does them all badly. If you’re trying to accomplish multiple things, better to do them all directly and separately, rather than trying to multiple things together.
            4. Finally, doing this does not in any way change the logic above should such “coercive” situations occur regardless.

            (In addition it is worth noting that redistribution does have downsides, which I won’t go into here, but are worth remembering; and that attempting to redistribute things other than money can be seriously destructive and should be avoided.)

            So, I am hoping that clarifies my position on such matters. 🙂

          • fion says:

            @DavidFriedman

            I will read up on intuitionism. I’m not familiar with it. I think it might not be the best use of our time to get too deep into it, since it would probably just end up being me saying “but what about [obvious apparent flaw in your philosophy]?” and you replying “[standard answer that I could have found on google]”.

            I mean heck, I don’t even see “rights” as things that exist in an absolute sense, but rather concepts that emerge as helpful rules-of-thumb from trying to achieve the best consequences by your actions.

            Actually, I do have one thing I want to ask. In my last comment I gave the example of the difference between shooting somebody in the head vs pressing a button that I know will cause them to die in another room. I feel a much stronger emotional response thinking about the first one, but I believe them to be essentially equivalent moral acts. Is your emotional response different for the two cases? Do you consider one to be more “immoral” than the other?

            To try to answer your hypothetical, I’ll start with my emotional reactions. I think I have a stronger negative emotional response to 2 than to 1. But I’m not sure if that’s just because I thought too quickly and I’m actually giving my “moral reasoning” answer. (The alarm was raised when I realised I was trying to justify my answer, which isn’t something I should have to do for an emotional response.)

            We’ve already agreed that “letting someone die” is something that people do all the time, whereas using the threat of death to make somebody do your bidding could have all kinds of complicated and unforeseen emotional effects on them. (Telling them “I could save you but instead I’m going to let you die” would probably also have complicated emotional effects on them, but at least they wouldn’t last very long.)

            I think the sort of question that I’d need to ask myself to come up with a proper answer would be to look at the next-to-leading-order effects. How grief-stricken will his family be if he dies? How horrified will his family be if he is forced to obey you for his life?

            I also think I have a problem with “less unpleasant than dying”. As an atheist, I don’t expect death to be unpleasant. I expect it to be the most neutral thing possible. What’s unpleasant is the pain caused by the sorts of things that kill you, the fear caused by contemplating your death, the grief caused when loved ones die etc. I would rather die than live a life of suffering.

            Just to be clear, yes. I do think that 2 counts as coercion. But I’m not sure whether in this case it’s worse than 1, for the reasons given above. (And others, but I don’t want to make my comment too long.)

          • fion says:

            @Sniffoy

            Arg, I agree with your curse!

            Also, that’s a really long comment, so I aplogise in advance for probably not giving an adequate response to everything you say.

            I completely agree that the definitions of words have nothing to say about the important question, which is, as you say “what should we do?”. In fact this was why I separated my two claims in my reply to Scott. I claimed both: “giving somebody more options than they had before can be coercion” and “giving somebody more options than they had before can be a bad thing”. I did not claim that “those things that can be described as coercion can be concluded to be bad things”.

            (By the way, the reason I referenced thingspace was to actively try to avoid falling into the trap you warn me of. The question I ask when I do decide to concern myself with the question “should I call this thing ‘coercion’?” is not “does this thing meet the definition of coercion?” but “does this thing fall in the same region of thingspace as things that are uncontroversially labeled coercion?”. In Scott’s old post, the examples are generally in a rather different region of thingspace from the things they’re being conflated with. MLK might meet the definition for “criminal”, but he is not in the same thingspace cluster as bank robbers. I make the claim that aggressive bribery is within the same cluster as all the things that you and I agree are coercion. So even if it doesn’t meet the letter of the definition, it is appropriate to use the same label for it. The labels should ideally reflect the cluster structure of thingspace, not the definitions we find in dictionaries. The reason I’m fleshing this out in detail is that you seemed to summarise my point as being about “how should we group concepts?”. It wasn’t. It was about “how are concepts empirically grouped by the properties they concretely have?”)

            Ok, I think you’ve reached the nub of it here:

            It contains examples of giving people more options that you would label “coercion”, but it contains no examples of giving people more options in a way that makes them worse off.

            I think I did give examples of giving people more options in a way that makes them worse off, if we allow “worse off” to include emotional as well as material effects. Having to choose between two unpleasant outcomes can be a very horrible experience. Interacting with somebody who seems cruel can be a horrible experience.

            Which is worse, somebody murdering your best friend or somebody telling you that they’re going to murder either your best friend or your second-best friend and you have to decide? There’s a reason the second one sounds like the plot of a horror film (well, there are two reasons, one is that it’s very unrealistic, the other is that) – it is much more horrifying than the first one. With the first one I can be like “the world is such a terrible place” but with the second one I’m like “what have I done?”

            Or to use the example I originally gave, I believe that being in the position of having to choose between [the thing you really don’t want to do] and [passing up the opportunity to make your financial life easier] is unpleasant in itself. In some situations it will be unpleasant enough to outweigh the difference in unpleasantness between the two options.

            I think the second half of your comment is mostly anticipating arguments that I do not actually wish to make. If you think I’ve missed something that’s relevant to the above, then please point it out to me and I’ll respond to it.

          • Sniffnoy says:

            Fion: Ha, that’s one more argument I had meant to say something about but forgot!

            But basically: I think what you’ve presented is a weak argument. The idea that having to make choices that one dislikes, or that interacting with someone who seems cruel, is sufficient to outweigh substantial material gain, is… not a claim I really find very credible. Moreover, even if we grant that it can happen sometimes, is that sufficient reason to prevent or punish such things? Is that a thing that an outsider to the deal can judge? If not, then, well, what are talking about it?

            As for labelling of things… things shouldn’t just reflect the cluster of structure of thingspace, they should reflect useful divisions in the cluster structure of thingspace, divisions we can act on. If you’re grouping two things as coercion in a way that will mislead people as to the appropriate response, I don’t consider that helpful. (Also, let’s remember why coercion is bad. It’s not because of the psychological pain of choosing.)

            (BTW, I should probably point out that you are misspelling my handle, which makes it a little harder to search…)

          • fion says:

            @Sniffnoy

            Sorry for misspelling your handle. I think I’ve been reading it wrong in my head every time I’ve seen it!

            I think your argument proves too much. If person A uses a threat of violence to ‘persuade’ person B to do something that is definitely in B’s material interests, then, by your argument, this sounds like a good thing? Never mind the stress that B feels at having their agency taken away from them, never mind A’s cruelty. What matters is that everybody ended up with more money/food/shelter than before.

            If not, then, well, what are talking about it?

            I wasn’t talking about what we should prevent or punish. I agree that some of the ways in which you can make somebody’s life worse by offering them more options than they had before should be legal. Perhaps some should be illegal, but I don’t know – I’m not a lawyer. The point I was making was that such situations exist.

            Also, let’s remember why coercion is bad.

            I’m starting to wonder if maybe we disagree on this. I think coercion is bad when it causes net suffering to the coercee because it causes net suffering to the coercee. (In some situations you would need to take into account other people, but most of the time the effects on the coercee will probably dominate.)

            Why do you think coercion is bad?

          • Sniffnoy says:

            Fion: Hoo, OK, finally got a chance to return to this.

            I want to focus on the main thing here which is the question of, what are we even talking about?

            I wasn’t talking about what we should prevent or punish. I agree that some of the ways in which you can make somebody’s life worse by offering them more options than they had before should be legal. Perhaps some should be illegal, but I don’t know – I’m not a lawyer. The point I was making was that such situations exist.

            OK. So what?

            Like, OK, agreed, paradox of choice situations exist. If that’s your whole point then we don’t have a lot to argue about (although I will continue to maintain that categories should contain actionable information and that grouping two things that merit entirely different responses under the single word “coercion” is a bad idea). The important question is what to do, and it seems we more or less agree on that. Although I think it’s worth noting that I’m not just talking about legality here. Again, see the Copenhagen article. Bad press, etc. It’s important not to punish such things and that doesn’t just mean by means of the legal system.

            (I should point out that by the way that your posts contradict each other and you might want to explicictly resolve which ones you’re sticking with. I mean, I guess you’ve kind of implicitly resolved it, but it would be good to explicitly resolve it. Specifically, in this comment, you state that you are using the word “coercion” in such a way that it is only coercion if the victim is made worse off, whereas in this comment you do away with that requirement. You seem to have gone with the latter but it would be good to explicitly resolve that.)

            As for the more minor points…

            I think your argument proves too much. If person A uses a threat of violence to ‘persuade’ person B to do something that is definitely in B’s material interests, then, by your argument, this sounds like a good thing? Never mind the stress that B feels at having their agency taken away from them, never mind A’s cruelty. What matters is that everybody ended up with more money/food/shelter than before.

            I think we need to dissect “good thing” here. “Good” in the sense of “what we should reward” is quite different from “good” in the sense of “what you should do” (although these two affect each other quite a bit for obvious reasons).

            Is it “good” in the sense that we should encourage people to do this, or refrain from discouraging it? Definitely not. We don’t want people going around trying to judge other people’s best interests for them and enforcing such with violence. We know how that goes.

            Is it “good” in the sense that you should do it in a realistic situation? Seriously unlikely. You do that, you’re marking yourself as the sort of person who, well, goes around trying to judge other people’s best interests for them and is willing to enforce such with violence. How do you think that is going to go for you?

            Is it “good” in the sense that in a thought-experiment scenario where this somehow has no other side-effects and everyone basically forgets afterwards that it happened (and you know this will occur) you should do it? Well… still no, because your victim may just say “well go ahead and stab me then because I’m not doing it” — which is, after all, the correct response, if they want to make it clear to any other bandits who may hear about this that they don’t give into threats. If however we make the further assumption (on top of everything above!) that you are sufficiently certain that they won’t do this and it will work out for the better — having basically handwaved away all the negative effects this would have in real life — then sure, I’ll bite that bullet.

            Why do you think coercion is bad?

            I mean, I’ve basically outlined this above. Basically I think we should generally go by the presumption that people are competent to act in their own interests, under which presumption coercion can be presumed to be fundamentally destructive (you’re not going to coerce someone into something if it was something they wanted to do anyway). I say “presumption” because of course this isn’t always true. But as a basis for action it’s a good one. So what happens when the presumption is false? Well, the problem is, how do you recognize such situations? I don’t think we can trust people to do that correctly. As such I think we can just recognize a heuristic that attempting to coerce others, even for their own good, is an act that should be punished. That doesn’t mean that every single instance of such an act necessarily makes the world worse off, but attempting to judge by that doesn’t make a whole lot of sense — in any case “what makes the world worse off” and “what we should punish” aren’t necessarily the same thing anyway. Even if there’s somehow magically one wise person who somehow knows how to properly deploy threats of violence to get people to do things that will improve their situation, actually doing that and it going unpunished would encourage other people to deploy coercion, and that’s not going to go very well; and if you successfully coerce the person into doing the thing that helps them, you’ve also revealed them as a good target for bandits, so likely you’ve harmed them on net after all!. (…I guess I’ve just undermined my point about these not being the same thing since I just argued that it does make the world worse off on net, but, you know, that’s dependent on external facts; in the crazy thought-experiment situation above, it doesn’t.)

            …that got a bit tangled. Hoping I’ve made my point fairly clearly?

          • fion says:

            @Sniffnoy

            Yeah, I’m wondering if maybe we should wrap it up. I feel as though we’ve gone past the “productive discussion” stage, but maybe I’m wrong.

            OK. So what?

            So the claims I made in my original reply to Scott are true.

            You are mistaken that my posts contradict each other. I apologise if the first one you link to wasn’t clear, but I was not using the word “coercion” in such a way that it is only coercion if the victim is made worse off. I already clarified this. (Sorry, I don’t know how to link comments…)

            In fact this was why I separated my two claims in my reply to Scott. I claimed both: “giving somebody more options than they had before can be coercion” and “giving somebody more options than they had before can be a bad thing”. I did not claim that “those things that can be described as coercion can be concluded to be bad things”.

            Both of my statements are true (so I believe), but they do not follow from each other, in either direction.

            Your argument seems to be “granted that both of your claims are true, but I think it’s very important to distinguish between ‘offering somebody something bad in order to make them do what you want’ and ‘offering somebody something good in order to make them do what you want'”. I agree that these are different sins, but they have in common that you are using your more powerful position to make other people do what you want.

            You are very interested in “do the different sins call for different actions?”. Yes, they do, but also different examples from each category require different actions. Telling your child “do you chores or you’re grounded” and telling a small country “give me all your gold or my army will shoot 10% of your population” are both examples of “offering somebody something bad in order to make them do what you want” but they invite very different responses from us!

            Thank you for answering my hypothetical so thoroughly. I’d like to take this opportunity to say that my much earlier hypothetical (which was aimed at exploring coercion-by-giving-more-options) is also “good” once we handwave away all the real-world details (but not otherwise).

            Why is “fundamentally destructive” a bad thing? Sorry if that sounds like a stupid question, but I feel as though there’s a gap between the things that we think are obvious.

      • meh says:

        As far as I can tell, the insurance solution correctly solves the Rich Man edge case.

        Why is the rich man allowed to buy the castanet factory, but not the insurance company? I think the solution is that no system can stop someone who has infinite god-like powers.

        • Why is the rich man allowed to buy the castanet factory, but not the insurance company?

          Private law? Oh, hang on…

        • Tracy W says:

          If he does buy the insurance company he’s still obliged to honour existing contracts.

          • meh says:

            if you had infinite wealth, would that end the matter for you?

          • Tracy W says:

            The hypothetical assumes the Rich Man is acting strictly to libertarian principles.

            If he isn’t, then there would be cheaper ways to wreck someone’s life than buying up companies all the time.

          • meh says:

            i have confidence the infinitely rich man could find a way and still adhere to strict libertarian principles.

            granted they would be as absurd as buying up companies, and not really arguments for or against libertarian principles.

          • TheWorst says:

            What Earthly mechanism would force him to honor contracts he didn’t feel like honoring?

            Note that in the real world, people reliably honor contracts only when a specific entity coerces them into doing so.

          • Note that in the real world, people reliably honor contracts only when a specific entity coerces them into doing so.

            On the contrary. Most people who honor contracts, for reasons other than feelings of moral obligation, are doing it to maintain their reputation for future contracts. There isn’t a “specific entity” coercing them. They are being coerced, on a very loose sense of the term, by their expectation with regard to the actions of lots of different entities.

            You seem to be using “coerce” to mean “give someone a strong reason to do something.” Is that a fair reading?

          • mjr says:

            Perhaps he could bankrupt the company, thus disrupting its ability to honor any contracts?

          • anonymousskimmer says:

            All he has to do is also buy (or contract) the mercenary company, pay it to kill you, and let his insurance company and the mercenary company duke it out. Eventually, after many dead mercenaries and security guards, someone will get you.

          • Tracy W says:

            @TheWorst, @anonymousskimmer, you are both fighting the hypothetical.

          • anonymousskimmer says:

            Yes, but theoretically we’re winning. 😉

        • benjdenny says:

          Forgive me if I’m missing the point, but couldn’t the infinitely rich man also just buy the government in a Democracy? I’m probably misreading the general room, but it seems like it’s veered more into a “how can we stop infinitely rich men” conversation rather than a “Libertarianism is uniquely unsuited to stop infinitely rich men” situation.

          • meh says:

            not if he had strict democratic principles. but yes he otherwise could. it is difficult to stop infinite power.

          • MugaSofer says:

            Depends on what those “democratic principles” are – he could start a campaign too convince people you’re evil, for example, so that they would vote for laws that harm you.

            … which feels a bit too close to reality for comfort.

          • TheWorst says:

            Correct. Even a moderately rich man can buy some of a government (as we’ve seen, over and over). A very rich one can buy more of it.

            This is an obvious argument for limiting how much of the local resources can be controlled by one person.

      • themountaingoat says:

        It isn’t an isolated demand for rigour because libertarians are saying that we should treat property rights as more important than many other moral principles. I haven’t seen an argument for that other than one from natural law.

        • Cliff says:

          Utilitarianism?

          • themountaingoat says:

            I haven’t seen a convincing utilitarian argument that absolute property rights are more important than other rights.

          • Cliff says:

            Moving the goalposts? First you say you haven’t seen an argument other than from natural law. Now you say you have not seen a convincing utilitarian argument. So is that an admission that your first statement was in fact false?

          • themountaingoat says:

            The two statements are not in disagreement.

      • Natural law keeps coming up , because if you try to argue against a libertarian on a consequentuialist or positive rights basis, they tend to insist that , no , coercion is always wrong, there are negative rights, and there are no positive rights, etc. Some kind of natural alw or absolute deontology an implicit basis that they can’t defend explictly (well, Rand tried).

        It’s not a fallacy to hold someone to the standards they set themselves.

        • MoebiusStreet says:

          Speaking for myself, I have argued against positive rights, but in a different way. When someone claims that people have “a right to healthcare”, “a right to a living wage”, etc., my first response is “how much healthcare do they have a right to? Who decides where that line is drawn?”. To my mind, a “right” whose parameters are defined by some third party is no right at all.

          My second response is one of those thought experiments. Suppose you’re on a desert island with just the folks that were on your (crashed) plane with you, and you’re lucky enough that there was a doctor on board, do you have the right to enslave that doctor, forcing him to provide care for the entire party?

        • Natural law keeps coming up , because if you try to argue against a libertarian on a consequentuialist or positive rights basis, they tend to insist that , no , coercion is always wrong, there are negative rights, and there are no positive rights, etc.

          People in this thread seem to regard anarcho-capitalism as the least defensible variant of libertarianism. I support anarcho-capitalism and I do not make that argument. My book The Machinery of Freedom, which I may immodestly report gets described as a libertarian classic, explicitly disavows that argument.

          I do agree that there are no positive rights, however. Avoiding the violation of rights is a good thing but it is not the only good thing nor an infinitely important good thing, so one does not have to believe in positive rights to believe that there may be (hopefully rare) situations in which one should violate negative rights.

          You may defend your statement by relying on the word “tend,” but at that point you have conceded that there are libertarians against whom your argument doesn’t hold.

        • @Moebius
          If rights aren’t built into the universe, they are nothing, or they are defined at the human level. From the point of view if contractarianism or constructivism , it’s perfectly OK to settle the kind and level of rights you will have by negotiation.

          @DF

          What does it mean to say there “is” a negative right? That kind of statement was what I meant.

          • If I say “I have a negative right not to be killed,” that, considered as a moral statement, means “if you kill me you are acting badly, and if I use force to prevent you from killing me I am not acting badly.”

            If I say “I have a positive right to be fed,” that means “if I am not being fed and you could feed me you are acting badly, and if I use force to make you feed me I am not acting badly.”

            That probably isn’t complete enough, but it should be sufficient to show what I think the words mean.

            I also think it is useful to talk about positive rights, but those are not statements about right and wrong.

          • Nearly Takuan says:

            Not a Libertarian. Not quite Marxist, but dating one. It shouldn’t matter, but it probably does, so I want to get that out of the way right now.

            Refer to the example above of “imagine you are stranded on an island and by chance there is a doctor in your party. Is she required to treat your injuries for free?” This seems to me a good illustration of a positive right. The Libertarian argument as I understand it is that if the doctor by chance happened not to be on your plane, or died in the crash, she would owe you nothing, and unless you personally are the reason she survived the crash (and we suppose for the sake of argument that you did not), the fact that she did survive should have no effect on what she owes you, so, therefore, in the possible-world where the doctor is alive and stranded with you, she continues to owe you nothing. (She may choose to treat your injuries anyway, of her own free will, informed by her own moral compass and the possibility that she needs something from you, but these are her decisions to make.)

            Meanwhile, coercing her to treat you by violent means infringes on her negative rights not to be threatened, not to be harmed, not to be murdered, etc. Your acquaintance Chad who was also on the plane and is currently threatening your crew’s only doctor has these same rights, but in this rare circumstance you may serve a greater good by violating his negative right not to be forcibly restrained, in order to preserve the well-being and good-will of another human being (especially one who is important to the local community).

            This seems perfectly coherent to me, and I can easily imagine ways to fill in any remaining gaps, edge cases, what-ifs, and just-sos by simply assuming some slight changes to the hypothetical scenario (and not at all ridiculous, like “surrounded by dire wolves and thousands of miles of darkness”).

            I am currently pondering what it is about this story that does not convince me to become a Libertarian. The best I can come up with at the moment is that I think there is a substantial difference between the decisions of an individual doctor, which I’m perfectly willing to assume on faith will usually involve some form of morality that I would generally agree to and call “good” on net, versus the decisions of, say, a “Big Pharma” company that will absolutely screw over individuals in order to make a profit, and can invent post-hoc justifications on the fly (including some that sound moral and principled under Objectivism/Libertarianism – at least as I understand it, which may be flawed).

          • publiusvarinius says:

            @Nearly Takuan:

            the fact that she did survive should have no effect on what she owes you

            I find these sorts of arguments very unconvincing. Every duty can be phrased in the form “if SITUATION then ACTION”, so it’s very hard to argue that particular situations cannot entail particular duties; and restrictions (“unless you personally are the reason”) beg the question by presupposing some libertarian principle.

            All in all, it’s coherent to believe that every doctor has the duty to assist fellow people stranded on an island for free, but some doctors do not exercise this duty due to circumstances beyond their control, such as not being stranded on an island or being dead.

          • Your acquaintance Chad who was also on the plane and is currently threatening your crew’s only doctor has these same rights, but in this rare circumstance you may serve a greater good by violating his negative right not to be forcibly restrained, in order to preserve the well-being and good-will of another human being (especially one who is important to the local community).

            I don’t think the “rare circumstance … violating his negative right” captures the moral intuition. Roughly speaking, I have a negative right not to be coerced means that it is appropriate to use force against someone coercing me, hence doing that doesn’t violate his negative right. Perhaps a better way of putting it is that my negative right is not to be coerced except when I am violating someone else’s negative right and coercion is the only way of preventing it. The point is sometimes put as the claim that it is wrong to initiate coercion.

      • Ethan C. says:

        I’m going to have to respectively disagree here, given that his idea of coercion through denial clearly exists in real life. If you have ever worked at a fast food chain, like McDonalds or walmart for another industry, you know what I am talking about. Many people who work there have little other choice, are on food stamps partly because those companies are able to lobby against raising the minimum wage (it’s estimated that walmart costs 6.2 billion in cood stamps according to forbes, no link because I’m on mobile- look for O’Conner 2014), and are coerced to do harmful illegal work because they have no other options (it’s hard to find info on this from fast food restaurants directly, but the National Council for Occupational Safety and Health did a survey of 1426 workers where over a third of the 80% injured claimed it was caused by pressure to work quickly).

        Libertarianism as a policy has been done before, in the late 1800s, and didn’t work then. We have no reason to believe anything has changed in favor of workers. Meanwhile we see clear examples of socialist policies (which are as far from weakman socialism as these questions are from libertarianism) working in parts of Europe, in countries that have the highest happiness in the world (UN world happiness report). As far as actual evidence goes the results favor government intervention in the economy. Remember the people doing this (the federal reserve in the US) aren’t politicians but trained economists.

        • quanta413 says:

          Libertarianism as a policy has been done before, in the late 1800s, and didn’t work then.

          Where? Give an example that isn’t “well, it sort-of kind-of had a few things in common with libertarianism”. You’re also going to need to show how it didn’t work. Maybe a collapse into civil war or famine. Something like that.

          • quanta413 says:

            Seriously? The example is so bad you fail to give even the slightest argument for why they should count as having a libertarian policy. You may as well have gone with “Nazis. Cue the ‘no true libertarianism’ argument of course.”

            The British East India company isn’t even vaguely close to libertarianism. They existed centuries before libertarians and then vanished about a century before libertarians were even a thing. They were a massive state-granted and financed monopoly with backing from Great Britain. If you want to argue that all libertarian government would collapse into what a completely different arrangement of power did (constitutional monarchy + company in alliance), then do it. No magical backdating. Just because the British East India Company was a company doesn’t mean it wasn’t also a government. And not a libertarian government at that.

            Seriously, the very page you linked mentions part of the causes being “exploitative tax revenue maximization policies of the British East India Company after 1765 crippled the economic resources of the rural population”. I was unaware that libertarians advocated high taxes. Libertarians also apparently must advocate violently obtained monopolies and constant violation of the non-aggression principle. Any argument for why all libertarian government will rapidly collapse into something resembling rule from abroad by aggressive foreigners? I’m not saying that’s impossible; I think the likely instability of libertarianism is one of the stronger arguments against it. Communism rapidly collapses into brutal dictatorships. But maybe you could make the effort.

          • SEE says:

            @1soru1

            Did you read your own link? It describes a case where a ruling body was, by armed force, imposing grain trade monopolies, a prohibition on ‘hoarding’, and increased land taxes.

            So, yes, ‘no true libertarianism’, in the same sense that Idi Amin was ‘no true Scotsman’.

            A much better example for you would be the continuing Irish Famine after repeal of the Corn Laws restricting trade in grain. The best that, say, the Mises Institute can come up with is that the underlying conditions were non-libertarian in cause, and the adjustment to free trade was accordingly harsh.

          • 1soru1 says:

            > Any argument for why all libertarian government will rapidly collapse into something resembling rule from abroad by aggressive foreigners?

            The proto-libertarian government in this case was Great Britain, so it is more like ruling foreigners abroad aggressively. If you don’t have the capability to regulate companies, then the companies will do the regulating.

            Saying ‘we are Maoists, not Leninists’ isn’t sufficient reassurance that anyone plotting a violent class revolution will have that turn out better than the other times it has been tried.

            Similarly, the doctrinal differences between modern Libertarianism and the Classic Liberalism of the Great Britain of the slave trade and East India company do exist, but they provide little reassurance that they would operate differently in similar circumstances.

            https://edition.cnn.com/2017/12/05/politics/erik-prince-private-spy-network-trump-administration/index.html

          • Similarly, the doctrinal differences between modern Libertarianism and the Classic Liberalism of the Great Britain of the slave trade and East India company do exist

            I can’t tell if you are aware that the classical liberals of 19th century Great Britain, in alliance with the Quakers and some other groups, were the people who ended the slave trade. Economics was given the name “the dismal science” on account of the opposition to slavery of the classical liberal economists.

          • 1soru1 says:

            I can’t tell if you are aware that the classical liberals of 19th century Great Britain, in alliance with the Quakers and some other groups, were the people who ended the slave trade.

            To the extent that Whigs were crucial in the abolition of the slave trade and later slavery in the British empire, it was precisely because they changed their positions from a pro-slavery one to neutral, under the influence of the moral rhetoric of the Quakers and other Christians. If you look at say the Constitution of the Carolinas, written by John Locke, you find:

            Every freeman of Carolina shall have absolute power and authority over his negro slaves, of what opinion or religion soever.

            That’s fairly unambiguous what it is talking about, wheras most of the quotes claiming to show Locke in opposition to slavery are actually using slavery as a metaphor for Catholic rule. At no point did any Liberal actually campaign against the slave trade using Liberal rhetoric; this would have been impossible, or contradictory, as it obviously inherently was a government banning things free men chose to do freely, i.e. tyranny.

            The one genuine Liberal contribution to the Abolitionist movement was Adam Smith’s thesis that slavery was less economically efficient than wage labour. Which was very likely true in the specific circumstances of the 18C Caribbean, but is far from a universal rule. Late 19C surgery, or early 20C pharmaceuticals, would easily suffice to make slavery at least as profitable as wage labour in niches, and it only needs one such niche to reverse a ban justified solely on those principles.

            John Stuart Mill is generally considered to have been the first major Liberal intellectual to write a full-throated condemnation of slavery, and the one that lead to the phrase ”dismal science’. And a very eloquent one it is too.

            Only, he did so in 1850, 17 years after its abolition.

            (seriously, read that letter; it would only need slight reformatting and some links to be a SSC blog post).

          • Protagoras says:

            The one genuine Liberal contribution to the Abolitionist movement was Adam Smith’s thesis that slavery was less economically efficient than wage labour.

            Hume advanced this thesis before Smith (not surprising Smith took it on board; Smith frequently endorses Hume’s theories).

        • Cliff says:

          Many people who work there have little other choice

          In that case, their employers must be providing them a really astronomical benefit, right? If the alternative is homelessness or death. If that’s coercion, don’t we want more?

          are on food stamps partly because those companies are able to lobby against raising the minimum wage

          Food stamps and other benefits raise workers’ reservation wages and make it so that companies have to pay more than they otherwise would. Walmart did not lobby for food stamps. Walmart’s pay starts above minimum wage in almost all places doesn’t it? And the idea that a higher minimum wage improves the lives of the most vulnerable is highly contentious, to say the least.

          • sharper13 says:

            I still haven’t figured out the people who in essence argue that lifeguards who throw someone drowning a life preserver is coercing them to grab on and taking advantage of them because they also get a salary to work as a lifeguard and throw those preservers.

            Just like the lifeguard didn’t create the waves, Walmart didn’t make the person poor and without the skills to get a better job and didn’t create the food stamp/welfare system. They’re just the ones who are throwing the life preserver to that group of them.

            If the OP actually believes that’s morally wrong, he’s free to go offer jobs which pay way more to any of Walmart’s employees. Some companies do, and so those people become not-Walmart employees instead.

          • poignardazur says:

            In that metaphor, McDonald is more like a passerby who has an occasion to throw a life preserver, but will only do it if the would-be-rescuee swears to pays him an immense amount of money. (I think Scott’s metaphor was about driving a dying man in a desert to a clinic)

            We all agree that this behavior is unsavory, but we disagree on which laws lead to the best results in these situations.

          • anonymousskimmer says:

            Walmart didn’t make the person poor and without the skills to get a better job

            Walmart eliminated the smaller stores. Then this happened:

            This analysis, published last month by the National Bureau of Economic Research, used data from Careerbuilder.com to measure how concentrated the market for different types of workers is in different commuting zones. For example, if there are only a few large employers of janitors in a small town, that market is more concentrated than a bigger city that has several janitorial companies to choose from.

            The results show that wages are higher in places where the concentration of employers is lower, even when correcting for the health of the local economy. So the large companies in those labor markets are exercising a kind of monopoly power, except instead of using that power to raise prices on their products or services for consumers, they’re lowering wages for employees who have few other places to work. In economist jargon, it’s called monopsony.

            http://money.cnn.com/2018/01/18/news/economy/big-companies-wages/index.html

          • sharper13 says:

            @anonymousskimmer

            Walmart didn’t “eliminate” the smaller stores. That implies a direct action on Walmart’s part to go an bulldoze them and chase the employees away or something.

            The people who lived nearby decided they were benefited more from shopping at Walmart than at by shopping at other stores, so they started patronizing Walmart instead. That’s pretty much the same way those other stores previously got their customers, by competing and being better at serving the customers.

            As for your monspsony argument, you neglected the fact that a concentration of employers for a particular employee set of skills is higher for higher skilled and paid employees and grows much lower as skill levels are reduced and pretty much anyone can do a particular job. The jobs Walmart hires for are the least affected by employer concentration. If you are an IP attorney looking for a job, there are going to only be a limited number of employers who want to hire that specific skill set at any given time. Conversely, most low-pay/low-skill Walmart employees could just as easily work at a fast food joint, a gas station convenience store, a warehouse worker, as a construction or farm laborer, etc… No one “specializes” as a shelf stocker and as a result is unable to get an equivalent near minimum-wage pay in another industry which doesn’t need shelf stocking skills.

      • Bugmaster says:

        I guess it depends on what you mean by “natural law”. If you mean something like, “a giant lookup table built into the substructure of the Universe, wherein actions are classified as moral or immoral”, then yes, it’s kinda silly. On the other hand, we are all biological machines, who have evolved in a specific way on a specific planet according to the rules of physics. In this case, both our utility functions, and the possible ways to maximize them, are to some extent pre-determined. Unlike “natural law”, this state of affairs is stochastic rather than deterministic, but still, I believe it does exist — and therefore, it can be studied.

      • Yosarian2 says:

        I would posit that for most people living in a democratic capitalism like the US, coercion and abuse of power by rich people or bosses or landlords or corporations is ALREADY a bigger source of unpleasant events and even personally catastrophic events in the day-to-day lives of most people then coercion by the government is. If we made our system significantly more libertarian, I think it would become even more so, and I think that is a major problem with the libertarian philosophy.

        Yeah the example given in the essay was pretty extreme and over the top bordering on absurdity, but that’s pretty common in philosophical thought experiments (I mean, pushing a fat guy in front of a trolley to try to stop it is also pretty over the top and absurd, isn’t it?)

        • Toby Bartels says:

          I mean, pushing a fat guy in front of a trolley to try to stop it is also pretty over the top and absurd, isn’t it?

          Yes, but that example is given to encourage people to test their moral intuitions, not to conclude that consequentialists are psychotic. There's actually been a very interesting discussion in these comments about how to respond to the threat of the Infinitely Rich Man, but if Nathan Robinson comes by and says that he wrote the article[^1] in order to spark that discussion, I'm going to be very sceptical.

          [^1]: We don't seem to quite know who all wrote it, but this is a hypothetical anyway.

          • moscanarius says:

            I mean, pushing a fat guy in front of a trolley to try to stop it is also pretty over the top and absurd, isn’t it?

            Yes, but that example is given to encourage people to test their moral intuitions, not to conclude that consequentialists are psychotic.

            Right on the money. The whole issue is that people are treating the thought experiment as a final moral veredict on the subject. They make thought experiments (which are fine), and then forget these are… thought experiments, not some pressing reality or some final statement of worthiness.

          • Yosarian2 says:

            I don’t think that anyone is trying to claim that libertarians are “psychotic”. The goal of thought experiments is just to try to establish that at least in theory there are situations where non-govermental behavior which follows the NAP can still be exploitive and tyranical even without a government law backing it up. Once people in a discussion agree to that basic concept (which Scott may find laughably obvious but a lot of libertarians do not) then we can move on to a more practical discussion of where to best draw lines and when to use what tool. But we do have to establish a baseline level of agreement on basic principles first.

            If people’s respose is “ok but that never happens in real life”, then great, that’s progres as well, we’ve moved from a relm of magical platonic truths to a relm of facts which can be discussed with evidence. (I do think that all of these effects do happen fairly often in real life, in fact, albiet usually in less extreme forms.)

            None of this is a “strawman” or an attempt to make libertarians look crazy. There are very good reasons why thought experiments like this help clarify these issues.

          • Toby Bartels says:

            I don’t think that anyone is trying to claim that libertarians are “psychotic”.

            Then you should read the conclusion to the article in Current Affairs. (Scott quoted almost the entire article but left out the conclusion.) This may be the source of our disagreement over the propriety of Scott's reply. (Or maybe not; I thought that Scott's post was pretty funny before I read that conclusion.)

          • Yosarian2 says:

            @Toby Bartels: Ah, I see. You are correct; I just went back and read the original, and yeah, he does have this little “answers” section at the bottom where he calls libertarians “psychotic”. That’s unfortunate, maybe he was trying to be funny but it really undermines his whole point.

    • Toby Bartels says:

      Since you said that Scott missed the point, I stopped laughing long enough to read the original article and, no, it's crap (and was probably never intended to be anything other than crap). (And fortunately, Scott quoted nearly all of it.)

      Questions 1 and 4 are ridiculous hypotheticals that only show that right-libertarian political philosophy is not a complete moral system. Maybe you can get interesting results if you present them to Objectivists, but they don't give any argument against right-libertarianism as a practical political philosophy in the real world. Question 3 strips away the contrived scenarios and just baldly asks whether right-libertarianism is a perfect philosophical system. The answer is No, so what?

      And as for Question 2 … Jesus Christ, this is the point that libertarians make all the damned time! Just as it is wrong for a private actor to rob you or kidnap you or murder you, so it is wrong for the state, regardless of what the king says or how the people vote or what the courts decide. Any article that asks that question and then acts as if it has made some kind of argument against libertarianism (right- or otherwise), deserves nothing in response but mockery and derision. That Scott's mockery took the form of an amusing story is just a bonus.

      tl;dr: The answer to Question 2 is, of course, No, and that is why I am a libertarian. (Although not a right-libertarian like the ones that the article is really attacking. You can tell that I'm a libertarian but not a right-libertarian by my use of the term ‘right-libertarian’.)

      • Deiseach says:

        Just as it is wrong for a private actor to rob you or kidnap you or murder you, so it is wrong for the state, regardless of what the king says or how the people vote or what the courts decide.

        Which means that I’d rather take my chances in Mandatory Cannibalism Village than Libertarian Utopia, because if you seriously mean the courts cannot order the local violent criminal nuisance locked up to keep him from stabbing, robbing, beating, mugging, commiting arson and the rest of his pleasant little hobbies, and I have to see him walking around on the streets free as a bird because oh noes, jail would be an intolerable violation of his personal rights!, then to hell with it.

        There are people who make other people’s lives hell because they are selfish, greedy, inconsiderate sons of bitches who are very much able to yell and stamp their feet about “my rights!” and who terrorise others because they don’t care about anyone but themselves. You get it in the kind of antisocial behaviour and tenants that terrorise housing estates but manage to keep out of the hands of the law (mostly because nobody is willing to go to the cops about them, they’re too afraid). Fines are no good – they don’t pay them. Summons to appear in court – they don’t turn up. ‘You broke the contract, we’ll permit the landlord to evict you’ – good luck with that if you can even get a court willing to grant such a decision, they’ll run off with a sob story to the social worker, local radio station, or political representative about how they’re a poor deprived persecuted person who is being heartlessly thrown out onto the side of the street with their young family and nowhere to live. Lecturing them about the rights of others is no damn good because they don’t believe in or care about that. The only effective deterrent is locking them up – and if Libertarian Utopia takes that away, then Libertarian Utopia should have to live with the consequences, and I hope they damn well like living beside someone who will stab you as soon as look at you. How do you deal, in a libertarian society, with continual and hardened lawbreakers if you can’t put them away for the protection of society? Move them on from your village to live somewhere else, where they then start up the same tricks? Or you can’t put someone in jail but it’s perfectly legal to hire private assassins to kill them?

        And before you give me the usual “this kind of behaviour will give you a bad reputation so no-one will be willing to enter into contracts or do business with you” answer – please, these are the kinds of people who if you won’t sell them something they’ll steal it, harass and threaten you until you hand it over, squat on private property and in force with their family and others to intimidate you if you try to move them on, and have no qualms about using violence themselves. Private security forces 24/7 to make sure your house doesn’t suddenly go on fire with you inside it – well, I hope they’re cheap in Libertarian Utopia because you’re going to need them when dealing with this type of person.

        • Egon Maistre says:

          Which means that I’d rather take my chances in Mandatory Cannibalism Village than Libertarian Utopia, because if you seriously mean the courts cannot order the local violent criminal nuisance locked up to keep him from stabbing, robbing, beating, mugging, commiting arson and the rest of his pleasant little hobbies, and I have to see him walking around on the streets free as a bird because oh noes, jail would be an intolerable violation of his personal rights!, then to hell with it.

          Good thing that this is not at all what it means in Libertarian Utopia. A minority, a very small minority, of libertarians are absolute pacifists (Robert LeFevre is the only big name example of which I am aware). The vast majority believe in the legitimacy of defensive violence and some sort of retributive violence. Minarchist libertarians believe it is perfectly legitimate for the state to lock up, forcibly extract monetary damages from, and punish criminals, in proportion to the extent they violated rights. So, no executing people for theft, but robbers can definitely expect to face some combination of fines, jail time, and caning (libertarians generally disproportionately favor reviving corporal punishment compared to the overall population). The libertarian, however, absolutely opposes locking up people who have not yet violated anyone’s rights. But it shares that in common with most of the non-totalitarian political philosophies.

          And that is if the state even gets involved in the first place. The libertarian love for guns and their use in self-defense and defense of others is well documented… and often denounced. Recall that this is the only political philosophy straw-manned as saying it is OK to shoot someone for picking a flower from your garden without permission (it isn’t). It is a topic of discussion among really hardcore libertarians as to whether private ownership of nuclear weapons is permissible (it is — just kidding… maybe).

          Source: I was an anarcho-capitalist libertarian for ~15 years, have a disproportionate number of libertarian friends, and still very much sympathize with the philosophy.

          • Brian says:

            You were an anarcho-capitalist libertarian. What changed and why? I’m new to the ideas of that school of thought and am interested to know both what persuaded you to subscribe to it for so long and what ultimately seems to have dissuaded you from it.

          • Egon Maistre says:

            General disclaimer: my response touches on topics that Scott has declared banned, so let it be understood that I am not attempting to open any of these topics up for discussion, I am merely attempting to honestly answer Brian’s query. Let’s respect the rules by not probing further towards banned topics.

            @Brian

            Let me see if I can boil it down to something fairly short that still manages to be clear.

            I don’t want to overstate the extent to which I am not a libertarian or AnCap. If I could push a button that would implement David Friedman’s vision, I would slam my hand on it with zero hesitation. Partly out of sheer curiosity, but also because there is some sense in which I still think that a fully implemented AnCap society would be best for future human development. But I would still say at the end of the day that I’m not a libertarian, but rather a radical mutation thereof.

            AnCap libertarianism attracted me initially because it was, to me, the obvious logical conclusion of a deep commitment to freedom, as inherently good in some sense, but also as key to good outcomes. Good consequences are good, people are not interchangeable units, so as great a latitude of freedom as possible is necessary for everyone to discover their own good consequences and pursue them as shallowly or deeply as they choose. I read the usual suspects of Rand, Heinlein, and Friedman (Milton and David). This is brief, but I am inclined to believe libertarians are born, not made. You either intuit that liberty is a crucial moral foundation or you don’t. See Jonathan Haidt on this point.

            Four factors pushed me out of libertarianism, to whatever extent I am out.
            1. There has always been this kind of “one great last stand” strain in libertarianism that is simultaneously defeatist and idealistic. This became increasingly vexing.
            2. The Ron Paul presidential campaigns brought in a lot of people who became prominent and took over libertarian institutions, but who I despised as the wrong kinds of people. This was, I believe, the source of the split between many libertarians getting involved in social justice politics and other libertarians going to the alt-right. Scratch a social justice or alt-right ‘libertarian’, and they probably came in via Ron Paul.
            3. I encountered, thanks to this very blog as it happened, the ideas of the Intellectual Movement That Shall Not Be Named, which is what I would broadly identify with at the present time. I do not feel I can be any more clear here without violating the rules, sorry.
            4. Related to three, I started to appreciate the wisdom of traditions that I’d previously believed to be irrational restraints that were impeding people’s ability to find and pursue their own good consequences. Long standing traditions relating to sexual conduct, distinct ethnic polities, and religious practice now seem to me to have been abandoned hastily and I would like to see their return in some fashion. Chesterton’s Fences: much more important than I thought.

            I think that sums it all up, hopefully not too briefly to be useful. I am willing to expand on any points relating to coming in or going out, except 3 and 4 for the reason previously stated.

          • Nornagest says:

            Talking about the Death Eaters or their ideas isn’t banned, you just can’t use the name because Scott doesn’t want his blog coming up with those search terms.

          • 2. The Ron Paul presidential campaigns brought in a lot of people who became prominent and took over libertarian institutions, but who I despised as the wrong kinds of people.

            “I’m still a liberal. It’s those people who aren’t liberals.” GKC on the Liberal Party early 20th century.

        • Toby Bartels says:

          Well, the standard right-libertarian minarchism that the Current Affairs article is addressing does allow for imprisonment by the state, as long as it's done by due process in proportionate response to an initial aggression by the potential prisoner (or something along those lines).

          Not me, though. I'm a bleeding-heart left-anarchist disciple of Tolstoy, so I support the complete abolition of prison. (And of all criminal punishment, but prison would be a good start. I mean, I don't support corporal punishment either, but it's more humane than the prison system in much of the First World.)

          Now, if you stipulate that this person is going to continue to do horrible things, no matter what, unless he is imprisoned (or killed), then I guess that he has to be imprisoned (with the ability to commit suicide in case he would prefer that). And given the existence of this person, any society that doesn't recognize his existence will suffer for it. (I'm not sure that this has anything in particular to do with libertarianism, however, since from the way you describe it, it sounds as if he's walking around free even in the contemporary Irish Republic.)

          So if you live in such a society, then I guess that you have no other choice but to get together with a small group of trusted friends, kidnap him, and lock him in your basement. Treat him as well as you can while ensuring that he can't escape, and prepare your defence for when you are caught. (If he really needed to be locked up, then you should win with a necessity defence, if the court is just.)

          On the other hand, if you think that it would be immoral for you to do this, well, my point is that it would be just as immoral for someone else (a state official or anyone else) to do this on your behalf. And if the ends don't justify these means (or if you're not a consequentialist and these means are forbidden regardless of the ends), then apparently it's morally preferable that he go free after all!

          (If the reason why it's OK for the state to do what private vigilantes may not do, is that the state follows due process, then hold yourself to a high standard when you commit your acts of vigilantism. Warn the person ahead of time (anonymously) what someone might do to him and under what circumstances, give him an opportunity to present a defence, and stick scrupulously to your original plan. Maybe other vigilantes are a scourge on society, but you can be as proper about it as you like!

          Or, if the reason why it's OK for the state is that the state's actions have been approved by the people, then get the people's approval before you act. In a democratic republic with no prisons, you'd campaign to get people to support them for certain criminals; in a libertarian utopia, you can support the same public opinion campaign. Once 51% of the population agrees with you (or whatever you think that the standard is), then you can act with impunity. (I mean as far as your conscience is concerned; but if you get public opinion overwhelmingly on your side, then you won't even have to hide what you’re doing. That's what the state does!)

          But if this all horrifies you, and you don't think that anything could justify kidnapping and forced imprisonment, then it's not going to be any more justified because the state gets involved. The state is not a god, but a group of people, and may only do what a group of people may do. (This position is called ‘rational anarchism’ by the author self-insert character in Robert Heinlein's The Moon Is a Harsh Mistress. I call it obvious: you don't have to be anarchist (which Heinlein wasn't) to believe it; you just have to accept responsibility for what you're asking people to do for you.))

          • Yosarian2 says:

            There are practical, utilitarian reasons why it’s generally better to leave justice in the hands of the state instead of in private hands. Private justice, no matter how well intended, frequently leads to feuds, maybe even wars; if you and your friends think Bob the Smiling Killer needs to be locked up, but Bob’s family and friends think that he shouldn’t be, then if you try to lock him up, they may attack you, ect. In a wide variety of societies, this problem very frequently led to patterns of tribal violence or family feuds or related kinds of long-lasting violence.

            On the other hand, if you can get everyone to agree that a certain way of deciding the issues has legitimacy, that refusing it is not an option, and further that anyone who refuses the orders of that court is themselves a criminal and the enemy of all, then you can avoid a lot of violence. It doesn’t necessarally have to be “the state”, but it has to be something that basically everyone accepts as legitimate and which has a way to enforce it’s decisions that again everyone accepts as legitimate.

          • Toby Bartels says:

            @Yosarian2 :

            Yes, but I'm hoping that Deiseach has more than merely pragmatic utilitarian reasons to reject taking the law into her own hands in this way. (I may be disappointed; this is the person that I once compared to Molly Weasley, after all.)

    • Cliff says:

      I don’t think too many libertarians care about any of those four specific points. They really don’t have any bearing on whether libertarianism is a good political doctrine or not. If the overarching point is that libertarianism is not a magic cure for all problems, including 100% of your society being insane and wanting you to die, then I will concede your point and we can move on. Of course the same point can be made about everything else.

      • themountaingoat says:

        The point is that people in other systems can’t kill you without breaking the rules. Under libertarianism they can. Most people think that the set of rules we suggest should stop those types of things from happening if everyone follows them.

        • Nornagest says:

          People in just about every system I can think of can kill you without breaking the rules, if they have enough [charisma|pull|Party connections|contributions to the county sheriff’s reelection campaign]. Scott’s whole point is that it’s grotesquely uncharitable to apply this sort of thinking to libertarianism and not to anything else.

          • themountaingoat says:

            Most moral systems and legal systems have rules in place that if followed would mean that even those in power couldn’t do whatever they wanted.

            Of course if those rules are not followed bad things can happen, but in that case basically all political philosophies are screwed and none of the rules matter in the slightest. Applying that type of thinking to Libertarianism would be saying that whoever enforces property rights will just enforce them in a biased way.

            The criticism of libertarianism is different, it is finding situation where even if the rules are followed by everyone to the letter bad things happen. To me it seems that such a system needs additional rules in place.

          • Nornagest says:

            That’s basically a restatement of your previous post, so I’m tempted to just say “see previous”, but we’re never going to get anywhere that way.

            Legal and ethical systems are made of human concepts, and generally speaking the simpler the systems and the bigger the concepts, the more work it takes to bring them down to the object level. NAP libertarianism is very simple. But that doesn’t mean it’s easy to enforce, it means that it’s foisting off most of its complexity onto the definition of “coercion”. Right now it exists almost entirely in armchairs, so that remains relatively undeveloped, but if there was ever a functioning system designed on its basis it’d need to have reams and reams of precedent covering what is and is not coercive. I guarantee you that precedent would cover cute tricks like those you’ve mentioned to John below.

            This is also its downfall, and why I’m not an NAP libertarian: its simplicity makes it so flexible in practice that it’ll inevitably get permuted and mutated and well-maybed into something totally unlike its intention. There are probably ways you could abuse this to lethal effect with enough money, influence, and cleverness, but like I said that’s true for literally every system. If you think NAP systems would be unusually prone to this, though, that there’s some simple sequence of magical actions that’ll let you blatantly and willfully kill a guy under them while leaving all the all good little libertarian robots helpless because it’s technically not coercion, you’re basically making the same mistake as those sovereign-citizen types that get worked up over fringes on flags.

          • themountaingoat says:

            @Nornagest

            I wasn’t aware that most libertarians thought of the NAP as something that needs considerable refinement and precedent in order to be fully developed as a legal system. I was under the impression that it was supposed to be an obvious principle that stood on its own and did all the work needed.

            If you added precedent and exceptions to NAP libertarianism you might well end up with something like our current system after you solve all the problems. We need to full system in order to argue about it.

          • Nornagest says:

            I was under the impression that [most libertarians thought of the NAP as] an obvious principle that stood on its own and did all the work needed.

            Most people think that about human rights, too. Most people haven’t thought about this stuff in enough detail. You can always find something to quibble with most people about, but it’s not going to be a very interesting argument.

        • John Schilling says:

          The point is that people in other systems can’t kill you without breaking the rules. Under libertarianism they can.

          How do you figure? The Non-Aggression Principle may be fussy to the point of uselessness around the edges, but killing people other than in self-defense is pretty much core Aggression and so against the fundamental rules of even the strict NAP-deontologist libertarians. And pretty much all the libertarians who aren’t anarcho-capitalists agree that there should be governments passing and enforcing laws like “no killing people except in self-defense”.

          • themountaingoat says:

            Self defence includes defence of property so you can buy the land around someone and kill them if they step onto your land, or merely starve them to death if they refuse to leave.

          • Cliff says:

            Self defence includes defence of property so you can buy the land around someone and kill them if they step onto your land, or merely starve them to death if they refuse to leave.

            You just conceded your whole argument, right? You first said that with a Libertarian political system you could just kill anyone you wanted and be within the rules. Now you say if that person trespassed on your property you could kill them. Why would they trespass on your property? And needless to say, murder to defend against trivial trespassing is not a core libertarian belief.

          • baconbits9 says:

            Self defence includes defence of property so you can buy the land around someone and kill them if they step onto your land, or merely starve them to death if they refuse to leave.

            Not really, most (pretty much all) people who support non-aggression principles don’t allow for wildly disproportionate responses. Your interpretation would imply that I could murder, rape or imprison any person who set a toe on my property without permission, and is of course wildly inaccurate.

            The non aggression principle says that a person aggressed against can defend themselves and their property and can seek repayment for damage caused by the aggressor. There is some debate about what type of transgression would allow deadly force, but that debate doesn’t imply that the answer is ‘any’.

          • Yosarian2 says:

            What I would say is that in a libertarian system, whoever owns the things you need to live and thrive can ruin or even kill you by just refusing to do buisness with you (which was part of the point of the first example, I think.) And there have been examples of that historically.

            In general I think libertarian systems work best in a situation like parts of the US earlier in our history, where anyone who was too poor to survive could just go west and get free farmland (or hunt buffalo, or trap beaver, or whatever) The free exploitatin of abundant natural resources that had no owner made libertarian government possible. In the modern world though I think it no longer is.)

          • John Schilling says:

            What I would say is that in a libertarian system, whoever owns the things you need to live and thrive can ruin or even kill you by just refusing to do buisness with you (which was part of the point of the first example, I think.) And there have been examples of that historically.

            What are some of those historical examples?

            Because the point of Scott’s response to the first example is that “whoever owns the things you need to live and thrive” isn’t a person, but a whole lot of people who would need to 100% cooperate to kill you in a libertarian system, but in absolutely any other system could kill you if substantially less than 100% of them agree on that point.

          • 1soru1 says:

            Medieval Iceland had slavery without state enforcement because no-one could survive by themselves, and more potential slaves than householders willing to keep a non-relative alive on a non-slavery basis.

            The thing about requiring many people to act in concert to deny you of (what would currently be considered) rights is that a market is a very efficient way of coordinating an arbitrary number of people. If you accept the argument that it is the most efficient way of doing that coordination, it follows that it has the greatest potential to be efficiently oppressive.

          • Yosarian2 says:

            @JohnSchilling
            >What are some of those historical examples?

            Just to name a few off the top of my head; a lot of the systematic discrimination against African Americans in the US was done by private individuals refusing to do business with them. For example, the contracts where people agreed when they bought a house that they would never sell to an African Americans. That often lingered on for some time with no government support. You would have lots of individual economic actors,including homeowners real estate agents and bankers, all agreeing to discriminate against African Americans in their private buisness dealings often due to social pressure, and that helped create areas of black multi-generational poverty. Which only stopped when the government came in and banned the practice.

            Another example are blacklists. In the early days of the labor movement, if a worker spoke out and was pro-union, he might be blacklisted by all of the factory owners in a city, making it impossible to get a job. In other words, private business owners censored someone’s ability to speak his political opinion by threatening to cut off his access to work, which he needs in order to get the resources to survive.

          • John Schilling says:

            Just to name a few off the top of my head; a lot of the systematic discrimination against African Americans in the US was done by private individuals refusing to do business with them. For example, the contracts where people agreed when they bought a house that they would never sell to an African Americans. That often lingered on for some time with no government support. You would have lots of individual economic actors,including homeowners real estate agents and bankers, all agreeing to discriminate against African Americans in their private buisness dealings often due to social pressure,

            Haven’t we been through this at least a dozen times here already? No, homeowners/real estate agents/bankers didn’t all discriminate against African Americans “due to social pressure”. Some businesses did this; other businesses saw a potentially lucrative market in actually selling stuff (including nice houses) to African Americans.

            The times and places where all businesses discriminated against African-Americans, that was generally because the government said “you must all of you discriminate against African-Americans, even if you’d rather profit by dealing with them, because 51% of the voters around here are bigots”.

            and that helped create areas of black multi-generational poverty. Which only stopped when the government came in and banned the practice.

            Right; when only 49% of the voters were bigots, the government came in and banned the practices that it had previously mandated, and said “yay us!”. And you cheered.

            But that only helps African-Americans in a 51% tolerant society. Liberty helps African-American even when society is only 5% tolerant. Or even just 5% more greedy than intolerant.

          • Yosarian2 says:

            It doesn’t take 100% to create great hardship on a community, and create a kind of repression that’s deeper than most governments can do. You are right though that in some places it was legally mandated, but it was a problem even in places where it wasn’t.

            How about the second example? Blacklists of known “troublemakers” seems like a clear and unambiguous example of wealthy people working together in order to use economic power to restrict individual liberty.

          • John Schilling says:

            It doesn’t take 100% to create great hardship on a community,

            OK, but that’s moving the goalposts quite a bit. Like, to the other team’s thirty-yard line, compared to the original posulate.

            and create a kind of repression that’s deeper than most governments can do.

            Really? You really want to bring up the sort of repression that governments can do? I promise I’ll leave the Nazis and the Communists out of it…

        • andagain says:

          That first statement could hardly be less true. In the US, for example, Congress, the Justices of the Supreme Court, and the President can certainly write rules to have people killed. Indeed, every government can do that. It might be difficult, and even require some rewriting or creative interpretation of constitutions, but if everyone in society is determined to have you dead, they can manage it.

          They could, for example, make it a capital crime to offer you, or allow you you take, food, drink, shelter or clothing. That is a much less hypothetical example than the anti-libertarian thought experiments given above. It happened, for example, to the Roman statesman Cicero, after his enemies came to power.

          Until the law was revoked, it was literally impossible for him to survive anywhere Roman law applied.

          Or they could just declare breathing to be a capital crime, and selectively enforce it against you.

          • Yosarian2 says:

            So, to do that in the US, you would need a majority of both houses of congress, and the President, and you’d need the Supreme Court to decide that it wasn’t unconstitutional, and then you’d need the police to make the arrest, a prosecutor to press charges and decide to push for the death penalty, the judge to go along with it, and the jury to all agree unanimously. And at any point in the process pressure from either the media or the public could torpedo the whole thing.

            Frankly it doesn’t seem like that much of a risk to me, in that form, in a modern democracy.

        • Tracy W says:

          The point is that people in other systems can’t kill you without breaking the rules.

          Most Western democratic countries had military conscription within living memory. Some still do. And I don’t know of any rules stopping conscription from being voted back in, in those countries that banned it.

          And once you were conscripted, and in a war, the military could keep ordering you into battle until the odds caught up with you. Or shoot you if you disobeyed.

    • AnonYEmous says:

      I. There is such a thing as effective coercion without any actual application of force.

      I think Scott’s response is meant to be that this is known, but that there’s a lot of ways around this kind of coercion, and also, well:

      I do not believe democracy is always right. But I believe it is a wise way to govern.

      So yes, effective coercion may exist without application of force, but libertarian philosophy is still a wise way to handle things despite not taking care of some edge cases because it does a good job handling most cases, since most cases either involve no coercion or force-based coercion. Plus, as Scott also notes, if a lot of people are set on doing something it’s hard to stop them.

      II. The source of coercion is not relevant inof itself. What is relevant is the effectiveness of that coercion.

      It sure seems like he acknowledged this, and even argued that most rational people would agree anyways.

      III. Property rights are not, as many libertarians claim, a natural law.

      His response is that nothing is natural or rather nothing is perfect philosophically, and therefore it’s not a problem if his philosophy isn’t either.

      IV. Allowing too much power to accrue in the hands of one person is inherently dangerous even if the legal system is maintained, because that person has the potential to act in ways that are not normally possible.

      You’re right that Scott doesn’t really respond well to this, so allow me to do so.

      It’s possible under any system for a person to accumulate too much power and abuse it. Robinson’s answer to this would be socialism, but Stalin tortured to death people he didn’t like (and his security chief Beria raped women that he did like). Sure, those two are supposed to be a failure mode of socialism, but I bet I could argue that fabulously rich people are a failure mode of capitalism! No, seriously, according to economists firms’ profits are meant to fall to zero in the long run (my dad certainly says this often enough). That probably doesn’t account for a wage paid to the owner, but it sure does account for large-scale profits that currently accumulate to the owner; those shouldn’t exist under capitalism, because competition will erode whatever advantage existed and price-cutting will take profit to zero. But protesting vociferously that capitalism’s fail states shouldn’t count is silly and I won’t do it; my hope is that honest socialists can follow the same standard.

      • Sniffnoy says:

        Someone else please correct me if I have this wrong, but I’m pretty sure it’s “economic profit”, not “accounting profit” (i.e. profit in the ordinary sense) that’s supposed to go to 0. The former being profit in the ordinary sense, minus the profit you could make by doing someting else (i.e. taking the opportunity cost and literally subtracting it out). So it just means that you couldn’t make more money by switching to doing something else, not that you’d be making no money.

        • AnonYEmous says:

          Well, I believe you still get paid a wage, but I don’t think it is only accounting profit. Either way though, it still shouldn’t allow you to get fabulously wealthy, so that’s a failure mode of capitalism.

          • Cliff says:

            “Either way though, it still shouldn’t allow you to get fabulously wealthy”

            No, that’s wrong. And Sniffnoy is right. Look it up.

        • Corey says:

          I believe in a perfectly competitive market actual profits are driven to 0. Take the textbook example of perfect competition, retail gasoline sales. The margins are indeed razor thin, gas pumps are basically loss leaders for convenience stores.

          Profits can only happen to the extent the relevant market deviates from perfect competition.

          • Cliff says:

            This is not correct, please look it up. It only takes a minute.

          • Economic profit goes to zero in a perfectly competitive market. The return on capital, which sometimes gets labeled “profit,” doesn’t.

            More important, there may be rare talents of great economic value. If you can write books so good that hundreds of millions of people want to buy and read them, you can have an income of a billion dollars a year. If you can predict which companies are going to succeed and which to fail, ditto, although that one takes a while since you have to build up either your own investment capital or the reputation that lets other people pay you to invest theirs.

      • TheWorst says:

        I think Scott’s response is meant to be that this is known, but that there’s a lot of ways around this kind of coercion…

        Name one.

        “Free Market Jesus would descend from Objectivist Heaven and punish him” is not an acceptable answer. Neither is “But rich people are magical fairies of pure heavenly intent!” If there are a lot of ways around this kind of coercion, there should be at least one that isn’t pure fantasy. The central flaw in libertarianism is that none of you seem to do this.

        If you know something that makes libertarianism something other than a combination of non-viable and utterly monstrous, then why not say what it is?

        • AnonYEmous says:

          Name one.

          “Couldn’t the victim just move to a different village?” interjected John Jay.

          “Couldn’t the victim just build his own house, and farm his own food?” asked John Adams.

          The first example doesn’t seem to be addressing coercion by rich people; those are the province of the latter examples. But it looks like that’s your major objection, since you’re writing stuff like this?

          “Free Market Jesus would descend from Objectivist Heaven and punish him” is not an acceptable answer. Neither is “But rich people are magical fairies of pure heavenly intent!”

          Here’s the answer: this shit can happen anywhere, under any ideology, and does happen everywhere under every ideology. The fact that you have to construct ridiculous edge cases, whereas I can point to direct examples of abuse under other systems (chiefly socialism; it depends what you consider an alternative to capitalism), points to the fact that libertarian-style capitalism is probably the better alternative.

          To put it simply: in order to prevent other people from abusing their limited power, you need to give someone near-absolute power. And that person may well abuse it; if he doesn’t, his successor probably will. I’d prefer to let people abuse their limited power because, in a Bayesian sense, I don’t think it produces so many negative outcomes. Maybe you think otherwise, but again – this is a ridiculous edge case for a reason.

          • Conrad Honcho says:

            This seems like a good time to bust out that old C.S. Lewis quote:

            “Of all tyrannies, a tyranny sincerely exercised for the good of its victims may be the most oppressive. It would be better to live under robber barons than under omnipotent moral busybodies. The robber baron’s cruelty may sometimes sleep, his cupidity may at some point be satiated; but those who torment us for our own good will torment us without end for they do so with the approval of their own conscience.”

            At least the infinitely rich man acknowledges he’s being a dick, and is doing so willfully and purposefully, using what limited power he has. The communist liquidating the capitalist class for the benefit of the proletariat not so much.

          • anonymousskimmer says:

            While the infinitely rich man may acknowledge his jerkery (though often enough our rich don’t – witness the dogooderness moralizing** of the likes of Peter Thiel, Warren Buffett, and Mark Zuckerberg), those who say “system X will make us all better off” don’t.

            Whether system X is:
            – an economic system
            – an educational system
            – anything in which a shared moral framework is more fundamental than personal empowerment

            Those who argue for libertarianism and try to convince you it’s better for you are moral busybodies (they simply aren’t omnipotent, even if they may alter society enough to have a positive or negative impact on your personal interests).

            Is it better to live under typically potent robber barons or typically potent moral busybodies? The former is just a subset of the latter.

            ** – But, of course, they have a bigger fish to oppose: government policy, society, other businesses with entrenched interests, etc….

          • Toby Bartels says:

            anything in which a shared moral framework is more fundamental than personal empowerment

            A big motivation for libertarianism is to avoid these situations, to make things as much as possible about personal autonomy/empowerment and rely as little as possible on a shared framework. (My email alert says that you originally wrote autonomy, which fits better with a focus on negative freedoms than empowerment does, so I can see why you changed it.)

            I will agree that right-libertarianism, and even more so the NAP alone, don't get you all of the way (not even for autonomy, much less for empowerment.) But I do think that it can help.

          • anonymousskimmer says:

            Autonomy without relative power is another word for nothing left to lose.

          • Toby Bartels says:

            Yeah, but power don't mean nothin' if it ain't autonomous. (This might be stretching the allusion a little bit.)

          • andagain says:

            in order to prevent other people from abusing their limited power, you need to give someone near-absolute power.

            I don’t see why: A parliament may have near-absolute power (the old line was that Parliament could do anything but turn a woman into a man, but they will not accept that limitation anymore), but it does not follow that any single MP has such power.

      • Tracy W says:

        firms’ profits are meant to fall to zero in the long run

        A) that’s super-normal profits that get competed to zero.

        B) that’s in the long run. In the short term one can accumulate a really large amount. The largest fortunes accumulated in capitalist countries tend to be accumulated by the people who first worked out how to use a newish technology profitably- e.g. railways or assembly lines or personal computers.

      • Yosarian2 says:

        It’s possible under any system for a person to accumulate too much power and abuse it.

        Sure, but I think the system that has generally done the best at avoiding that has been constitutional democratic republics, generally with a govenrment significantly stronger then that espoused by libertarians.

        In the US, if someone, no matter how rich, tries to directly abuse their power in a way that infringes on government prerogatives like “raising their own army” or “enforcing their own private set of laws” ect, the govenrment can stop that. It’s a testament to the strength of our institutions that no one really tries to do that to any great extent. In a libertarian society where the govnerment was significantly weaker compared to the power of the wealthy, I am less confident that the government would be able to prevent a slow slide into some modern form of feudalism or oligarchy.

        Basically, in a society like ours, the govnerment and the wealthy provide an unofficial kind of check and balance on each other’s powers. If the government was weakened significantly, I think that would increase the risk of the kind of situation we’re talking about.

    • IV. There’s certainly a point there and from an anarcho-syndicalist, or most sorts of anarchists really, I think a libertarian would have to take this as a serious criticism. But I think the libertarian could rightfully accuse basically any other political philosophy of utter hypocrisy on that point.

      • ragnarrahl says:

        What precisely is the anarcho syndicalist solution to “everyone in all the labor unions (that’s the “syndicalist part” right?) hates you and wants you to die and shuns you in an environment filled with ravenous direwolves?

        • I’m saying that they can make point IV non-hypocritically, not point I.

          • ragnarrahl says:

            I think the point IV version of what I said is literally just delete the direwolves and add in that the labor unions shun non-shunners. The language of “buy” in IV is verboten for the an-synd, but the reality of bargaining dickishly remains.

    • carvenvisage says:

      I’m sorry, but much as I usually love your writing, here you’ve totally missed the point of the article you’re attempting to parody. You’ve picked apart the fine details of each hypothetical case, while ignoring the broader point that each hypothetical is meant to illustrate.

      Did you read to the bottom of the article? I was thinking something similar until I got to the part where they gave the answers (bad sign), and all 4 were “ARRRRROOOOOOOOOO, FUCK LIBERTARIANS”.

      • Edward Scizorhands says:

        And as I read through them, I thought “this is exactly what terrifies me about modern unlibertarian society.” If you are worried about blacklisting or being hounded from your job, is your fear really that the libertarians will take over and blacklist you? Or that some random Twitter mob driven by a major prominent philosophy will form to do these things right now?

        • sharper13 says:

          Yet, it seems to terrify some people that others (libertarians) may want to take over and….

          …. leave them alone to do what they want!

          • andagain says:

            The libertarians would also allow everyone else to do what they want, even the things that the libertarian-dislikers consider to be evil.

            And if you consider some act to be evil, you generally want to stop people from doing it.

    • Hyzenthlay says:

      I’m sorry, but much as I usually love your writing, here you’ve totally missed the point of the article you’re attempting to parody. You’ve picked apart the fine details of each hypothetical case, while ignoring the broader point that each hypothetical is meant to illustrate.

      The broader point is invalid if the hypothetical scenarios designed to prove them are ridiculous.

      I don’t think he missed the point at all; I think he cleverly demonstrated why the “presenting a bizarre, highly specific hypothetical scenario taking place in an alternate universe” is a flawed style of political argumentation. The cannibalism scenario is so pure an example of this that it almost functions as a parody on its own.

      Asking people to ignore the stupidity of the scenarios themselves and just focus on the point that those scenarios are trying (and failing) to make is, itself, missing the point.

    • John Schilling says:

      and if you allow a large amount of power to accrue in a small number of hands the severity of such incidents will increase.

      Isn’t large amounts of power accruing in a small number hands pretty much the essence of government? One president, fifteen cabinet secretaries, five hundred thirty eight legislators, and 4.094 trillion dollars in annual expenditures. If you’re measuring power in terms of wealth, the average member of the US house of representatives commands power roughly equal to the richest capitalists in the world. And congresmen unlike capitalists have institutions expressly designed to coordinate their power in united efforts that no “infinitely rich man” can in reality hope to match.

      • Edward Scizorhands says:

        If your goal is influence, being in charge of a budget of $100 million beats having a personal fortune of $100 million.

        • quanta413 says:

          Nitpick, it can’t be worse to control the budget without more restrictions on your control of the budget. But it could be equal if the budget is once only. If it’s a flow though like yearly? It’s probably not even close at that point.

          • Edward Scizorhands says:

            The influence I get from spending someone else’s $100 million, even once, is superior to that from spending my own $100 million.

            People know that I am more free with someone else’s money. I may or may not spend my own money, and people in charge of hiring prostitutes to get their company the contract know this.

  8. Yaleocon says:

    Without even considering the fact that it’s ridiculous, and a straw man, and whatever–the Infinitely Rich Man makes a LARGE NUMBER of people’s lives much, much, much better in his quest. Like, his whole neighborhood, all his old friends, his wife, his old boss and coworkers, they all get MASSIVE payoffs.

    And the story specifically stipulates that some of these were necessities. What, was your wife dying preferable to never seeing her again? All of your friends who hit hard times–isn’t them getting paid off by the IRM preferable to never getting out of those hard times?

    The only way to coherently critique the scenario is to make a huge deal of how a single person is treated and how he should not be so treated, even though people literally survive due to the IRM’s grudge. Honestly, that seems like ignoring such a massive utility gain that any theory which fails to endorse it is even less plausible than straw-man libertarianism.

    Try again, Robinson.

    • Scott Alexander says:

      Also, I’d really like to know how even a non-libertarian society bans that. Pass a law that it’s illegal to pursue vengeance even if you make sure to use methods that aren’t illegal?

      • Jack V says:

        I’m getting a headache tracking who is weak-maning whom, but the way *I* ban that is with a progressive taxation funding a minimum income so no-one is that rich.

        • Thegnskald says:

          Cool. I create a social media following and use my social power to push for the individual in question to get fired.

          Or I constantly call the city over every tiny violation of ordinances.

          Or I contact the IRS and anonymously tip them off this individual is engaging in tax fraud.

          Or any number of things.

          Wealth isn’t the only form of power, and this particular problem probably isn’t solvable.

          • eelcohoogendoorn says:

            This. Power comes in many forms. Every time I have been fucked over in life, it has been at the hands of bureaucrats with unchecked power in some form or another. By contrast, the infinitely rich man is entirely hypothetical to me.

            Yes, people with more power than you will be able to use it against you; I cant think of a single system of rules where this isnt true.

            Someone above brought up the holocaust as a supporting example of how people with too much power and abusing it are not a hypothetical problem. I couldnt reply there, but I cant resist pointing out here how dumb that is. Hitler is an example of unstable democracy, not a problem of unstable distribution of wealth.

          • moscanarius says:

            Yep. And since the power of money has been removed, the share of social media / popularity / irrational hatred on the total power pool has increased. Where money could counterbalance them, now there are fewer counterbalances, and they rule stronger.

        • quanta413 says:

          Because never in the history of man have the poor managed to harm the rich.

          I don’t even see why it’s supposed to be that comforting that something is illegal when I’m being pursued by a hypothetical genius with a lot of resources and no other goal in life but to screw me. People do illegal things all the time. Nothing in the world but the most obscenely expensive investment of resources can help you that much against that determined an attacker. Even U.S. Presidents get shot every few decades and not by assassins paid for by the richest, but often by lone nuts.

        • Minimum income is only good in so far as you can spend the money on something, and progressive taxation, which already exists, hasn’t stopped fantastically wealthy people from existing, unless you want the top brackets to go to 99% or something, and then you have to find a way to stop people gaining income from non-income sources such as capital gains and speculation by raising taxes there too, so at that point you might as well just have a flat out wealthiness cap and send in the people’s militia.

          One reason billionaires aren’t commonly fucking with poor people on an individual vindictive level in our existing society is that the most implausible part of the story is the part where they’re hanging out in the same bar to get in an argument to begin with. Bill Gates is never going to pay your wife to leave you and use his wealth to forge documents to get you accused of a crime, because to Bill Gates you simply don’t exist.

        • sharper13 says:

          Just for the sake of argument….

          If you have

          a progressive taxation funding a minimum income so no-one is that rich

          , does that imply some sort of mechanism for collecting and enforcing that progressive taxation? Does that mechanism provide some individuals the power to do the very thing you’re solving against? Are you just trading people with wealth and power from serving others in voluntary transactions for people with wealth and power from enforcing a system of progressive taxation, i.e. the public choice conundrum? How do you solve that in turn? Who will watch the watchers?

          • MugaSofer says:

            I don’t think the existence of a simple taxation system requires handing anybody Eleventy Billion Dollars, or anything equivalently dangerous.

          • sharper13 says:

            @MugaSofer,

            Currently, with a less progressive tax system than argued for, about 367 people collectively control a little over 4 Trillion dollars a year in spending on whatever they decide as well as the power to kill pretty much anyone they desire for any reason they can hide or justify.

            So I’m not all that comforted, given the current examples in the world.

          • about 367 people collectively control a little over 4 Trillion dollars a year in spending

            51 senators + 218 representatives + the President comes to 270 people and collectively control about 4 trillion dollars a year in spending. Who else are you counting as necessary? Making it 2/3 of the senate only increases the number a little.

          • 1soru1 says:

            Only $1.1 trillion is discretionary spending; doing anything to the rest (social security, etc) is no more politically and constitutionally feasible than simply confiscating private wealth by statute.

          • @1soru1:

            We are assuming coordinated action by a majority of both houses, possibly a 2/3 majority of the senate, plus a majority of the Supreme Court. Under those circumstances, what limits them to the “discretionary budget”?

          • 1soru1 says:

            The same thing that limits them from raising income tax to 90%, nationalising the banks, etc. They need to win elections to stay in office, and they need to hold elections to keep the support of the military.

          • Toby Bartels says:

            If you're going to ignore precedent and just stick to the structure of the Constitution, then there’s no need for a supermajority in the Senate. More than 50% (or 50% and the VP, who is otherwise not needed for any of this) is sufficient to change the Senate rules.

      • It’s not a given that you have to have huge wealth disparities in the first place.

        Libertarianism has a threefold problem

        1. It encourages wealth disparities by discouraging redistrubution.

        2, It allows absolutely huge amounts of , coercion by private individuals.

        3. It has private law as the final arbitor, and therefore rests on the assumption that the law enforcement agencies would never be so venal as to be bribed by Evil Rich Guy.

        Also, I’d really like to know how even a non-libertarian society bans that. Pass a law that it’s illegal to pursue vengeance even if you make sure to use methods that aren’t illegal?

        Someone sufficiently obnoxious tends to find themselves being prosecuted under edge-case or catch-all laws. Or asking for a lock of Hilary Clinton’s hair really is a central example of a physical threat.

        • Thegnskald says:

          You do understand that libertarianism is a different set of political beliefs than anarcho-capitalism, right?

          The most extreme version of libertarianism, minarchism, still doesn’t have private law as the final arbiter.

          It is possible to be both libertarian and anarcho-capitalism, but they describe different things; one is about the proper relationship between the rulers and the ruled, and the other is about the best way to structure rulership.

          You can be a libertarian (or socialist) monarchist, for another example of the difference.

        • Toby Bartels says:

          Someone sufficiently obnoxious tends to find themselves being prosecuted under edge-case or catch-all laws. Or asking for a lock of Hilary Clinton’s hair really is a central example of a physical threat.

          That particular obnoxious person had already been convicted of fraud when he asked for a lock of Clinton's hair. Jailing him for a ‘solicitation of assault’ that was not taken seriously by anyone was not the valiant efforts of an only slightly corrupt state to use a catch-all law to stop somebody otherwise unreachable; it was a vindictive abuse of state power by a corrupt judge to further punish somebody whom the judge personally disliked and who was already down. His being obnoxious, even fraudulent, in other ways (which he certainly was) does not and cannot justify it.

        • ragnarrahl says:

          “Someone sufficiently obnoxious tends to find themselves being prosecuted under edge-case or catch-all laws”
          That’s not the guy PURSUING vengeance getting prosecuted. That’s the bureaucracy serving as your scenario’s Infinitely Rich Man.

        • sharper13 says:

          Regarding “wealth disparities”:
          With a monopoly government type of system, you have at least one massive power and wealth disparity, the people running the government vs. everyone else.

          So I think you’d have to first explain why that setup is ok, but that having an opportunity for other people to improve the lives of others with voluntary transactions in order to gain more wealth for themselves than other people have isn’t ok. Wouldn’t politically selecting people to control a massive wealth and power disparity be less, not more, likely to produce abuses? I mean, we at least have examples of those abuses actually occurring over and over again around the world throughout history, right?

      • DocKaon says:

        By limiting the wealth of individuals through progressive taxation and making a lot of easy options to use or going after common targets illegal. Fraud, barristry, libel, defamation, tortuous interference, anti-SLAPP, and whistle-blower laws are all tools to prevent the wealthy, powerful, and popular from attacking the weak and unpopular. They are clearly imperfect. Harvey Weinstein demonstrates that a powerful person can get away for a long time with incredibly bad behavior in the real world using this sort of threat. Of course, nothing in the real world prevents someone offering anti-vengeful rich guy insurance so why would you expect to exist in libertarian world?

        • ragnarrahl says:

          Because we don’t spend our time worrying about infinitely rich guys. Our resources are spent worrying about the bureaucracy. Some forms of insurance do address that in limited ways. Take that threat away and we can focus on less likely threats.

        • Cliff says:

          Is the enormous cost of progressively taxing away high wealth worth the potential benefit of avoiding an evil Bill Gates who uses his billions to ruin individuals (while enriching others around him) instead of philanthropy? All those other things we would still have under a libertarian political system.

        • Consider, for a real world case, a local government that wants to force home owners to sell to a buyer the mayor favors. It does so by imposing large fines on any home owner who doesn’t for trivial but real offenses–not having the lawn mowed this week and the like.

          The more general point being that the problem you are looking at requires someone with a lot of power, and that is a much more common situation in a governmental system. A local mayor can do it. The rich man case requires someone who is not merely rich but enormously richer than anyone else–probably richer than a Gates or Buffet in the real world.

          • jw says:

            In fact cases of a powerful politician ruining someone’s life out of spite are pretty common.

            So since the equivalent libertarian strawman example requires a mythical infinitely rich person.

            Point, libertarianism.

          • 1soru1 says:

            The most mundane and common example of abuse of power to rule someone’s life is a business owner sexually harassing an employee.

            Like car crashes, it rarely makes the headlines, because it happens every day.

          • John Schilling says:

            The most mundane and common example of abuse of power to rule someone’s life is a business owner sexually harassing an employee.

            There aren’t enough business owners for that to be the most mundane and common example of anything. Most sexual harassment is conducted by employees. Is there any data on the relative rate of sexual harassment in private corporations vs. government bureaucracies?

          • anonymousskimmer says:

            I don’t get exactly what you’re arguing for or against?

            We have this in the US currently – it’s called Kelo v New London.

            Is this an argument against eminent domain as it’s an easily abusable process by the government?

            Aren’t finable offenses subject to citizen review and court review?

            Didn’t we have corporate red-lining (as well as government red-lining)? Wouldn’t corporations still red-line (/or otherwise create odious contracts with heritable clauses) if the courts hadn’t ruled it unconstitutional (/contrary to the first sale doctrine)?

          • John Schilling says:

            Didn’t we have corporate red-lining (as well as government red-lining)? Wouldn’t corporations still red-line [etc] if the courts hadn’t ruled it unconstitutional?

            Some corporations would, some wouldn’t. But there are many corporations where there’s usually only one government. Maybe two in the United States. But it’s generally easier to find a different corporation to do business with than to find a different government to do business with, and less likely that all corporations will conspire against you than that the government will decide against you.

            Note that the historical redlining that exacerbated the effects of racism in e.g. 1940s Chicago, worked in part because it was coordinated by a cartel explicitly chartered by the federal government to make sure mortgage funding went to the Right Sort of Neighborhoods. Any bank that didn’t play by their rules, didn’t get federal insurance for its loans.

          • anonymousskimmer says:

            I’m aware of the historical red-lining.

            Technically we have three levels of government in the US (usually) – the municipal governments do lobby, sue, and otherwise push back against the state and federal governments.

            (Aside: This is my argument that if Janus v. AFSCME is passed the way it is expected to pass that people should be able to opt-out of the local and state taxes which are used to lobby the state and federal governments. Especially the local taxes, as local governments have zero recognition in the constitution.)

            But it’s generally easier to find a different corporation to do business with than to find a different government to do business with, and less likely that all corporations will conspire against you than that the government will decide against you.

            Under a libertarian framework this leads to unequal balkanization (especially when the corporations are the house builders); is this a better situation than governmental discrimination? The latter leads to the various suffrage and civil rights movements as a means of resolution, the former tends to lead to actual war as a means of resolution.

          • Toby Bartels says:

            This is my argument that if Janus v. AFSCME is passed the way it is expected to pass that people should be able to opt-out of the local and state taxes which are used to lobby the state and federal governments. Especially the local taxes, as local governments have zero recognition in the constitution.[emphasis added]

            Why if? That would just bring local governments to the position that government-employee unions are in now.

          • John Schilling says:

            Under a libertarian framework this leads to unequal balkanization (especially when the corporations are the house builders); [this] former tends to lead to actual war as a means of resolution.

            1. What wars are you thinking of as examples?

            2. It absolutely does not take a corporation to build a house, and to the extent that corporations do build houses they are usually sufficiently small and numerous that cartelization is impractical.

          • anonymousskimmer says:

            @Toby

            Why if? That would just bring local governments to the position that government-employee unions are in now.

            Right now public employee unions can directly lobby the government (as embodied in their employer) using fair share fees. At least those PEUs which are in fair-share states.

            @John
            2) I agree, but probably the majority of housing today now takes place in large sub-divisions by single builders. Empty single plots of land aren’t that frequent in most cities.

            1) Libertarianism hasn’t really existed, so I was thinking in terms of wars between balkanizing states such as the US Civil War, the various battles between US and Canadian citizens, and things of that nature. When it is your government you do have collective power to change it, when it is someone else’s government it’s much more difficult, when you don’t like all of the someone’s who live next door it’s a Hatfield v McCoy situation.

          • Toby Bartels says:

            I see your point; I hadn't realized that the anti-union side was framing the argument in that way, but now you got me to read up on all that, so I'm grateful for that. In any case, I like your suggestion.

          • anonymousskimmer says:

            Thanks 🙂

          • John Schilling says:

            Empty single plots of land aren’t that frequent in most cities.

            Empty plots of land aren’t that frequent in cities because, being cities, people have already built things like houses and apartment buildings everywhere they will fit. At any given time, hundreds or thousands of the existing houses will be for sale by their individual owners and hundreds or thousands of apartments will be available for rent from small-business landlords. It would require a grossly implausible conspiracy for them to all privately refuse to sell or rent to a particular individual.

            And if you must have a new-build house, you don’t need to go that far out of most existing cities to find vacant plots of land. So it would seem to take a very narrow and peculiar demand for housing, to place one entirely at the mercy of large real-estate development corporations.

            I was thinking in terms of wars between balkanizing states such as the US Civil War

            So, you’re using the American Civil War as an example of how bad things can get if we let corporations have too much power, and therefore we need to have as strong central government in charge? Because I’m thinking that’s a bit of a stretch.

          • anonymousskimmer says:

            @John

            Fine, show me an example of a libertarian-governed nation and I’ll show you what’s wrong using examples solely from libertarianism.

      • Protagoras says:

        There are existing laws which criminalize patterns of behavior, the individual components of which are separately legal, including some of the laws against harassment that seem directed at exactly this kind of thing.

        • Alexander Turok says:

          I was about to say exactly that. Another example is blackmail laws. It’s legal for me to tell your wife you’re cheating on her. It’s legal for me to ask you to loan me 1,000$. It’s not legal for me to threaten A if you don’t do B.

          Laws against tortious interference and predatory pricing are based on a similar concept, actions that are otherwise legal can be punished if the intent is to destroy someone’s business.

      • anonymousskimmer says:

        Bring back dueling.

        No, ideally not to the death. Not necessarily with guns. Just the ability to say that you have a grudge and want to duke it out with someone.

        Regulated fighting between people who have grudges against each other has a place.

        Deal with it one-on-one (or small group on small group, if that’s your kick), and it can be resolved without blowing up into a societal mess.

        You’ll likely get far, far more sincere apologies than you do now – people might actually be incentivized to see their actions from the other person’s point of view, and realize how wrong they were.

      • b_jonas says:

        > Also, I’d really like to know how even a non-libertarian society bans that.

        I have a partial answer.

        The infinitely rich man offers to pay for your wife’s expensive medical treatment. We solve this by having mandatory health insurance which pays for the expensive medical treatment of everyone. It pays for regular dialysis of people who have to get that for their whole remaining life, which might last for years. I don’t think it pays for any treatment that costs 100000 dollars a month for the rest of your life, but when private individuals offer such ridiculously expensive treatments, it’s are more often than a confidence trick that extracts money from very desperate people. Sometimes people have go to private clinics for medical treatments that are expensive but not that expensive, but we usually consider that a bug in the current implementation of the healthcare system.

        The infinitely rich man makes the protagonist unemployable by buying all castanet factories. We solve this with some general measures helping people who are unemployable for more mundane reasons. There are a lot of jobs offered directly by the government. This is not really by design, it’s just a side effect of everything else a non-libertarian government wants to do, such as healthcare and education controlled by the state, bureaucracy, workers sweeping cigarette butts thrown on the street because people don’t bother walking five meters to the nearest trashcan, etc. These jobs are varied enough that almost any adult in good health can work in one of them. A period of education might be required first, but the government also pays for the education. The government-controlled jobs have much stricter rules for how they are allowed to discriminate potential employees than what private businesses. A government-controlled workplace is not allowed to fire or refuse to hire you just because you are against horses. And for people who are unemployable even after attempting such government-offered jobs (which happens the most often for people living in the country where commute to any city is prohibitive), the government pays unemployment benefits to support them.

        The infinitely rich man buys the gas company so that the protagonist cannot heat his home. Our society tries hard to avoid people freezing to death, so it has measures against this. The gas company operates under state control, and can’t suddenly stop serving you or ask an unreasonably high price. And if the protagonist decides to move to a cheaper apartment because he no longer gets the high wages of a castanet factory worker, he can still be sure his home will be properly heated. This is because the government has laws similar to minimum wage laws for minimum requirements on what homes can be built. Without those laws, a construction company would decide to bid under other companies by building a house where heating is so bad that about once in ten years, when the winter is particularly harsh, people freeze to death. Desperate poor people might still buy such a house hoping that their financial situation might improve before such a winter arrives. In reality, some people still freeze to death in their homes in particularly harsh winters, but we consider that a bug in our implementation of the government.

        The infinitely rich man turns the environment of the protagonist’s house to a desolate waste land. I don’t believe we have good protection against this. Some real homes suddenly lose a lot of value without anyone’s intent on vengeance.

        The infinitely rich man orchestrates events that make the protagonist’s car obsolete. It also makes most other people’s cars obsolete. I am quite sure that we don’t have good enough protection against this, because something just like that has happened. The television broadcast companies have discontinued analog television broadcast, so anyone who has a sufficiently old television can now no longer use it, they can’t even watch the channels ran by the state. This has affected a lot of people. To continue watching television, you need to buy a new device next to your television from your own money. The marketing campaigns said that this was OK because you only have to do it once, and you can keep using the device forever, but there’s nothing stopping the broadcast company to do a similar change again twenty years later. The protagonist will have to buy a new electric car, or make do without a car.

        The infinitely rich man pays all the protagonist’s friends to never contact him again. We don’t have good protection against this happening. The other Scott’s blog says that some people’s lives can be made miserable by online harassment, that all your friends will shun you because of a campaign on twitter turning them against you. If you hire someone who knows how to manage such campaigns, you might be able to ruin someone’s life for much cheaper than what the infinitely rich man did. We have some laws against slander that are trying to stop this, but they don’t really work except against very stupid opponents.

        Most of the trouble that has fallen onto the protagonist could happen even without anyone acting in vengeance. You can get very ill and have to pay for an expensive treatment. Your wife may leave you through no fault of your own, and you can become very lonely from that. You can become unemployed because the job you’re qualified for is outsourced to third world countries. Your home can lose value, the technology you own can become unusable. Everyone can hate you because someone successfully does a campaign that makes everyone believe that everyone who cooks meatballs with cinnamon is irredeemably evil, and you just happen to like cinnamon that much.

        In the hypothetical libertarian society that Scott describes, you can probably buy insurances against some of these problems. You can buy a health insurance that will pay for your wife’s expensive treatment if he gets a rare illness. The protagonist will likely already have owned such an insurance. My question for you all in turn is this. Which of the other situations could you avoid by buying an insurance in that libertarian society? Can you buy an insurance that replaces your obsolete technological devices? Can you buy an insurance that makes your friends not leave you just because cinnamon eaters turn out to be the most hated group of the next decade?

        • We solve this by

          A simpler version of your answer would be “we solve this by not permitting it to happen.”

          This gets back to my point about the difference between arguing for outcomes and arguing for institutions. You are doing the former. Thus for instance you write:

          We solve this by having mandatory health insurance which pays for the expensive medical treatment of everyone.

          That’s an outcome. Since there is a finite amount of medical resources, someone has to decide whether a claimed medical need is legitimate–if the person is really unwell (consider the question of whether wanting sex reassignment surgery counts as a medical need) and if so what treatment would work and be worth funding.

          So in your society the evil medical bureaucrat who wants to get you informs your wife that unfortunately her illness does not qualify for help, or only for a medicine which in fact doesn’t cure it–but if she left you he might be able to reconsider the matter. Similarly for your other cases. None of them require a more implausible scenario than the infinitely rich man story in a hypothetical libertarian society.

    • It is not clear that utilitarianism is the one true ethics.

      Allowing huge disutility against the few for the benefit of the many is what is criticised in Omelas, and what constiutional constraints are partly intended to avoid.

      • Yaleocon says:

        Well yeah, I’m no utilitarian. But “utilitarian” and “consequences don’t matter at all” are not your only two options. I favor one of the options in the middle, where consequences matter, along with many other things (double effect, duties, desert, even virtue).

        And as for “disutility against the few for the benefit of the many”, yeah, that can be bad. But if your only alternative is “disutility against the many for the benefit of the few”, the situation becomes more complex. What are the alternatives in Omelas, and how could they be brought about, and by whom? The story is notably lacking in providing those details.

        To me it’s always seemed like it’s more a parable people use to feel morally self-righteous–“oh yeah, I’m totally One Who Would Walk Away, look at me”–rather than a situation to think about seriously and philosophically.

    • Anaxagoras says:

      Yeah, I was noticing that reading the scenario. Okay, filling in the lake and replacing the neighboring lots with landfills is probably socially bad, and so is buying up everything and preventing them from providing you with services.

      But engineering advances in automotive technology, paying for your wife’s treatment, and offering your friends ways out of financial ruin (provided he did not engineer that ruin himself) are all good things. I’m not sure why Robinson included those in there; they don’t make the scenario any more fair, but they do make the outcome way less bad than he could have had it be.

      • sharper13 says:

        Some of the obvious responses to that were left out of the (excellent) reply.

        If you live in a house on a lake, you should have purchased rights to preserve/use/protect/look at that lake when you bought your house, likely coming as part of the deed. If you didn’t, then it’s a bit much to complain later when someone else buys the lake you didn’t want to buy part of the rights for and does something else with it.

        Similarly, you should plan ahead and contract for employment terms, such as when you can be fired. Contract for services, including renew-ability. Contract for road use rights to be able to travel to and from your property. How many people in the real world are going to buy land and a house with no road rights and thus no contracted right to enter/exit it? It’s really difficult to sell property that doesn’t come with access rights as part of the deed.

        That whole part of the scenarios makes a lot of straw man assumptions about how a world with privately enforced contract rights would work, while ignoring how these types of things have already worked in the real world of property rights.

        • MugaSofer says:

          If you live in a house on a lake, you should have purchased rights to preserve/use/protect/look at that lake when you bought your house, likely coming as part of the deed. If you didn’t, then it’s a bit much to complain later when someone else buys the lake you didn’t want to buy part of the rights for and does something else with it.

          Even if they did it for no reason but to spite you?

    • Edward Scizorhands says:

      I was sure this was a Robinson piece, but, no, this was a combined effort by Current Affairs. They brought The Best And Brightest out for this one.

  9. shakeddown says:

    I was disappointed. When I read the title, I thought it was going to be Scott taking the premise and running with it in an actually interesting direction to show how comparatively unimaginative the current affairs article was. Not responding by being just as unimaginative as it.

    • Toby Bartels says:

      I thought that Scott's post was justified because it is funny. (Isn't that on the list? True, necessary, kind, or funny?) It's not a serious rebuttal, but then the original article was not a serious article either. (Also, Scott was more imaginative; you can tell because he went meta.)

      • Edward Scizorhands says:

        I can see the CA editors (and some commenters here) stomping their foot and saying “no, no, no, take us seriously.”

        Scott has a whole Anti-Libertarianism FAQ so it’s not like he’s incapable of seeing the problems. But he recognizes an attempt to waste one’s time as an attempt to waste one’s time.

        • Toby Bartels says:

          And we'll take them seriously when they write serious articles. Which they do! This just wasn't one of them (and I didn't get the impression that it was meant to be either).

    • Lasagna says:

      I was disappointed. When I read the title, I thought it was going to be Scott taking the premise and running with it in an actually interesting direction to show how comparatively unimaginative the current affairs article was.

      I thought he did exactly that. Scott’s article is funny, and treats the CA article with the appropriate level of seriousness (very little).

    • CEOUNICOM says:

      “unimaginative”

      You keep using this word.
      (insert Inigo Montoya)

  10. cvxxcvcxbxvcbx says:

    There are probably a lot of people who just have never heard anyone argue with charity before. Or at least, not in any context that has anything at all to do with politics.

  11. Michael Cohen says:

    “Suppose there’s an evildoer who punishes all evildoers who do not punish themselves. Does the evildoer punish himself, or not?”

    *who punishes all and only all evildoers

  12. Tedd says:

    Scott, this thing you do where you have an obvious stand-in for the other side in a debate say stupid things leaves a really, really bad taste in my mouth. You’re a good writer, but that makes these sorts of rhetorical tricks worse, not better.

    And the worst of it is, you’re the person I first encountered who clearly defined the worldview rejecting this sort of thing! Do you really think you could look Nathan (or whoever) in the eye and say that this was a charitable interpretation of his article, and not just a sneering caricature?

    • Cliff says:

      Who is the obvious stand-in for the other side who says stupid stuff? He quotes the actual article, that’s the other side. The guy making the same argument about democracy as was made about libertarianism isn’t a stand-in for the article author, right?

    • LadyJane says:

      The Current Affairs article was genuinely bad enough that it deserved this treatment. Scott responded to strawmanning with strawmanning, and I don’t see a problem with that. This article was not a good-faith attempt to convince anyone that libertarianism was flawed (and if you want an example of what an actual good-faith argument against libertarianism looks like, check out Scott’s Anti-Libertarian FAQ). It was a cheap attack on a nonsensical bastardized version of libertarian philosophy that no one in real life actually believes in, centered around what seemed like very deliberate misinterpretations of libertarian ideas. It had no purpose except to let the author and his/her ideological brethren feel good about themselves by laughing at just how ridiculously stupid-evil-wrong libertarians are.

  13. userfriendlyyy says:

    I agree that is a poor straw man attack. For a much better critique of libertarians check out this 6 part series.
    Journey into a Libertarian Future
    1. 2. 3. 4. 5. 6.

    4, 5 ,and 6 are the best.

  14. MawBTS says:

    Fuck you’re funny.

  15. Lillian says:

    “Suppose there’s an evildoer who punishes all evildoers who do not punish themselves. Does the evildoer punish himself, or not?”

    He punishes himself in his capacity as an evildoer, but not in his capacity as an evildoer who punishes all evildoers who do not punish themselves. Paradox solved.

  16. Jack V says:

    I am… kind of lost. I agree those criticisms are kind of bizarre. In particular #1, we don’t need some bizarre wolf-hypothetical. “People were systematically discriminating against group X so much that group X had serious problems being sold basic day-to-day things even though you’d think it was against their economic interest to do so, and the government had to actively ban doing so” is an actual thing that actually happened during segregation.

    But I’m thinking of the motte and bailey thing. For many, many viewpoints, there’s a spectrum between “people who hold a mostly sensible version of it, whether or not it’s sufficiently distinct from mostly sensible interpretations of other political systems” and “people who actually believe a stereotype parody version of it”. It’s usually the case that the people holding the more sensible interpretation say, “this obviously what everyone really means, don’t spend time debunking the crappy version, no-one really believes that you’re just straw-manning us”. And people who believe the crappy version spout it loudly all the time whatever anyone says.

    And that’s true, whether the sensible people or the parody people are actually a small minority. How do you tell whether a particular viewpoint is “a sensible movement with a few high-profile crackpots” or “mostly crackpots with a few high-profile philosophers who grafted a sensible meaning onto the rhetoric, even though no-one really acts like they believe that?”

    In motte and bailey terminology, all viewpoints have a motte and bailey. Sometimes the motte is widely known, clearly useful, even though, like all worthwhile ideas, it’s surrounded by an umbra, a bailey, of people aping its superficial aspects without really understanding it. Sometimes there are crappy ideas that are basically all bailey (i.e. something people believe without really any redeeming features) although its adherents can sometimes cobble together a plausible motte afterwards and pretend that’s what they really mean if they’re pressed.

    How do you judge what to engage with? Do you judge how widely the bailey is believed? (I seem to meet more straw libertarians than what Scott describes as real libertarians?) Do you judge how correct and non-facile the motte is? (All the things Scott describes real libertarians believing I think are good points, and I know people who correctly point out that sort of thing, but all the libertarians who achieve any sort of political success seem to be people who are socially progressive, but economically what I describe as pro-corporation.)

    • Lillian says:

      “People were systematically discriminating against group X so much that group X had serious problems being sold basic day-to-day things even though you’d think it was against their economic interest to do so, and the government had to actively ban doing so” is an actual thing that actually happened during segregation

      Other way around, there were so many self-interested white capitalists willing to do business with blacks the Southern states had to pass Jim Crow laws to keep them from doing so. Even then there was enough loophole abuse that armed mobs felt compelled to terrorise defectors. They were of course much more willing to maim and murder blacks who got “uppity”, but plenty if white business owners were threatened with arson and vandalism if they did not maintain segregatory practices.

    • Nick says:

      There’s an unresolved tension between talk of weakmen and talk of mottes and baileys, something that has bothered me since Scott popularized the latter term. My suggestion is to engage with actual people’s actual beliefs, one or a few at a time, and not a generalization in your head that says 21% of libertarians are Cartoon Parody Libertarians so you should be 21% unfair to them. If the libertarian you’re talking to is a Cartoon Parody Libertarian, go ahead and pull a Current Affairs on them. If the libertarian you’re talking to is David Friedman, repent and sin no more have some more interesting objections. And the reason this works is that, as you point out, movements are always ‘guilty’ of having a motte and a bailey, which makes accusations against the movement as a whole vacuous—but that’s not true of an actual person, who has no good reason to defend the bailey, and whom you can call out if they retreat to the motte.

  17. mseebach says:

    I think the first puzzle can compellingly be addressed on it’s own premises.

    First, base case: If a society exists, regardless of its form of government or political philosophy, regardless of what provisions its constitution contains, where literally every single individual believes strongly that cannibalism is correct and just, and John should be eaten, John will be eaten, one way or another.

    Now, in the perfect libertarian utopia, if even a single other person believes that John shouldn’t be eaten, this person can invite John to live on their property, and John will now not be eaten. John and his friend is free to advocate to whomever to join them, and, because people generally aren’t monsters, some people eventually will. If no-one else, they’ll have a good chance to recruit the next 18-year-old next year. This outcome is perfectly consistent with the premises of the puzzle.

    To depart a bit from the puzzle, let’s consider the corollary: In a democratic society with the same beliefs, eating John is the law. Even if the law is outdated, and only maybe 30% of the population actually feel very strongly about it, the law is there, and damn the consequences, SWAT-teams will pick up John if he doesn’t present himself for eating. There is no non-agression principle, John’s individual feelings about being eaten are regrettable, but irrelevant in the face of the legitimate needs of the community. This happens year after year, and the vast majority of citizens cringe a bit when it happens, but the issue just never seems to make it to anyones political agenda, because very few voters are willing to swing their votes based on this issue. After all, this is just one person, and there are issues of gun control, immigration, taxation, foreign policy and trans-people serving in the armed forces on the line here. So John gets eaten, and the next year another guy gets eaten, and people online have principled arguments about the social contract and the merits of moving to Somalia.

    At least for the isolated purposes of the puzzle, the libertarian society will allow for a better outcome for John than any society that allows the collective to coerce the individual in any way. Of course, the utopia described is still a straw man, it seems highly unlikely that a society that is willing to fully ostracize John for his unwillingness to be eaten would come to believe in the non-agression principle in the first place.

  18. And it just got me thinking – what if this whole world is just a thought experiment by a communist with a freshman-level understanding of political philosophy trying to weak-man libertarianism? And then I thought – frick, I better get some really good vengeful-rich-person insurance.

    Oh Scott, you magnificent bastard! I laughed abruptly and loud enough that I’m worried I startled my neighbours.

    I grant this kind of snark doesn’t really suit you, but A+, would read and laugh again.

  19. rahien.din says:

    You’re just blowing raspberries at people you think are stupid.

    I’m still undecided as to libertarianism (or my location along its axis). Every time I ask if the soul of libertarianism is preventing coercion, why doesn’t it prevent coercion? and if people are naturally-good enough to mitigate the dangers I see in libertarianism, why aren’t they naturally-good enough to mitigate the same dangers in other systems?, I never get an informative answer. I get a lot of government-is-an-evil-magical-force-and-we-hates-it. And then I think but government was chosen and designed by libertarians who didn’t want to be libertarians anymore.

    And then, I update farther away from libertarianism. Kind of like now.

    • sohois says:

      Though I’m not the author, I would be quite surprised if this was intended as an entreaty for libertarianism. Making fun of some bad arguments is never going to qualify as a strong argument in the other direction, it just forewarns anyone against taking such weak men seriously.

      for that matter, stating that you updated away from libertarianism appears a very strange reaction. I can’t comprehend what new evidence you got that leaves you less convinced in the political position. It’s a comic article. Even if you don’t find it particularly amusing, or you think that there are unaddressed arguments against libertarianism which this avoids, there should not be any new information here that leads to you change your confidence in any way.

      • rahien.din says:

        sohois,

        I agree that these are funny stories, and that the real butt of the joke is bad journalism. They are not intended to explain or describe utilitarianism.

        I have genuine curiosity about libertarianism (and, snarky and smug as this stupid article is, it aligns with some of my questions). The answer I get or observe is “But consider [certain convenient contingency]” or “Well that would never happen” or “We can solve that specific problem with [certain exotic protocol]” or “Libertarianism still wins because, uniquely among all institutions and organizations, government magically makes everything worse.” These responses are devoid of information.

        Scott’s stories, at least, are not intended to respond to requests for information… but they’re isomorphic to those other responses.

        So I am not judging them on the grounds that they are explications. I’m judging them like I would judge a medical test. Whenever I run the [listen to a libertarian] test, the result is “applause light,” and this is an apparent feature of libertarianism. I couldn’t tell you why that is, but I’m well-justified in updating away from it.

        • Cliff says:

          “Whenever I run the [listen to a libertarian] test, the result is “applause light,” and this is an apparent feature of libertarianism. I couldn’t tell you why that is, but I’m well-justified in updating away from it.”

          What?

          • rahien.din says:

            In my experience, libertarians seem to be bad sources of information about libertarianism.

            There are a variety of ways I could interpret that phenomenon. Most of them lead me away from libertarianism. Some could be neutral. None leads me toward libertarianism.

        • quanta413 says:

          So I am not judging them on the grounds that they are explications. I’m judging them like I would judge a medical test. Whenever I run the [listen to a libertarian] test, the result is “applause light,” and this is an apparent feature of libertarianism. I couldn’t tell you why that is, but I’m well-justified in updating away from it.

          I’m curious about what philosophy you find this isn’t true of because your response reeks of motivated reasoning. I agree with very little Marxists think about politics or that Marxists have done, but I can find Marxists who make coherent and reasonable points. Most people arguing for any political philosophy will give… not terribly good responses. You have to seek out people who actually thought about it seriously for a while and read their positive arguments on their own ground to learn anything about it. If you’re pinging random strangers on the internet, you will never learn anything.

          Do you think Bryan Caplan, Scott Sumner, and Tyler Cowen make no valid argument for libertarianism?

          • quanta413 says:

            @self

            Too late to edit my response to be less rude and I apologize to rahien.din, but I’ll add some more constructive things.

            For Marxists whose perspective I’ve found helpful, Freddie DeBoer and Domenico Losurdo (Italian Marxist who wrote Liberalism: A Counter-History).

            I’ve honestly not read a lot of cogent critiques of libertarianism per se, but I think most of the good points I’ve read against liberalism applied equally well to libertarianism. There’s not actually a huge amount of daylight between the mainstream of their philosophical ideas.

            Rigid libertarian deontologists don’t interest me much for the same reasons revolutionary communists wouldn’t if revolutionary communists hadn’t “succeeded” enough times now that it seems like I ought to pay attention. If libertarianism was reached by rapid destruction of the existing order the same way communism was, I can’t image the results would be good. It’s just a question of how many millions of bodies. I believe that culture and politics have to be in concordance to some extent and you can’t really have a functioning political system without a matching culture.

          • rahien.din says:

            If you’re pinging random strangers on the internet, you will never learn anything.

            This is the exact opposite of my experience. Pinging random strangers on the internet to learn things is why I registered for SSC in the first place. What are you here for? To learn nothing?

            I don’t think you are here to learn nothing, because you describe the “ping random strangers” model very nicely :

            I agree with very little [adherents to X] think about politics or that [adherents to X] have done, but I can find [adherents to X] who make coherent and reasonable points.

            Where X = some philosophy or position I don’t fully agree with

            I have found that works as a pretty good tertiary source.

            I’m curious about what philosophy you find this isn’t true of.

            (Let me gently suggest that this is not a valid response. There is a relativity to this quality. “When people thought the earth was flat, they were wrong. When people thought the earth was spherical, they were wrong. But if you think that thinking the earth is spherical is just as wrong as thinking the earth is flat, then your view is wronger than both of them put together.”)

          • quanta413 says:

            What are you here for? To learn nothing?

            I suppose it depends what I want to learn about. Learning about political or moral philosophy is nice when it happens, but not my goal. On top of that, I spend far too much time in amusing arguments, which would reveal me to be lying to myself if I thought that was really the main reason I was here. If my primary goal was to learn about a topic in and of itself, my time would be much better spent with books.

            The main things I learn about here are different. The most common thing I learn is the existence of certain points of view or communities. I am very interested in this. If I find a point of view novel enough and want to learn it in any meaningful depth, I will go find a book about it or go read a blog. The second thing I learn is reams of trivia about all sorts of things. Battleships, guns, economics, law, etc. The third thing I learn is where my beliefs are wrong or weak, but I find I usually learn this when playing defense so to speak.

            But I don’t think I usually gain much here if my goal is to probe some specific system that I’m not already familiar with. If I want to do that, I get a book. If I was very lost, I could probably get very good book recommendations here for many topics. Attacking is much easier than defending and understanding the validity an attack is much easier than understanding the validity of a defence, so by attempting this through argument I am likely to mistakenly think I am much cleverer than I am and every philosophy much stupider than it is. Especially since I may even lack the background to understand the defense (which may be totally correct).

            Let me gently suggest that this is not a valid response.

            Sure, it is not a logical argument against. But neither should the fact you find nothing but applause lights cause you to update in any way. Especially when almost all political arguments are applause lights. Or maybe it is only that I perceive most of them that way. So I am curious who or what you find doesn’t behave that way in public politics. I find almost all vague philosophical political arguments consist of applause lights. If I read a Paul Krugman column, there’s something like a 50% chance it’s applause lights, and I’m pretty sure he could do better than that if he wanted to. If I read a random NYT, Post, or WSJ editorial, I think I’m taking similar odds.

            EDIT: Also, I just like arguing with people as long as there’s a decent chance they’re vaguely sportmanslike about it. I like arguing a lot. I almost left this out. It’s pretty important as a psychological motivation.

          • rahien.din says:

            I like arguing with people too! This is one of the few situations in which I have found it doesn’t work. I am beginning to believe that libertarianism is a feeling.

            Sure, it is not a logical argument against.

            I go farther than that. I don’t think you can say it at all. If I say “I have found that palm frond roofs leak more than other roof types,” and you reply, “I’d be curious to know which roofs you have found that never leak,” you have not made a valid response.

            I’m curious about what philosophy you find this isn’t true of

            I am curious who or what you find doesn’t behave that way in public politics.

            Quite honestly, I am trying very hard to avoid providing a list.

          • quanta413 says:

            I go farther than that. I don’t think you can say it at all. If I say “I have found that palm frond roofs leak more than other roof types,” and you reply, “I’d be curious to know which roofs you have found that never leak,” you have not made a valid response.

            I think of it more like you said “I don’t like this system because it’s not 98% efficient at energy conversion.” I think you are asking for something not technically impossible, but so unlikely that it’s worth asking what better alternatives you have in mind.

            The main difference in my mind is that you are either

            (less likely possibility) (1) making a very subjective claim that all libertarian arguments you see are applause lights yet your central example seems to be “random guy I pinged on the internet”. It is technically possible this has happened and you have never read someone who’s not preaching to the choir, but I find that much, much harder to believe than other possibilities. Because I haven’t yet found a belief set equally broad that can’t actually manage this on accident after enough tries no matter how much I disagree with it or think it morally craven. It’s possible I’m just willing to engage in more creative interpretation though. This pattern would also strike me as an incredibly bad way of investigating anything as I’ve already mentioned.

            or

            (more likely possibility) (2) Engaging in hyperbole when your real claim is more like “libertarians are more likely to engage in applause lights than (some other group).” But neither you nor I is doing anything remotely resembling an unbiased survey to answer this question. This possibility is partly why I ask “who do you think this is not true of?”. If your only answers to fill in “some other group” are basically “people I agree with”, then this is pointless, and I can save a lot of time by not having this conversation.

            And I have found authors who make non-applause light arguments and it wasn’t hard for me (but hey maybe that was luck). I’ve mentioned them and will again later. Tell me what you think of them.

            Quite honestly, I am trying very hard to avoid providing a list.

            I don’t really intend to respond to the list either way, so I promise not to talk about the list even if you give it. But a more positive question if you like

            I am beginning to believe that libertarianism is a feeling.

            Have you read any books by David Friedman or Bryan Caplan or Friedrich Hayek? Any blog posts by Scott Sumner or Tyler Cowen? Although I think Cowen’s style has a tendency towards… mysticism. Maybe that’s what you mean by applause lights; I’m not a huge fan.

            If so, would you claim that their arguments are based upon a feeling rather than a philosophy roughly as coherent as liberalism or conservatism as those philosophies are exercises by real people and not as platonic ideals? I’d say David Friedman and Bryan Caplan are freakishly close to following a platonic ideal to the point where that’s a lot of why I disagree with their absolute positions even if I often agree on the margins. I’m too small c conservative to be up for that sort of radicalism.

            I’m not saying they are right. I don’t agree with everything in libertarianism. But it’s not that esoteric of a philosophy; it’s not immensely different from liberalism if you don’t go full anarcho-capitalist. For example, I found The Road to Serfdom to be a shockingly mainstream book by modern standards compared to all the nonsense I had seen written about what it supposedly was. Hayek explicitly lays aside the possibility of cradle-to-grave welfare as being a possibly valid government program for the book at least (maybe he says differently in other places) and spends all his time arguing against basically Soviet-style Collectivism and Fascism. I found it stunningly mainstream and with strong overlap with most American sources I had read on the left, center, and right.

          • 1soru1 says:

            The thing is, Hayek’s ‘other writings’ contain parts where he himself explained what he really meant by _Road to Serfdom_, with the benefit of hindsight, for example.

            At the time I wrote [The Road to Serfdom], socialism meant unambiguously the nationalization of the means of production, and the central economic planning which this made possible and necessary. In this sense Sweden, for instance, is today very much less socialistically organized than Great Britain or Austria, though Sweden is commonly regarded as much more socialistic. This is due to the fact that socialism has come to mean chiefly the extensive redistribution of incomes through taxation and the institutions of the welfare state. In the latter kind of socialism the effects I discuss in this book are brought about more slowly, indirectly, and imperfectly. I believe that the ultimate outcome tends to be very much the same, although the process by which it is brought about is not quite the same as that described in this book.

            It is Hayek’s own interpretation of road to Serfdom that dominates the picture of what it says, explaining why libertarians generally see the argument against Stalin-style socialism and single-payer health care as being fundamentally the same thing.

          • rahien.din says:

            Okay! I will look more into those.

            Thanks for your patience, for proving me wrong, and for providing some guidance. (Especially, despite that my initial response was a pure vent of frustration.)

          • quanta413 says:

            @1soru1

            Good point on Hayek.

            I think his original writing was closer to the mark before hindsight. Charles Murray supports a universal basic income as do some left-libertarians. A lot of European states with much stronger welfare states than the U.S. do better on Heritage’s economic freedom index than the U.S. , and heritage is pretty conservative.

            @rahien.din

            Thank you, that is an extremely high compliment. One of the nicest ones I’ve gotten. Especially since I wasn’t very gracious. I’d recommend The Road to Serfdom most, but it may be kind of boring in some parts because the mainstream of U.S. political thought now is pretty much against nationalizing industries. But Hayek was in a context where a lot of governments still wanted to do more of that. I really like Bryan Caplan’s “Selfish Reasons to Have More Children”. Not a book on libertarianism, but I think it gives a good idea of how Caplan thinks and what he thinks people should maximize using their freedom and how they should go about it. Scott Sumner’s blogs are mostly on monetary policy and over my head, but I can understand his other stuff.

            I you read any of the Machinery of Freedom (David Friedman) e-mail me about it. I’m interested in your impression. I’ve only read chapters from Friedman’s draft on other legal systems and his blog posts. Haven’t yet had the desire to put Machinery of Freedom on the stack of things to read soon.

            And on a related note, I’ll get to reading The Case against Education. I’ve just been a bit delayed.

    • LadyJane says:

      Scott isn’t a libertarian, at most he’s vaguely libertarian-leaning. He wrote an Anti-Libertarian FAQ and still endorses many of the points he made in it. And even if he was a libertarian, why would that matter? This post is clearly not meant to promote libertarianism as an ideology, it’s meant to criticize some really insane and horrible strawman arguments against libertarianism.

    • gbdub says:

      Oh come on. Robinson constructed arguments that rely on a series of implausible strawmen.

      Scott merely breathed life into those strawmen, let them populate a world, and observed their behavior, so that the rest of us might see how meager their stuffing really is.

    • Donnie Clapp says:

      I’m genuinely sorry to stop and single you out, but I am a bit lost. When did libertarianism come to mean “not preventing coercion”? Or maybe to put it less accusingly, why do you think it means that, are there people who say it means that, and where can I find them to go argue against them?

      I thought libertarianism basically boiled down to the idea that people should be able to do whatever they want with what they have, as long as they’re not infringing on anyone else’s ability to do the same. That’s a pretty big “as long as” and, at least in my admittedly surface-level understanding, pretty much answers the sort of question you’re asking.

      Libertarianism doesn’t include “No coercion” in its list of commandments, but it does include “You can’t hurt someone else unless they want to be hurt.” Doesn’t that naturally cover all the sorts of coercion we’re worried about here?

      • rahien.din says:

        “No one has infringed upon my ability to do what I want” is a statement of the necessary and sufficient conditions for “I have not been coerced.”

        • quanta413 says:

          That is not true. Your condition is sufficient but not necessary. Coercion involves the use of threats or force. Coercion necessarily infringes on someone else. But infringing on someone else is either breaking the terms of an agreement with them or limiting their rights in some way. You can infringe without using coercion. Just walk away from a contract you signed.

          Or limit their rights by acting in a totally legal, nonviolent, and nonthreatening manner but always with an eye to frustrating their goals; that’s pretty much the hypothetical infinitely rich psychopath with a weird hard-on for not just speeding up the whole process by hiring a hitman or some CIA interrogators.

          • rahien.din says:

            Maybe I should walk it back and say “No one has deliberately infringed upon my ability to do what I want.” But I consider that sufficient for coercion.

            Here’s the deal :

            Coercion involves the use of threats or force.

            This is a genuine question. That question is the exact thrust of the first and last stories. Can the exercise of rights be a tool of coercion? Can you coerce via a conditional gift too large or dear to be rejected?

          • quanta413 says:

            Here’s the deal :

            Coercion involves the use of threats or force.

            This is a genuine question. That question is the exact thrust of the first and last stories. Can the exercise of rights be a tool of coercion? Can you coerce via a conditional gift too large or dear to be rejected?

            I am not sure what you mean by this. The very definition of coercion both dictionary and legal means something involving threats and force. Lots of bad things don’t involve coercion. Why stretch a category to include lot of other things and make it less clear? We already have the word “infringe” for describing different situations. Why lessen the power of our language by making it overlap more with coercion?

            I think the answer to the first question is yes. You can exercise your rights in order to engage in coercion. If someone tries to rob me, and I stop them by pulling a gun on them and telling them to back off, I have coerced them not to rob me. And I have done so by threatening to exercise my right to self defense.

            I think the answer to the second question is no, and to answer yes would in most cases needlessly redefine coercion. Offering someone an option strictly better than any option they have completely without me is not coercion. But the counterfactual has to be completely without me. If I first maneuver them into that place myself personally, it may or may not be coercion depending on whether or not I used force or threats to get there. It’s possible to get into weird areas on the boundary of the definition through extremely judicious use of things bordering on force but not quite force, but I don’t think this is particularly interesting. I could just infringe on them without any force or threats at all then offer the strictly better option. This would not be coercion, but it might be a form of aggression or it might not be depending on the remaining details of the hypothetical.

            I would say the relevant question is more about what do we think about things that are unpleasant or wrong but aren’t coercion. More broadly than the hypothetical of bizarre sticklers for hurting you without threats or force, how much legal or moral duty does a person X have to others to mitigate things that aren’t person X’s fault?

          • rahien.din says:

            I get that the broader concept of “infringement” may be easier to apply here, and philosophically that’s kind of what politics is about, anyway. There is real value in that. But I hesitate, because the concept seems so broad that it applies to all kinds of problems we don’t really even care about solving, all the involuntary and minor infringements that are simply a fact of life. If I am not paying attention and I unknowingly cut you off in the line for the grocery store checkout, that’s an infringement. It’s not coercion. Nor does it even matter. We can start from “infringement” but we still have to decide which infringements the political philosophy will attempt to address. Maybe A. the coercive ones, and B. the major ones?

            We should just use the words for those things.

            As to “bizarre sticklers” and whether they are even worth addressing, maybe I can give you an example of how this affects my daily life.

            As a medical student I was taught to never comment on how nice a patient’s belongings are (especially if that person is from another country) because some patients will think that you are asking for them to give you that belonging. And the importance of the physician-patient relationship means that they can not refuse.

            This is not voluntary on the physician’s part – who would demand their patient’s belongings? That doesn’t happen even in cultures where this belief is prevalent. Nor does the comment exactly constitute a threat or force, because the patient neither sees this “gift” as a payment nor as a requirement for continued care. But, the interaction is indeed coercive.

            I am not trying to say that libertarianism must address that exact situation, and I am not providing it as some kind of counterexample to the philosophy. I am trying to demonstrate the type of coercion I will have to account for if I am to become more of a libertarian.

            The other thing is, if all these “bizarre sticklers” are truly just marginal cases, we have to ask ourselves why that is so. Are they marginal cases despite our current system, and would remain so under libertarianism? Or are they marginal cases because of our current system, and would become more heavily exploited under libertarianism?

          • quanta413 says:

            We can start from “infringement” but we still have to decide which infringements the political philosophy will attempt to address. Maybe A. the coercive ones, and B. the major ones?

            I agree. This gets into issues of proportionality of allowable response whether the favored solution is government, weird anarcho-capitalist protection companies, or self defense. It gets pretty complicated pretty fast.

            I think the fundamental problems faced here are similar for most philosophies. What I find weird about the current affairs article is it skips past what I think is usually the most difficult thing for libertarians to work out! Solving coordination problems. In the hypotheticals, libertarians have apparently solved this better than anyone yet! This does not jive with my impression that David Friedman wrote a book arguing and describing how it would be possible to coordinate certain government functions privately. To get to a libertarian society, an enormous amount of current institutions would need to be reformed, replaced, or abolished. It would take a lot of time and a titanic effort to do this without doing a lot of damage.

            As a medical student I was taught to never comment on how nice a patient’s belongings are (especially if that person is from another country) because some patients will think that you are asking for them to give you that belonging. And the importance of the physician-patient relationship means that they can not refuse.

            This is not voluntary on the physician’s part – who would demand their patient’s belongings? That doesn’t happen even in cultures where this belief is prevalent. Nor does the comment exactly constitute a threat or force, because the patient neither sees this “gift” as a payment nor as a requirement for continued care. But, the interaction is indeed coercive.

            This is a very interesting example because the element of coerciveness comes through misunderstanding and without intention. It also might partly come from an understanding that doctors have a moral obligation to treat people. I’m still chewing on it, and I don’t have any new thoughts about it.

            Mostly unrelated, I think I have an even weirder case where you might be obligated to coerce or murder someone who is infringing on your rights even though they have no negative intentions and no way of knowing what the hell is going on. Next OT maybe.

            The other thing is, if all these “bizarre sticklers” are truly just marginal cases, we have to ask ourselves why that is so. Are they marginal cases despite our current system, and would remain so under libertarianism? Or are they marginal cases because of our current system, and would become more heavily exploited under libertarianism?

            While this is somewhat of a worry, my personal impression is that it’s usually simpler and easier to just screw someone over in a more blatantly immoral manner.

  20. “Yeah, well, about that. I know this guy named David Friedman, whose hobby is designing weird insurance systems for anarcho-capitalist utopias based on, like, the laws of medieval Iceland or something. Anyway, he sells ‘rich person gets a weird grudge against you’ insurance. I have loads of it. However much you try to bribe my friends not to talk to me, his company will pay more to bribe them to ignore you.

    That’s a game that will be one by whoever has the most money, which , in the original hypothesis, is the Infinitely rich man. So basically you are fighting the hypothetical. Mr Nett -Worth loses because he isn’t wealthy enough to win, not because winning is impossible however wealthy you are.

    “Oh, but huge disparities of wealth don’t occur!”

    Don’t occur where — in our word, with it’s progressive taxation, and anti-trust laws?

    • Robert Liguori says:

      Yeah.

      But I don’t think there is a political system of any kind which isn’t vulnerable to 51% (or rather, crucial takeover of deciding apparatus) attacks. And, if anything, libertarian hypercapitalist soceities are a lot harder to execute these kind of total attacks in. Getting a majority of people to vote against a minority is done, a lot. Taking over a Communist committee and allocating the people’s resources away from those hateful bastards who don’t deserve it is done, a lot.

      But it’s really, really hard to have one person have enough wealth to personally counter a widespread, bought-into insurance policy.

      Now, engaging in a non-total campaign of fuckery using superior resources is also done, a lot. But cutting the examples down to the level of the real and plausible is, for most people who put out articles like the one Scott is critiquing, not even remotely the point.

      • themountaingoat says:

        Right now there are people that have enough wealth they could easily out purchase the entirety of the people in certain countries, and that is with redistribution and other laws in effect. I don’t see why you think someone having that much purchasing power is hard.

        • Edward Scizorhands says:

          Because not everyone else in the world is a dainty flower who simply rolls over for the rich man.

          If Bill Gates started buying up Mauritania, there are people whose jobs it is to notice that and start buying it up ahead of him.

          Walt Disney had to do a whole lot of work to buy up useless swampland in Florida without anyone realizing what he was doing, and that was before automated computerized records, and he still couldn’t pull it off.

          • themountaingoat says:

            If someone manages to own all the land surrounding wherever you are currently living it under libertarianism they have the power of life or death over you. I can easily see someone managing to own the property surrounding someone else’s residence. They could even own the properties through shell corporations so the person wouldn’t know until it was too late.

          • Edward Scizorhands says:

            They could even own the properties through shell corporations so the person wouldn’t know until it was too late.

            This is what Walt tried, and even then it didn’t work.

          • Donnie Clapp says:

            “If someone manages to own all the land surrounding wherever you are currently living it under libertarianism they have the power of life or death over you.” —@themountaingoat

            I’m sort of new here. Are there a lot of libertarians arguing against access easements? I would consider them a perfect example of the proper role of government in a libertarian society: making sure nobody can prevent anyone else from enjoying their own property.

          • sharper13 says:

            Are there a lot of libertarians arguing against access easements?

            No, if you asked an actual libertarian, they’d be forced to point out that no one could sell property for much of anything without it coming with access rights along with it, because no one would want property without the ability to access it.

            But they were talking about strawman libertarians, who have never heard of contracts and contract rights, as opposed to the real ones who base a good chunk of their actual ideas on them. See also private roads and property with road rights, which no libertarian has ever been asked about before.

      • Deiseach says:

        But it’s really, really hard to have one person have enough wealth to personally counter a widespread, bought-into insurance policy.

        If the premiums only cost mere pennies per year? How much coverage can they realistically expect? Will the insurance company have an upper cap on the policy (“we’ll cover up to ten million dollars worth of vengeance, but if Mr Moneybags is willing to spend twenty million on screwing you over, tough luck!”) as they do with medical insurance and expenses, or refusing to insure people who live in floodplains or other areas that regularly will get hit by hurricanes and flooding? How about if Mr Moneybags instead decides to buy the insurance company and close it down?

        If we’re talking about Really, Really, Maybe Doesn’t Own All The Money In The World But Damn Near Rich Guy, how high would an insurance company be willing to go? Jeff Bezos was (briefly) the World’s Richest Man with an estimated fortune of $90 billion dollars. If he decided he would devote 99% of that to crushing you into the dust, would an insurance company be there to match him dollar for dollar?

        Looking it up, AXA insurance company had $147.5 billion “ordered by annual sales for the latest Fiscal Year that ended March 31, 2017” so they could outspend Mr Bezos, but would they be willing to devote that much (nearly two-thirds of their annual revenue) to one customer?

        • Thegnskald says:

          The whole situation is kind of puzzling to me.

          In the real world, you could get away with punishing one person, once.

          If you keep targeting someone for perpetual vengeance…

          …well, don’t be surprised if you get a bullet in the morning while walking your dog.

          In the real world, civilization is built in large part on making sure people have something to lose.

        • John Schilling says:

          so they could outspend Mr Bezos, but would they be willing to devote that much (nearly two-thirds of their annual revenue) to one customer?

          Hmm, how many customers would they get next year by being known as the company that took down Jeff Bezos in defense of the little guy?

          But Jeff Bezos didn’t get to be a hectobillionaire by spending billions of dollars on wasted efforts. So long as the insurance company credibly commits to outspending Bezos (say, with contract penalties that will drive them into bankruptcy if they don’t), then the actual result will probably be a negotiated settlement where the insurance company throws a big “Jeff Bezos is awesome and is totally right about horses” party in exchange for his not paying outsiders to join in his grudge.

        • Tracy W says:

          Insurance companies spread their risks by re-insuring for a percentage of their liabilities for the policies they write.

          So it wouldn’t be just one insurance company’s resources Jeff Bezos was taking on, but a large swathe of the world’s total financial markets.

          Although perhaps a simpler option would be to start setting up castanet factories to hire the insured guy. When the Infinitely Rich Guy goes to buy one, you charge him 20% above your costs. You then repeat. (Or call your mates: “Hey, rich idiot here! Here’s how to take him for a fortune!”) You could make money on this contract.

        • Tracy W says:

          On thinking about it some more; this insurance could actually be profitable. Rich Guy will bribe any employer of your client to fire your client? Set up Alice’s Insurance Co.’s subsidiary, hire your Client as the CEO of your subsidiary, and sole employee. When Mr Thaddeus Nett-Worth III, Esq comes along with his bribe, accept it, put the money in the bank, wind up Alice’s Co and set up Bob’s Co.

          If we assume that Mr Nett-Worth manages to get your company sign a contract whereby you can’t just keep setting up subsidiaries so as to accept bribes for employing your client, well, call up one of your suppliers’ sales staff, take them out drinking and negotiate a discount on your next purchase in return for the knowledge that if they hire your client a Mr Nett-Worth will give them a big payout.

          Buy another lakeside property for your client. Also buy the surrounding properties. Triple the prices when Mr Nett-Worth comes shopping.

          Deputise a bunch of interns to be friends with your client, normal hourly rate, you keep the bribes to stop.

          Continue in this style until Mr Nett-Worth can’t afford this any more. Collect your bonus for the insurance company’s most profitable year ever.

      • rahien.din says:

        But it’s really, really hard to have one person have enough wealth to personally counter a widespread, bought-into insurance policy.

        How wealthy do you think the owner of that insurance company would be? What do you think they would be capable of?

        Also, Scott’s counterstory is implausible because Mr. Nett-Worth is rich-and-clever enough to convert the entire world to electric cars, but not rich-and-clever enough to bribe an office worker to “accidentally” shred a particular insurance policy…

        • Tracy W says:

          You’re fighting the hypothetical. It specifies that Mr Nett-Worth is a strict libertarian who is acting according to libertarian principles.

    • That’s a game that will be one by whoever has the most money, which , in the original hypothesis, is the Infinitely rich man.

      He isn’t supposed to be literally infinitely wealthy, just very wealthy. The insurance approach in effect combines the wealth of everyone else to defend against him.

      Currently, the richest American (possibly Jeff Bezos) has a wealth of a bit under a hundred billion. The U.S. national income is about nineteen trillion dollars.

  21. Slashdot Poster “Moraelin” on Anarchism

    If only it were that simple. I think if you look at history without the selective glasses of how it fits Ayn Rand’s _novels_, you’ll find that there was a subtly different common thing there: whenever you lost your rights and liberties, it was actually simply because rich and powerful fucks wanted more riches and power… and nobody could stop them. Yes, taking over the government is a popular way to do it, but even there the trend isn’t necessarily “giving the government too much powers” but equally often “the government wasn’t strong enough to stop them.” You can see such examples as: – the 1075-1990 civil war in Lebanon: rich fucks and political parties simply hired their private armies and started a devastating civil war, making everyone’s lives miserable in the process. Yeah, I’m so sure that in the ensuing war where extortion, theft, robberies and collecting “customs” at random checkpoints are everyday occurences was soo much better than a government you can control through democratic means. Newsflash: once those warlords managed to take over a region by just having a larger militia than the government’s army can do anything about, they _were_ the de-facto local governments, and they didn’t give a flying fuck about your thinking they should have less rights. – the chaos in Somalia: ditto. Nobody was in a position to stop the warlords’ private armies from plunging the country into chaos – various coups where an impotent government just couldn’t do more than watch a bunch of mercenaries or CIA/KGB agents shoot their way into the palace and have it their way. Yeah, I’m sure the people now live so much better because they didn’t give the democratically-elected government even enough power to defend itself. – heck, even the russian revolution: communism didn’t happen because of someone giving too much power to the Tsar’s government (though it already had too much,) but because said government couldn’t stop Lenin – the fall of the Roman Republic: it didn’t happen because some erosion of rights by the Senate, but because Caesar marched with the legions on Rome and nobody could stop him. Then he got all those titles and rights at sword point, as a conqueror. And, oh, the third century crisis? Yeah, it was soooo great that the government couldn’t even elect an emperor after the praetorian guard murdered the previous one. I’m sooo sure that after a century of internal warfare and millions dead, they were still congratulating themselves for having an impotent Senate 😉 – some corporation simply getting powerful enough to do what it damn pleases, because nobody has the guts any more to say “no” to the largest employer and land owner in the country. Do a bit of googling on “United Fruit Company” and how the term “banana republic” came to be. Etc. It seems to me that past a point such cases can _only_ be stopped by a strong enough government. Yes, an uncontrolled government is dangerous too. Which is why you have to control it. If you still (think you) are a democracy and don’t want it snooping on you, fine, organize it so it can’t without a mandate. But “OMG, governments are evil, keep them away” attitudes are just bloody stupid. That’s how worse problems come to you further down the line. A government is nothing more than how a large number of people organize to live together in _some_ way, and mostly because anarchies are even worse and power vaccuums just beg to be filled by the biggest bastard. And arguing for a government which _is_ a power vaccuum, at most buys some time until some bastard gets powerful enough to replace it with himself.

  22. Peter Gerdes says:

    “But surely, somewhere, there are incredibly stupid libertarians who think morality consists of the Non-Aggression Principle and nothing else, don’t believe in any other kind of virtue, and aren’t just holding it as a sacred but non-final principle the way you hold not torturing people?”

    Whether there are or aren’t such libertarians really doesn’t matter because, as a matter of linguistic meaning, libertarianism refers to a political philosophy and as such any claims about the nature of virtue or how people should other than as a political body simply aren’t part of libertarianism. Its about as relevant to question at hand as the fact that some libertarians believe the moon landings are a hoax.

    • Ermm..no. Quite a lot of libertarians make appeals to deontological principles concerning force and coercion. Objectivists make a virtue of the fact that their political philosophy is derivable from their ethical philosophy.

  23. Peter Gerdes says:

    Ok, yes these are silly criticisms even for deontic libertarians because libertarianism is merely a political philosophy not a complete moral theory. However, chance the examples around so the fact that people aren’t inclined to be exceptionally vindictive works against the deontic libertarian and one can still get some appropriately extreme horrible consequences.

    In particular, instead of considering cases where non-government coercion is used to harm someone simply consider cases where coercion is necessary to help everyone. For instance, consider a scenario in which a deadly plague is spreading and threatening to wipe out everyone on the planet but one individual turns out to have a natural immunity and the medical community is convinced that with a small sample of his bone marrow we could mass produce those antibodies and save 90% of the world’s population from an agonizing death.

    Now, we don’t need everyone to be a miser or selfish. Maybe this man has heard that the disease kills 99/100 men but only 80/100 women and likes the idea of living in the post apocalyptic world. Worse, he might own a stocked doomsday bunker so even in the unlikely event one could coordinate social pressure he could simply ride it out.

    Does this example apply against most libertarians? Well, sorta. Most libertarians tend to vacillate between claiming some kind of inviolable deontic mandate and pointing at practical benefits and examples like this force them to retract their deontic arguments and engage with the same policy facts as everyone else.

    • Jiro says:

      You can’t make a rule “coercion is permitted if there’s one person who is needed to solve a deadly plague”. You’re not a prophet and you don’t know the specifics of the situation in advance. What you have to do is make a rule “coercion is permitted if it’s an extreme situation where we really need to do it”.

      And rules like that will not be used only against people with the cure to plagues.

      • And absolutism about noncoercion means you have to put up with an uncured plague. Do you have a good solution?

      • Toby Bartels says:

        You make the rule that coercion is forbidden, period. And if it ever turns out that you really need to commit coercion anyway in order to stop a deadly plague, then you break the rule and accept the consequences of having done so. (In Common Law, at least, there is such a thing as the necessity defence that can be used here. It must be argued after the fact, and the burden of proof lies with the defendant.)

    • thevoiceofthevoid says:

      Is there not an obvious solution here? Have as large a group of people as will coordinate agree to pay the donor off for his bone marrow. I find it hard to believe he wouldn’t donate for a million dollars, and you could probably get a million people to donate a dollar each to save the world (and their own skins).

  24. JPNunez says:

    In Libertaria, the gay men association pays a protection agency a hefty cost to not be flogged by the various church’s protection agency, who in turn pay to flog gays.

    After some very expensive mini wars, the protection agencies achieve a peace treaty. One gay man will be flogged anually, but he must submit voluntarily to the opposite agency. Obviously, the gay men association decide to pay a compensation to this symbolic victim.

    What is the expected pay for the voluntary flogged gay man?

    • Garrett says:

      The forms of libertarianism I’m familiar with still have a government who’s primary purpose is to protect people from acts of force committed by others. So a church protection agency who was actively going out and flogging gays would be arrested under some form of assault/battery/conspiracy charges and sent to jail.

      This might be a better criticism of forms of anarcho-capitalism. However, I’m not familiar with any broad movement among the US church organizations to specifically be violent towards gays. At worst, to shun them. No doubt, you can find a few wackos who’d be willing to chip in some money to flog gays under some religious principle. But would there be enough of them, or enough money spent on it in order to be able to overwhelm the gay protection agency?

      And, why would the gay protection agency agree to such terms where someone needs to volunteer to be ceremoniously flogged? And given the strange sets of kind people exhibit, why would you think that they couldn’t find at least one legitimate volunteer every year?

    • Toby Bartels says:

      Far be it from me to suggest that gay men are all into sadomasochism and would be crawling over each other for the opportunity to be the person flogged every year. Indeed, for the vast majority of gay men, this is a ridiculously and insultingly false stereotype.

      But for some of them, it is true! So the expected price is negative.

  25. Doesntliketocomment says:

    The only part of this post was the punchline, but the punchline was really, really funny.

  26. Murphy says:

    I have mixed feelings re: libertarianism.

    I don’t think it’d be that great, if followed dogmatically, for producing a society I’d actually like to live in but it makes a nice default or leaning. If someone in charge wants to make a rule the onus should be heavily on the rule-maker to justify adding the rule rather than on the people to justify why it shouldn’t be there. There are problems that a libertarian approach kinda sucks at solving in the real world (scott has run through many examples) and there’s quite a few that it solves quite well.

    As a general philosophy it’s kinda nice with a fair bit of consistency that means many of it’s positions lead fairly well from the base premises. Of course there’s some gaping exception to that consistency in some of the more popular forms of libertarianism but what movements don’t have a little of that.

    I have a particular soft spot for the Geolibertarians even thought they’re a tiny subset.

    • Edward Scizorhands says:

      Pretty much no philosophy survives an extreme take-it-all-forever-no-exceptions treatment.

  27. Corey says:

    Probably a FAQ, but this seems like a good place to ask: how does insurance even work in a libertarian system? In currently-existing markets, it needs heavy regulation to be usable and making the markets more libertarian (e.g. having multiple competitive legal systems) could only exacerbate this.

    Moloch will incent insurers to minimize payouts by e.g. burying customers in fine print, and He is relentless. Brand reputation may push back against this but that’s an asset to be used up when times are tough, and Moloch guarantees times will get tough eventually (especially in the absence of regulations on financial stability).

    Is insurance considered an exception and allowed to be regulated? Are there recursive layers of insurance insurance you’d buy to ensure your insurances pay out? Or is there some other solution?

    • Murphy says:

      I think the general idea is that people will move to insurers that have a better record of not screwing over their customers or don’t have that kind of small print.

      Though it may be overly optimistic.

      When times are good an insurance company that’s no nonsense and pays out has a competitive advantage.

      But the moment you start talking about things that might pose an existential threat to the insurance company that kinda goes out the window.

      The optimal for survival might be a company that pays out without much stress most of the time but writes it’s contracts such that it technically has the option of going full bastard and simply doesn’t take that option in normal times.

      For example a company that sells income insurance that pays out dutifully to claimants while the economy is good.

      Anyone who goes with other companies without theoretical, never-enforced, fuck-you clauses during good times suffers a constant disadvantage vs people who don’t and such companies and the companies themselves realistically pricing risk also suffer that constant disadvantage, losing market share and being noncompetitive in the market .

      Then when a really big crash happens and it looks like lots of people will actually claim endangering the company they can actually enforce all that almost-never-enforced fine print and avoid paying out.

      Sure it will hurt their brand but that only matters while the brand itself has a higher cash value than the money to be lost. So they can just take all that money they saved and use a little of it to re-brand. Perhaps buy up one of their less solvent competitors and repeat the same game under their so-very-trustworthy-logo until the next crash.

      After all, evolutionarily optimal strategies don’t have to be nice.

      It wouldn’t work in a market with perfect information where everyone is an actuary who can spot underpriced risk but in a real market?

      • Murphy says:

        Note, my example is based on a friend of mine who worked at an insurance underwriter that did basically that during the 2008 crash. They were all nicey nice before the crash, he was part of the team of staff hired with the jobs of finding every excuse in the book to screw claimants when it looked like people would start claiming in large numbers.

        it’s the reality of business.

        Remember in all things: past good behavior is not predictive of future good behavior when enough money is on the line.

        • Protagoras says:

          What kind of insurance spiked in claims during the crash?

          • Murphy says:

            Income insurance.

            Basically people who are worried about things like suddenly being laid off or their employer folding or serious illness/injury leaving them unable to support their their family or being left unable to pay their mortgage.

            it tends to be expensive insurance.

            There were people who’d been dutifully paying their insurance for 15 years and as soon as the crash happened and it became clear that lots of companies would be folding in the near future the underwriter decided they wanted the policies off their book.

            Crashes don’t happen overnight, it can take months for big layoffs to start.

            So they 1: overnight raised monthly premiums by thousands of percent to avoid any renewals and 2: hired on teams of people with the sole job of invalidating the maximum number of remaining contracts of people who claimed, lots of fine print that hadn’t been enforced much before because highly used.

            In both cases relying heavily on either fine print or missing fine print in the contracts.

            basically fucked lots of hard working honest people in a surprisingly lightly regulated area of insurance.

            The parent company apparently got in hot water in the states around the same time because they did medical insurance and fell afoul of the law when they tried to play a similar game with people with HIV. Though they dragged *that* through the courts for so long that many of the claimants simply died while it was still being appealed and partly led to creation of US government regulations against rescission.

            They got bad press for the HIV thing but the Income insurance thing was apparently dry and boring enough that it never got media attention.

    • Protagoras says:

      I’m not an expert on the history of insurance, but as I understand it, the evidence does not support your pessimism about how much insurance companies are concerned with reputation (in a time of far less regulation, Lloyd’s paid out incredible sums after the San Francisco earthquake). I think the place you’re most likely to see insurance which is designed to weasel out of paying out are cases which are themselves generated by regulation; cases where the law requires that insurance be purchased by people who don’t really want it, and so where such people buy absolutely the cheapiest, crappiest insurance that complies with the law rather than looking for insurance that will really protect them (because they don’t think they need the protection).

      • 1soru1 says:

        > Lloyd’s paid out incredible sums after the San Francisco earthquake

        If you look into the details of Lloyd’s of London, it is a real stretch to call it an example of a for-profit insurance firm; it’s either a weird anachronism or an intrusion from another timeline where everything works that way.

        I hear that from some of the windows in their head office, you can sometimes see zeppelins.

    • John Schilling says:

      Is insurance considered an exception and allowed to be regulated?

      In almost all forms of libertarian politics or philosophy that anyone actually advocates, everything is allowed to be regulated at least to the extent of enforcing contracts and prohibiting fraud. As long as you’re not going out of your way to make things easy on crooked insurers, that’s really all you need. Others have already noted the long history of reliable insurance providers in ages of capitalism far less regulated than our own.

      In an anarcho-capitalist society, which only a small minority of libertarians favor, you might need an insurance policy against being defrauded by your insurance company, which isn’t an entirely ridiculous thing but the possibility of the whole system collapsing into a cartel of insurance/security providers as de facto overlords is part of the reason most libertarians don’t favor that system.

      • Murphy says:

        But more than just contract enforcement is important when it comes to insurance companies.

        if the underwriter doesn’t actually have enough assets to cover their liabilities you may not discover that until you try to enforce the contract. At which point it’s too late and you and everyone else who bought from them is SOL.

        • John Schilling says:

          Selling insurance without the necessary assets (or bonds or reinsurances) to cover plausible liabilities would almost certainly constitute fraud.

          And in this case a fraud that wouldn’t be terribly hard to detect, so I’m willing to trust that between Consumer Reports, the advertising agencies of rival insurance companies, and random nerds on the internet, we’ll have a pretty good idea of who is trying to sell unbacked insurance.

          • Murphy says:

            Note, not talking about libertarian-land:

            real world insurance underwriters seem to reasonably regularly run into the problem of not actually having enough assets to cover their liabilities, particularly when unregulated, poorly regulated or weakly regulated.

            It’s hard to know for certain how deep the pockets of underwriters really are in a society with strong privacy norms never mind whether they’ve taken on other liabilities and it’s a dry enough subject that the normal consumer newspapers read by the kind of people who would be buying the insurance rarely cover anything except the final fallout.

            Random nerds on the internet don’t even get a look in.

          • John Schilling says:

            It would almost certainly be possible to hide your net worth very effectively in a libertarian society, but it isn’t necessary or inevitable. And, absent deliberate fraud of a sort that I think wouldn’t be to difficult to ferret out, it should also be possible to verifiably reveal a firm’s assets and liabilities.

            I’d like to think that when it comes to selling insurance, the reputational advantage from opening your books would manifest as an advantage in the market.

          • Edward Scizorhands says:

            I have first-hand observations of insurance companies telling their employees “it doesn’t matter if the claim should be covered, just deny the claim and see if they refile.”

            The claim was eventually covered, but I can only imagine how much stress the person was already under with their spouse dying and they didn’t need that shit in their life.

  28. mobile says:

    So electric energy flourishes and gas-powered transportation becomes obsolete because a guy had an argument about a horse? Libertarianism sounds pretty awesome.

  29. carvenvisage says:

    But how do you know these aren’t just thought experiments? Like, you’re saying this is some kind of crypto commie bamboozling, but I don’t even see the author’s name listed, so how do we know he’s a commie?

    ..Wait, nvm, I read to the bottom of the article. My bad, sry.

    _

    Btw, isn’t is good to have your wife’s life saved when she would have died otherwise? Like, if she’s dead you don’t get to speak to her again either.

  30. DocKaon says:

    Anti-vengeful rich person insurance would never work.

    A person’s risk of pissing off a rich person is an idiosyncratic risk which is known the purchaser. A person who doesn’t interact with rich people, isn’t argumentative, or a likely target for sexual harassment doesn’t need the insurance and won’t buy it. The people who do need people like journalists, aspiring actresses in Hollywood, service workers in businesses that cater to the wealthy, individuals with strong unpopular political views, etc… know they’re at risk and would buy it. A risk spiral that would make it unaffordable to basically everyone who would benefit from buying it.

    As a purchaser how do you prove that your misfortune was the result of animus from a billionaire rather than something else? It’s not like your employer is going to tell you that you got fired because the billionaire owner of his largest client has a vendetta against you. The insurance company is going to be rightfully concerned about fraud and moral hazard.

    • Murphy says:

      it’s definitely easily gamed.

      If your friend is already rich and you have Anti-vengeful rich person insurance what’s to stop you from having a “fight”, him offering $1,000,000 to your other mutual friend Bob to stay away so that the insurance company is required to pay $1,000,001 to Bob to stay.

      Free money for Bob.

  31. “You’re…McDonalds employees? Why are McDonalds employees doing no-knock raids in body armor looking for marijuana?”

    “I…I don’t know.”

    “You’re G4Ss? Why are G4S employees doing no-knock raids in body armor looking for marijuana?”

    “Why the surpise, we already do a ton of other security work”.

    • Deiseach says:

      “You’re…McDonalds employees? Why are McDonalds employees doing no-knock raids in body armor looking for marijuana?”

      “The corporation is diversifying into private prisons, duh!”

  32. Lasagna says:

    Thank God for you, Scott. This was wonderful.

    I followed Current Affairs since it started. Nobody would ever accuse me of being a socialist, but Nathan Robinson is a good writer, and there were lots of interesting and thought provoking articles.

    But it just went rapidly downhill. Then finally they published the single most noxious article I’ve ever read: “Bring on the Sensitivity Readers”, and that was that. I gave up. Oh well.

    • DM says:

      ‘. Then finally they published the single most noxious article I’ve ever read: “Bring on the Sensitivity Readers”, and that was that’

      Really, the literal worst thing you’ve ever read was an article defending the use of what’s effectively market research by publishing houses who want to avoid bad publicity? I mean, not that I’m in favour of ‘sensitivity readers’-mostly because I think anyone who takes such a job will be more sensitive than 90% of the people they’re claiming to represent-but really? How can an article advocating that publishing houses choose what the publish partly to avoid backlash by idiots, possibly be worse than things that advocate policies that actually kill people or are transparently bigoted? I mean, didn’t you ever read an article in support of the Iraq war? In support of an economic policy you think is massively wrong-headed and costing billions? If your really reacting that strongly, your level of distaste for ‘social justice’ stuff is way out of whack with reality.

      • moscanarius says:

        Congratulations, this may be one of the most over-the-top reactions to an over-the-top comment that I’ve read in this site. If you keep up the good work we can turn into old days Slate magazine in no time!

  33. moscanarius says:

    I liked the parody, thanks for writing it!

    It’s funny how most of the criticism can be best described as a demand for perfection. Oh, [your ideology] can’t deal with [hypothetical-bad-cannibal situation that nobody can really deal with]? THEN IT IS WORTHLESS AND YOU ARE IMMORAL. Some people accept nothing short of perfection before even considering your contribution. Your contribution, mind you: ideas of their liking get a pass.

    I doubt there is any ideology out there that is perfectly consistent, justly applicable to every situation everywhere on Earth, and completely immune to irrational behaviour and human ill will. Showing an inconsistency or a loophole should not make anyone discard whatever political/moral philosophy, as loopholes can be found in all of them. The question is how big, how damning, how numerous, how difficult to compensate, and how awful in practical terms those loopholes will be – preferentially discussed with some empirical data.

    This sort of hypotheticals are not useless or evil in themselves. It’s just that some people tend to extrapolate from “can’t deal with hypothetical, is not fully consistent” to “can’t deal with anything remotely similar to hypothetical, makes no sense at all and is evil”.

    • Deiseach says:

      Oh, [your ideology] can’t deal with [hypothetical-bad-cannibal situation that nobody can really deal with]? THEN IT IS WORTHLESS AND YOU ARE IMMORAL.

      If you’re a religious believer, you get this exact kind of WHAT ABOUT EXTREME CASE? HA, GOTCHA! YOU’RE EITHER A HYPOCRITE (IF YOU BREAK YOUR RULE TO AVOID THE BAD CONSEQUENCE) OR AN IDIOT (IF YOU KEEP THE RULE AND THE BAD CONSEQUENCE HAPPENS)! thinking all the time. It’s the same line as “so if you’re anti-abortion, surely that means you want to charge women who get abortions with murder and put them in prison? Ha, gotcha!” which an awful lot of otherwise sensible people seem to think is some kind of stunning put-down that leaves the pro-life side sprawled in the dust speechless and defeated.

      • Jiro says:

        If you’re a religious believer, you get this exact kind of WHAT ABOUT EXTREME CASE? HA, GOTCHA!…thinking all the time.

        Religions (at least the kind that we’re talking about here) claim to be sources of absolute truth that come from an omniscient being, so gotchas are fair..

        • Lasagna says:

          Ladies and Gentlemen, Exhibit A. 🙂

          Don’t mean to give you a hard time. But you’re basically doing what Deiseach complained about. Most religions, including the ones we’re talking about here, don’t claim to be sources of absolute truth. Christianity, in particular, is very much aware of the failability of man. It’s kind of a central tenet.

          The different sects of Islam, Christianity, Judaism, you name it, all regularly engage in nuanced discussion and disagreement about important theological issues, both within the sect and with other sects and religions. It’s rare to come across someone claiming they’ve figured it all out, no more discussion permitted.

          • There’s a Muslim tradition that a mujtahid (expert on religious law) who gets the right answer to a legal question gets two rewards in heaven, one who gets the wrong answer gets one reward. One interpretation of the terminology is that Sharia is law as it exists in the mind of God, enforced by God post mortem, fiqh law is the imperfect human attempt to figure out the implications of Sharia, which is why there are four Sunni schools of law with differing interpretations, all mutually orthodox.

        • Conrad Honcho says:

          There’s an easy out though. Something about now knowing only in part but one day knowing in full…

  34. Eponymous says:

    Wow, that article is something else. I mean, there are plenty of reasonable objections to strong forms of Libertarianism; resorting to tortured thought experiments is inexplicable.

  35. Deiseach says:

    He may leave the village, but it is certain death, for thousands of miles of desolate wolf-infested wilderness stand between him and other humans and he has no food. (The wilderness is also privately-owned, and he cannot pay the admission fee.)

    I’m not exactly sure how this is meant to work? If the wilderness is privately owned, then it’s not precisely a wilderness if you have to pay an admission fee to get into it (and does this mean if a starving wolf in the winter time ventures down to the village and eats some sheep/a villager, the owner of that patch of wilderness is liable to pay for the damage caused?) and secondly, if my village are going to eat me, I’m blessed if I’m going to be such a good libertarian that trespass in order to save my life is unthinkable – I’m crossing your privately-owned wilderness and not paying the fee, so take that!

    Also, on a wider scale, if there are no prisons, how then are things like “you didn’t pay the entrance fee” enforced? By fines? Yeah, go ahead and fine me, I’m not going to pay, now what? You won’t put me in prison because nobody has the right to use force to confine me against my will, so what next – you increase the fine? I keep not paying it. You seize my goods to pay it? Okay, and how do you have a right to use force to take my property away from me, but you can’t have jails? Oh no, I get a reputation as a person who breaks contracts, how awful, that means nobody will do business with me – yeah, and you people are going to eat me or shun me to such an extent nobody will do business with me anyway. I don’t see how I’m being punished here?

    • AnarchyDice says:

      Reminds me a lot of the Dr. Who episodes with Clara Oswald, once you’ve threatened someone with death, you’ve already hit your maximum threat so you’d better be prepared to actually back it up.

    • Garrett says:

      Libertarianism does allow for the government to react to crimes by using appropriate force. Criminal trespass might be enforceable through a prison sentence as it is right now. Enforcement of contracts might also be done pursuant to law as well. Generally, it doesn’t ban the use of force, it bans the initiation of force.

      I do agree that making someone an outcast without other options does create hazards, however.

    • Murphy says:

      how then are things like “you didn’t pay the entrance fee” enforced?

      Easy: the punishment for all crimes is death by cannibalism.

      it doesn’t count as aggression because they’re already violating your property rights which is itself aggression.

  36. AnarchyDice says:

    I had a curious thought about how everyone else in IRM world would react. That world has just seen the rich man start buying up all property and utilities and castanet factories and friends nearby their hated target. Wouldn’t any savvy marketplace start filling that area with new castanet factories or things like trying to get the hated target a new house somewhere else so they can force IRM to buy all the property around it too? Would the hated target be swarmed with gold-diggers trying to become “friends” with the hated enemy so that they too can get these 1M salaries by renouncing their former friend?

    How would you game the IRM if you knew about his plan?

    • poignardazur says:

      THANK YOU.

      Yeah, I’d thought about gaming the “friends paid off to leave” part, but this whole scenario is pretty gameable. Which is in part the point of libertarianism: if people try to throw money to game the market, there’s no shortage of people willing to make money getting it back to normal. No insurance needed.

  37. Garrett says:

    Why is it that Natural Law assumptions must be specious? Alternatively, what separates a specious from a non-specious Natural Law assumption?

  38. John Schilling says:

    Fortunately, libertarians oppose gun control. Thus the local dire wolf population was all but annihilated in an almost orgasmic crescendo of AR-15 fire within a day of that unfortunate species’ conscription into service in ridiculous thought experiments. It is rumored that C. Montgomery Burns maintains a viable breeding population on his estate, but nobody ever visits him so who knows.

    Also, it is well known that once libertarians cut away the thicket of government regulation, human creativity will flourish unbounded and lead to heights of both artistic and technologic achievement almost unimagined by lesser societies. Hence, it won’t matter that there aren’t any roads, because, well, you know.

  39. Error says:

    I think getting the final joke here should be used as a litmus test for political nerdery.

    Also, this whole thing is awesome.

  40. Hello fellow statists. I figure I’d take some time during my Sunday afternoon to bless you with some of my turds of wisdom and experience with a topic I frequently see here and on libertarian discussion boards.

    Some Background: Several years ago before going to the production side of the industry (logging, mills, etc.), I was a forestry intern for a major timber company that owns millions of acres across the world. This was not a “make coffee for your boss” deal. I was actively managing land and spending time in the field every day, interacting with government officials, and essentially being groomed for a full time position before deciding to go to a different outfit. I learned a great deal about land use.

    My main job was generally silviculture (managing contracts for roughly half a million acres of tree farm) but I spent a fair amount of time in our engineering department and land use departments. These are the guys that set up timber sales, build roads, and handle property access issues.

    The “who will build the roads?” question has become an inside joke for libertarians. So much so that they don’t even debate or discuss it anymore. They just regurgitate “privatize everything!” without any thought to the implications or how this gets done in the real world. From my 2-3 years on Reddit, I’ve also noticed most of them don’t even understand the basics of property management issues or that private roads even exist. So here’s my breakdown…

    Complex private roads networks already exist in the US: My state has thousands and thousands of miles of privately built and owned logging roads for secondary transport of wood fiber from the landing to their destination. There are even mills located at the end of logging roads where you will never hit a public highway.

    They are barely regulated: I live in a state with some of the strictest harvest practices in the country. Outside of fish passage and some rules on unstable slopes, there are no limits on truck weights or how your road can be built. A lot of the mainlines are built on top of old railroad beds (rail logging was big in my state before the 40’s) where they actually punched through large hills and over streams to flatten the land. Down in the southern US, it’s even less burdensome. You can basically point a bulldozer where ever you want and make one.

    Easements, easements, easements[*]: For all the talk about privatization of roads, it’s unbelievable how many libertarians I’ve seen online and in real life who don’t even know what easements are. If you’ve ever shared a driveway with neighbors in a rural area, you might know what I’m talking about.

    For tree farms, it’s on a huge scale. Roads go through multiple ownerships and parcels. Legal contracts have to be written up between owners for access. Payment can be anything. Commonly it can be a one time fee to access a timber sale, agreeing to keep the road in good condition, paying per truck, paying per thousand board feet, the possibilities go on and on.

    Even for the purposes of logging, easements are a huge mess. Entire departments exist just to handle road access issues. I’ve gone over hundreds of easements and while most are simple, others are not. My state is a checkerboard of private, state, and federal land. All of which require easements. This reduces redundancy of roads which means more space to farm trees.

    They’re expensive: Holy shit are they expensive. It’s a huge burden, even for a multinational corporation and shortcuts will always be taken. I drove on some pretty sketchy cliffs to get to certain harvest units. I’ve never heard a libertarian explain how the cost would be managed in their ideal world. Remember, no large corporations exist without government in Libertopia (haha, I know).

    Keep in mind, these aren’t even paved roads. They’re just gravel with barely enough room to squeeze a log truck through.

    Gates, gates, gates: I can’t tell you how many hours I spent unlocking, locking, and going through multiple and often redundant gates between property owners. The thought of highway-level traffic volumes going through these roads is laughable. Don’t even get me on started on key management.

    Security: Yup, just like public roads, we have to pay for security to keep riff raff out. Another huge cost.

    Public access: There is almost zero incentive to allow these roads to be open to the public. With my master keys, I could easily maneuver through multiple counties without ever hitting a public road, but the rest of the public – nope. The liability and security issues are too much. Occasionally some would be opened up for hunting season, but that is slowly going away.

    So to wrap this up, the next time somebody tells you that private roads have never been given a chance, they’re full of shit or just ignorant. This industry has used them for over a century in the western US along with private railroads and secondary transport in private waterways. They serve their purpose for one reason only – to get logs off the landing and to a mill. The only other industry I can think of that would have large road systems would be mining and I’d wager they deal with the same issues we do.

    To think private business could handle the requirements of all of society’s transportation needs is absolutely ridiculous, especially since they don’t have the ability to use eminent domain.

    At some point, I’d also like to discuss other issues from my business point of view that relate to libertarianism. Especially the topics of public lands, the history of environmental law, and labor.

    Anybody who knows the history of the timber industry knows that a lot of libertarian ideas have already been tried in the western US and were a disaster. For people, for the environment, and for markets. Private towns, company stores, “homesteading,” the list goes on. It’s already been done.

    tl;dr: Private road systems exist in the US. The thought of them being on a mass scale for society is ridiculous. Libertarians have strong opinions on topics they know nothing about.

    [*]

    An easement is a nonpossessory right to use and/or enter onto the real property of another without possessing it. It is “best typified in the right of way which one landowner, A, may enjoy over the land of another, B”.

    https://www.reddit.com/r/EnoughLibertarianSpam/comments/2okxca/my_experience_with_private_roads_and_why/

    • Evan Þ says:

      Great! Private roads exist! Where there’re incentives, they’ll be built!

      With existing private roads, like he says, there aren’t any incentives to open them to the public. But where there are incentives, they’ll be opened. Gate issues will be dealt with. Someone will make a road access department, and deal with all these issues, so normal travelers won’t need to worry about it.

    • Thegnskald says:

      I’ve driven on privately-built-and-operated toll roads, and curiously, because they were designed for traffic (rather than being designed for privacy or keeping cattle in, with traffic as a secondary or tertiary concern), they didn’t have those issues.

      This is the worst kind of extrapolation.

    • Kyp says:

      Libertarians don’t engage with the “who will build the roads?” question because it isn’t asked in good faith almost ever. It’d be like expecting a feminist organization to routinely engage with “haha but who will make my sandwiches?” when they know that there’s zero interest in an actual conversation about it. The person posting here seems to have real concerns, but acts as though he can’t possibly understand why a topic used derisively would not be taken seriously (and is it any wonder, given the unconcealed dislike for libertarians he clearly feels?)

      “Why won’t anyone provide a thoughtful answer to my bad faith barb?” is not a real question and I’m not inclined to treat it as such. This is sort of the whole thing that led to this post, that people present dishonest readings, isolated demands for rigor, or sarcastic and insulting takes (libertarianism is caused by autism professor, for example) and then act shocked when they don’t produce legitimate conversation.

      There’s a lot of people willing to legitimately engage on these topics scattered throughout these comments, it seems unnecessary to approach it from such an obtuse angle.

  41. eqdw says:

    Scott does a much better job of making this point, I think, than I do, but since it’s rather important to me I’d like to say it in my own words too:

    The reason I am a libertarian, and the reason that most the libertarians I know are libertarians, is not because the hypothetical ancap utopia would be the perfect system of human organization

    The reason we are libertarians is because we believe that a society that is marginally more ancap than the one we currently live in would be marginally better than the one we currently live in.

    It’s frustrating that I have to explain this, because it feels to me kind of like saying: “Oh, you’re thirsty? Well if you drank infinite water you’d drown, so fuck you I’m not giving you any water”. I’m sure you could produce a handful of idiots who would, with a straight face, argue “yeah but if I was drowning I’d never need a glass of water again!”. But the pro-water people, generally, rightfully think this is stupid.

    To make this point concrete wrt the four hypotheticals posed:

    —-

    Q1: Is eating the boy’s corpse after he dies the only potential violation of libertarian principles in the village? Is every single other aspect of this completely permissible?

    Conditional on acceptance of the cannibalism ritual in the first place, a world in which the hypothetical happens as posed is strictly better than a world in which the society brings him to the sacrificial altar at gunpoint.

    Or, to moderate the point even further: at any individual step along that scenario, replacing the use of force with social coercion is a marginal improvement

    (note, personally, that I think social pressure like that is also coercive, and so if I was tasked with defending the NAP in this situation, I would short circuit the entire scenario and say “nope, those tribesmen are still evil”)

    —-

    Q2: Is there a meaningful difference between coercion by the state and coercion by private entities?

    No. Our current world sees coercion done by private entities as orders of magnitude worse than coercion done by state entities. I believe that coercion, regardless of source, is bad. One way of living this principle is to advocate for private coercion to be seen as less bad. However, the goal of this isn’t to eventually get to the point of “lol private coercion is A-OK”. The goal is to get to a point of “we are ambivalent about coercion with respect to it’s source; regardless of source, it is equally bad”

    —-

    Q3: Can you construct a theory of property rights that does not suffer from internal incoherence or depend on specious natural law assumptions?

    > Does not suffer from internal incoherence
    Yes I could, but it would probably have to be scope-constrained to the point of uselessness, so let’s not

    > specious natural law assumptions
    They’re _all_ specious.

    The reason I care a lot about property rights isn’t because of some bizarre hypothetical regarding property rights being the metaphysically superior morality. It’s because when these rights are put into practice, they reliably generate better outcomes than systems that do not honour/emphasize these rights

    Does this mean that a world of perfectly enforced fully privatized property rights would be the ideal world? No! But it does mean two things. First, that (again) marginally moving from where we currently are, to a world with marginally stronger property rights would be a net benefit. And second, that marginally moving from where we currently are, to a world with marginally weaker property rights would be a net cost.

    —-

    Q4: Questions for Libertarians: Has the non-aggression principle been violated? Should the Infinitely Rich Man suffer any civil or criminal penalties for his actions?

    Actually this one I won’t use “marginal libertarianism” to defend, but will defend as actually totally 100% fine on the face of it. Scott got it right: We already have tools that perfectly solves this problem. But I will add to that the following:

    1) It is actually currently both legal and possible for Bill Gates or someone to do this right now! There’s no law, no coercive force to stop him. And yet he doesn’t do it. This is strong evidence for throwing out the entire hypothetical as absurd

    1b) Peter Thiel has done exactly this to Gawker, and it is not clear to me that this is an unimaginable tragedy

    2) The hypothetical completed erases everybody’s agency. Why is it (eg) immoral for the rich person to offer this money to the wife, and not equally immoral for the wife to accept it? According to the rules of the hypothetical, the wife will die if she doesn’t get $100k/mo. She sure as hell wasn’t getting $100k/mo before the rich guy entered the picture, which means the status quo outcome is “she dies”. If the rich person comes on the scene and gives her money, this is considered immoral. If he doesn’t, she dies. We the reader are led to believe that the morally mandatory outcome for this scenario is that the rich guy withholds money and the woman dies. Meanwhile, we’re expected to take it for granted that the woman will not make this decision. And yet, she’s held blameless for choosing the money, even though we are led to a position where we are supposed to believe that the morally mandatory outcome is “woman gets no money, and dies”

    3) (This is mostly tongue in cheek) It’s not even clear to me that the rich man’s actions are evil. Sure, _one guy’s_ life is ruined. But look at all the other consequences:
    * He massively subsidizes the castanet industry, transferring his own wealth (via bribes) to the other factory owners
    * He fully satisfies the global demand for castanets, via his roadside free castanets giveaway
    * He lowers the cost of housing in your neighbourhood, giving poor people a cheaper place to live
    * He turns dozens of your friends into millionaires
    * He saves your wife’s life
    * He massively reduces global CO2 emissions

    Hell, now that I write this out, I’m not even sure I’m joking anymore. I expect that most of your knee-jerk reactions would be “BUT IT’S WRONG to do all those things if it involves destroying someone’s life” but are you sure of that? Since we’re defining this guy as evil, every dollar he gives to someone else is a dollar he can no longer use to do evil, but someone else can use to do good. Even if all of those people spend every dollar selfishly and hedonistically, the net utility generated more than offsets the destruction of _one_ life. If I really wanted to be a smug jerk, I could point out that the improvement to air quality that his electric vehicle push generates probably saves more than one life.

    But more importantly than any of that: We live in a world where right here, right now, where lawmakers compel this exact tradeoff on our society. Remember the terrible wildfires in socal a month or so ago? Did you know that the state of California forces prisoners to work as “volunteer” firefighters? I recall hearing some news articles a few months ago (can’t find source offhand) that said that many of these prisoners were being denied parole solely on the basis of that they were fighting the fires, and if they were paroled they would stop fighting the fires and then then fires would destroy more houses and claim more lives.

    I can’t underscore this enough: not two months ago we had the government intentionally keeping prisoners in prison longer than they otherwise would, for the sole purpose of _enslaving_ them, with the justification that their sacrifice serves a greater good. And huh, we didn’t even need strict ideological libertarianism to get to that point!

    • Swami says:

      Eqdw,

      Excellent comment.

      As a classical liberal, I tend to agree directionally with libertarians on many issues. My parting of the ways is with their axiomatic bottoms up way of thinking based upon some imagined natural rights, or, at the other extreme, their taking what are best viewed as useful heuristics and treating them as absolutes.

      I guess somewhere along the line I came to the conclusion that rights and sacrosanct beliefs are best viewed as heuristics which are so important that they should rarely be questioned. But sometimes they have to be questioned, especially when running into other rights or sacrosanct beliefs. When I run into strong arguments with libertarians, it always seems like they are treating these sacrosanct heuristics as actually beyond question.

    • Thegnskald says:

      I think the issue with the “marginal improvement” approach is that marginal libertarianism tends to make things worse, not better.

      This isn’t because libertarianism is worse than what we have now – on net, I’d rather have a libertarian society than the one we have – but because the same forces and incentives which pervert government in the general case, likewise pervert liberalization itself.

      Russian privatization is a great example of this. In theory, they went from a mafia-run “socialist” state to a capitalist one. In practice, things went from “run by the mafia” to “owned by individual members of the mafia”.

      Regulation is another; deregulation tends to focus on removing the social support structures, leaving in place an unbalanced, easily-pillaged mess, precisely in a fashion which makes it easy for companies to make money, but often without the incentive structures that would make it work. So a power grid gets deregulated, so the companies can charge what they like, and competitors can theoretically undercut them – but other regulatory structures around building power plants remain in place (which was dealt with by the original power plant either through subsidy or grandfathering), so the competition can’t actually arise to compete.

      Marginal moves to libertarianism tend to make things worse, not better, because everything has already been built around the existing structure. It is like remodeling a wall a brick at a time.

      • Cliff says:

        In Virginia, we had a law that brewers could not sell full size beers at the brewery. This law, like so many others, served no legitimate purpose. That law was removed four years ago. Since then, the craft beer industry has boomed and created maybe 100 new breweries and thousands of jobs. There are literally hundreds of regulations just like this on Virginia’s books right now. Regulations that serve absolutely no legitimate purpose and that we could just get rid of without any other change and greatly improve life for Virginians- especially those toward the bottom of the income scale. And Virginia is certainly not unique in this respect.

        • Thegnskald says:

          No legitimate purpose?

          What a fascinating law. Were the legislators drunk when they passed it? Maybe they got drunk at their local brewery on cheap beer.

          • gbdub says:

            It certainly served a purpose – usually it made the legislators’ friends in competing industries happy. So just not a legitimate one.

            I think it was Maryland where it’s even worse. There’s a bill / law (not sure its current state) to limit craft breweries from selling too much beer at their taprooms (and to force them to close at 9). One of the main sponsors has multiple family members in the (macro) beer distribution business.

            My favorite part was this same legislator saying they had visited a taproom and were “shocked” to find it operating “more like a bar” as if that were some horrible travesty the rest of us are supposed to be shocked by.

            And beyond corruption, there is of course the simple problem of legislators being dumb. Just because they think a regulation will serve a purpose doesn’t mean it actually will.

          • Cliff says:

            Generally they are anti-competitive giveaways drafted by the big businesses that dominate whatever the industry is

          • Thegnskald says:

            Cliff –

            Wasn’t the previous law that you couldn’t serve full-size beers at breweries that didn’t have a restaurant attached, rather than that you couldn’t serve them at all?

            Which is a requirement mixed-beverage (beer and liquor, basically) establishments already had to follow. I am having trouble finding any permits to just sell beer, so it isn’t clear to me that this regulation change isn’t itself a form of crony capitalism benefiting breweries over bars.

      • eqdw says:

        I know this is a shitty cop-out answer but:

        If what you’re doing gets perverted into crony capitalism, then that wasn’t a move in the direction of marginal libertarianism, and I am not in favour of it.

        I agree that this happens, a lot, and that’s why it’s incredibly important to understand the nuances of why libertarian ideas are good. That’s why it’s important to understand the specifics of any given system before making changes to it. And, most importantly, that’s why it’s important to pay attention to what leaders _actually do_ instead of just what they say they’ll do

        • Thegnskald says:

          Regulatory capture may mean that crony capitalism is the most likely outcome, rather than an unusual exception. The same processes that make construction of regulation a corruption-frought enterprise may apply just as much to removing regulation.

      • Salem says:

        I think the issue with the “marginal improvement” approach is that marginal libertarianism tends to make things worse, not better.

        OK, but now this is a serious, fact-based discussion we can have.

        Do, on net, marginal moves in a libertarian direction, such as the privatisation of previously nationalised industries, lowering marginal tax rates, legalising homosexuality, softening the drug laws, removing excess regulation, allowing council tenants to buy their houses, ending state-granted monopolies, etc, have a positive or negative effect? And if some are positive, and some negative, then which ones, and on what margins?

        I live in the UK, where all these moves have been made in the past few decades, and they’ve all been huge successes. As a result, I support further marginal moves in a libertarian direction, until the results stop being so positive. I can understand how different people, in different societies and contexts, might take a different view. But note how different such a discussion looks from the CA article.

        • Thegnskald says:

          To pick the one example I am aware of in the UK, didn’t the privatization of public transit result in a maintenance deficit that ultimately led to the transit contracts being bought back by the government?

          Which is to say – didn’t the companies just pillage the public transit system for as much money as they could, then make more money when the government was forced to take it back over?

          • Salem says:

            No, that is not an accurate summary. In fact I take a privately run train most days, which runs far better than the publicly run train at the other end of town.

            What you may be thinking of is that RailTrack, a company which was granted a state monopoly on railway maintenance, went bust, and its investors lost everything.

            In other words, the facts are almost the opposite of each of your paragraphs.

            But perhaps in other countries things have gone differently. For instance, I do agree with you that the Russian voucher privatisation schemes, and the cash-for-loans in particular, appear to have been very bad (although I am far from an expert and perhaps they were the least-bad option at the time). The key point is it’s a fact-dependent enquiry, which should shame the people at Current Affairs who pretend to be a news magazine.

          • spkaca says:

            “didn’t the privatization of public transit result in a maintenance deficit”
            The maintenance deficit already existed when the railways were in public ownership. I can remember what they were like. They didn’t get noticeably worse or better after privatisation. The main issue in the UK I suspect is/was the great age of the infrastructure.

    • Iain says:

      The reason I am a libertarian, and the reason that most the libertarians I know are libertarians, is not because the hypothetical ancap utopia would be the perfect system of human organization.
      The reason we are libertarians is because we believe that a society that is marginally more ancap than the one we currently live in would be marginally better than the one we currently live in.

      This is a very good justification for taking libertarian stances on specific issues. Libertarians are clearly right about the correct marginal direction on a lot of issues: pretty much anything to do with criminal justice, as just one example.

      Nevertheless, it seems dangerous to elevate this observation to a general principle. Above, Thegnskald argues that deregulation and privatization often go badly. In the same way that these arguments can be true without proving that marginal moves towards libertarianism are always bad, it’s a bad idea to look at places where more libertarianism would help and conclude that it is always good.

      It would certainly be convenient if marginal moves towards Ancaptopia were nigh-universally improvements on the status quo. That doesn’t mean it’s true. My experience matches what Swami says above: too often, libertarians take a genuinely useful heuristic and become too heavily invested in it, distorting their analysis of situations where other approaches might work better.

      Duct tape is a great solution to a lot of problems, but it would be a mistake to align my entire political philosophy around the idea that duct tape is always best.

      • themountaingoat says:

        Exactly. I am sceptical of libertarians precisely because many of them seem to believe general principle so strongly. I think there is a strong need for good evidence based criticism of individual regulations but if someone thinks that all regulation is bad (when they can’t possibly have looked at all possible regulations) I cannot really assume they are arguing in good faith.

        I think if libertarians want to make society more libertarian they need to argue for libertarian principle and entirely stop arguing the extreme views that all regulation is bad. You should never have much confidence in such an extreme position anyway.

        • Skivverus says:

          (when they can’t possibly have looked at all possible regulations)

          This is not, in the libertarian mind, a point in favor of regulations. Or laws in general, for that matter – it means their adherence becomes a matter of faith in the regulating authority rather than in the actual beneficial effects of the laws in question, proportional to the ratio of known to unknown laws.
          That there are good regulations – or conventions, or standards, or customs – is not reasonably disputed; the trick is figuring out which ones those are. The libertarian take is that the burden of proof should be on the regulation’s existence, rather than its challenger(s).
          One can certainly argue that there should be escalating standards of proof for both regulation and challenger – we’re pointing out the existence of idiotic laws, not the absence of idiotic lawbreakers.

          • themountaingoat says:

            The belief that we should merely be sceptical of new regulations isn’t one I see commonly articulated by self described libertarians. I see more arguments that are either from a general principle or attempting to justify a general principle from those people.

            For existing regulations that means you should be aware of the arguments for why the regulation was put into place when you discuss repealing it for chesterton’s fence reasons.

            I am not entirely sure what you mean by the beginning of your comment. Talking about something supporting regulations in general is silly, we should always discuss individual regulations on their merits. There is some merit of course to discussing our level of trust in lawmakers but you should apply that scepticism when lawmakers try to remove laws as well. If you distrust them you would expect them to remove only those laws that are good.

          • eqdw says:

            @themountaingoat not singling you out on this, a lot of people should hear this. But yours is the comment that prompted my thought:

            The availability heuristic is a very poor guide to what “real libertarianism” is.

            I don’t want to get into a no true scotsman argument, because it serves no purpose and (as I said above), I don’t care about implementing an ideology. But, in general: The kinds of people, ideas, and policies that are typically elevated to public awareness and labelled as “libertarian” are frequently the most absurd and ridiculous examples. Additionally, many policies that could reasonably be called libertarian are not characterized as libertarian when they gain mainstream appeal.

            Legalizing marijuana is fundamentally a libertarian idea, and yet most people would not consider this a representative example of libertarianism.

            I know very very few libertarians who take an ideological, categorical “NO REGULATIONS” position. Of the ones I do know, they’re all idiots. It is my understanding that being skeptical of regulations is a mainstream libertarian position.

            Also, a point of clarification: most libertarians I know are skeptical of regulations not because they think regulations are somehow intrinsically bad, but rather because they do not have confidence in the ability of a regulator to fairly regulate. They assume corruption will exist, and want to remove opportunities for it to exist.

          • themountaingoat says:

            @Eqdw I don’t believe it is an accident that the ideas labelled libertarian are often so ridiculous. When you tie your identity to a political viewpoint that has such specific policy prescriptions it becomes very easy to cease to be rational and objective about the policies in question. I don’t think anyone should identify as libertarian for that reason.

            The marijuana debate is a perfect example of how I wish libertarians argued all the time.

            Most libertarians are probably motivated by the belief that corruption is everywhere but it isn’t clear that more libertarian ideas would help that. If any regulations are good then we would expect politicians to remove those ones first. Libertarians also seem against several other policy suggestions to reduce corruption such as limiting lobbying.

          • Mark V Anderson says:

            The libertarian take is that the burden of proof should be on the regulation’s existence, rather than its challenger(s).

            Yes this is the key point. Too often I see someone complaining about the bad effects of some unregulated pursuit and advocate some new law, and yet never look at the actual effects of the regulation, seemingly just assuming that a regulation will improve the situation. That is, look for market failure and never government failure. And when something is already regulated and there is an issue, it seems that few activists consider the idea of de-regulating, instead the usual direction is to regulate even tighter, which in my experience makes the problem even worse. I am certainly not the first person who’s made comments like this, but in actual discussions about actual government making actual laws it seems mostly missing.

            And yes, mountaingoat, “genuine libertarians” are usually pretty radical, looking for wholesale changes in government. This isn’t something that will happen in real life, even if you didn’t have the natural resistance of professional politicians and civil servants (part of public choice issues). Even the voters won’t vote for such things, because radical change is scary, and is justifiably scary, it is dangerous. Chesterton’s fence and such, as you say. But that doesn’t mean that incremental libertarian type changes are not good. Take the best ideas and leave the rest behind. I don’t call myself a libertarian because I think going whole hog is a bad idea, and I disagree with the NAP. But I am libertarian leaning.

          • themountaingoat says:

            @Mark V Anderson Your views don’t actually seem that far from mine. However so many libertarians have such strong unsupported views I have developed a tendency to immediately assume bad faith as soon as I hear anything close to “taxation is theft”.

            There are of course many awful government programs especially in the states. I think many of those programs could actually be removed without too much backlash if aggregate demand was high enough that people could easily find jobs elsewhere in a hurry. I am somewhat leftist because I believe that we are currently strongly demand limited and increasing aggregate demand is currently the most important function of government.

          • eqdw says:

            It is unclear to me if you all are saying this, exactly, but I feel the need to clarify what I was trying to say in blunter and more direct language:

            It is not fair to consider “That one weirdo who is always posting retarded memes on Facebook” to be a representative example of libertarianism, and then to consider “a staff writer for vox dot com” to be a representative example of progressivism.

            Either compare respected academic representatives to each other (and pick, say, academic economists like David Friedman), or compare shitty strawmans to each other (and pick, say, the progressives who say “Stalin did nothing wrong” (I have personally met someone who said this, exact quote, in absolute sincerity))

            If you’re going to compare the worst possible example of group one with the best possible example of group two, and then conclude that group one is insane and group two is reasonable, then you are being unreasonable

            And again, disclaimer: it is not clear to me that you all are saying this. But it sounded like you might have been, and this was my original point

          • Skivverus says:

            A slight tangent: there’s a concept in programming known as “technical debt”, where as more and more code gets added to a program, the amount of time required to understand (and thus debug) it grows in tandem. Paying down technical debt involves refactoring and removing code rather than adding any: no new features, but it allows for faster coding and fewer bugs in the future.

            I think this concept is transferable to law. More to the point, I think when it comes to regulation we have a sizable amount of technical debt (not just financial debt) that needs paying down.

    • PDP10 says:

      > 1b) Peter Thiel has done exactly this to Gawker, and it is not clear to me that this is an unimaginable tragedy

      I was anticipating the bit that lampshaded that, but scott took the high road on that one. So insert this bit at an appropriate point:

      “It seems so open-ended though! Does your rich-person-grudge insurance really cover anything that I could ever do to you?”

      “Oh, not at all. It specifically doesn’t cover financial or personal harm done to me as a consequence of circumstances only incidental to the grudge. So for example, if I drive drunk and kill a random person, you might back that person’s family in a wrongful death lawsuit against me. In that case the insurance wouldn’t kick in because the central issue is driving drunk, not an insane grudge by a wealthy crackpot. I’d need drunk-driving insurance, which believe me has vastly higher premiums.”

  42. jw says:

    I love how socialists call libertarians out on the carpet all the time nowadays.

    The is complete and total bullshit because:

    Libertarianism has never erected a Berlin Wall to keep its citizens from escaping.
    Libertarianism has never demanded collectivism from the farmers and taken all their food to leave them to starve.
    Libertarianism has never demanded strict adherence to its values and killed those who disagree in struggle sessions or tried and convicted them in star chambers.

    I’m flabbergasted at how Socialists/Communists (hey don’t complain I group these, we’re strawmannirg here aren’t we) can ridicule libertarianism. Libertarianism, as Horrid, as Socialists make it out to be DID NOT kill 100 million people in the 20th century….

    Socialists are heaving those stones from the biggest glass house ever built.

    • Murphy says:

      “Libertarianism has never”

      Sure, but the hypothetical political system of purple-hat-evil-ism under which all choices are made such as to maximize evil and/or the number of purple hats worn also has never done any of those things.

      People are still allowed to criticize purple-hat-evil-ism if I push for it and the comeback of “your system: X million deaths, mine : zero!” rings hollow when my system has simply never got the chance to commit the big atrocities.

  43. antilles says:

    It’s kind of amazing that Scott argues for treating your opponents with charity and not picking the worst, dumbest defenders of a viewpoint in a post mocking the worst, dumbest defenders of a viewpoint.

    • Thegnskald says:

      Scott is attacking somebody for NOT treating opponents with charity.

      Scott himself has written exhaustive criticisms of libertarianism.

      So he is attacking somebody who is nominally on his side for making terrible arguments.

      This is a norm I would like to promote, not dissuade.

      • antilles says:

        It is, incredibly enough, possible to chastise someone for not behaving with charity or clear thinking while oneself engaging in charity and clear thinking. “Mocking, thinly veiled satire” is… not that. And usually Scott’s satirical writing is broader and both sides get some licks in, because usually he focuses on better targets.

        • Thegnskald says:

          He has already given libertarianism its “licks”. He has an entire essay dedicated to this purpose. Go read that; he has already done what you are demanding he does, and all you are asking him to do now is repeat himself.

          This is an isolated demand for rigor in the context of somebody who has already exercised that rigor.

          • antilles says:

            “Being fair to critics of libertarianism” is not equivalent to “being willing to engage in criticism of libertarianism.”

          • Thegnskald says:

            In the context of steelmanning and charity – yes it is.

            He has already steelmanned their argument. He has demonstrated extreme charity towards their viewpoint.

            I don’t think you understand these concepts.

            What he is not being, is Nice. But he is being both right and necessary.

    • Murphy says:

      it’s mocking the attackers of a viewpoint.

      • antilles says:

        “The other person started it” is rarely a clear or adequate justification in real conflict; it is even less helpful in intellectual debate since arguments rarely have a clear point of origin or distinct boundaries. Saying that Current Affairs is “attacking” a viewpoint so it’s okay to attack back is just bad, bad excuse-making grounded in poor framing.

    • Edward Scizorhands says:

      Current Affairs isn’t some random college kid’s blog that Scott scoured the Internet for hours to find.

      • antilles says:

        Scott’s view is about what promotes good debate by responding to the strongest version of an argument, not about discrediting bad actors because of their relative prominence. That approach is far too tactical to jive with his stated goals.

        • Nick says:

          It’s not okay to single out prominent bad actors, but that’s not what Current Affairs is. Robinson is a friend of the blog (well, speaking loosely at least), and Scott’s engaged him fruitfully before. I think you could see it as mocking an intellectual peer for falling below his usual standards, and I’m not sure there’s anything wrong with that.

        • Edward Scizorhands says:

          If this was some random’s kid blog, we could, with charity, assume that the kid had never before encountered “but what if it’s taken to the logical extreem!” before.

          Given that there is no good faith effort on CA’s part (“libertarianism is psychotic”), the charitable view is that they meant it all as a joke, and will laugh at Scott’s joke in return.

          • Toby Bartels says:

            Yes, exactly! Current Affairs was having a laugh, and Scott had a laugh back. (And Scott was funnier too.) If you take it seriously, then it's an uncharacteristically uncharitable attack on the weakest arguments. But if you take it as a joke, then it's just funny.

    • moscanarius says:

      It’s called the Principle of Charity, not the Adamantium Law of Endless Unrestricted Charity.

      The principle is stated to remind people that their opponents are not necessarily evil or stupid or lesser than them just because they disagree. Let’s say it fully: a person concerned with knowledge and justice should try to seek reasons why their opponents could be right, or at least justified in their wrongness, or at the very least misguided. This is done by addressing the best versions of their arguments, believing they could be right about something even if they express it poorly, and assuming that most of the disagreement will more often boil down to incomplete information than to stupidity, and to stupidity more often than to malice. The assumption of lack of malice also demands that a fair person be kind to their opponents, as you would be kind to someone who is misguided but has the heart on the right place.

      Nothing about this principle means you should never criticize a stupid argument or one made in bad faith. Nothing in it says you should not mock people back when they mock you; it’s just a reminder not to dwell too much on the worst arguments, not to be too harsh on the worst offenders, not to assume everyone you meet has a bad argument, and not to assume that bad arguments are all the opposition is about. Scott has given thoughtful consideration to antilibertarian criticism before; this time, he is gently mocking a bad half-serious position that happened to appear on a big website. It’s as simple as that.

  44. Tenacious D says:

    The best libertarian dystopia in fiction (way better than cannibal villages surrounded by direwolf-infested wilderness) is Snow Crash. It’s got private roads, passports for every subdivision, a guy with his own nuke, and a floating pentecostal refugee camp. But I think I’d prefer to live there than in other fictional dystopias such as 1984, Hunger Games, Brave New World, The Giver, etc. Of course, not living in a dystopia is best, but as eqdw points out, marginal improvements matter.

  45. P. George Stewart says:

    Great stuff, rare to get such high level intellectual stimulation and a wicked sense of humour in such a neat package.

    I think the NAP isn’t quite the heart of libertarianism that some think it is. It takes the “negative” tone (what NOT to do) over the positive tone (what SHOULD be done). But I think this is a mistake re. levels of discourse. “Negative” freedom is indeed the fundamental freedom in a social context (i.e. positive freedom is nobody’s business but your own, negative freedom is where the question of other people comes in), but I think the NAP actually derives from a deeper, positive account.

    The trouble is, we don’t have the lingo for it. It should be something like “the Principle of Moral Inertia.” That is to say, the principle that human being, being human, ought to be considered innocent until proven guilty, which means human beings ought to be allowed to go their own way without interference until and unless there’s a good reason to interfere or to stop them. Here, the law is intuitively deeper than most of the philosophical/moral thought around these issues. (IOW, the answer’s been staring us in the face all along.)

    So we have a picture of individual agents as notional trajectories through possibility space, going all over the shop, criss-crossing, going off on their own to explore recondite nooks and crannies, etc., etc., with those trajectories involving control of stuff in one way or another (and the very first thing found out of the state of nature, available for control, for all of us, the jumping off point for all of us, is of course one’s body and faculties; next, what objects one finds to use or control in nature; next, what one gets to control via voluntary exchange). And we, together as all the human beings there are, are a community of such exploration paths through possibility space.

    The question of freedom is then whether and under what circumstances we (acting in concert, or via proxy) should let any given person A continue along their own trajectory under their own steam, and whether, and under what circumstances we should interfere, stop, knock off course, etc.

    And it’s only at that point that the NAP comes in, but putting it as “non-aggression” still doesn’t get the point: the fact is that you, Person B, can initiate aggression, interfere, etc., etc., with person A’s trajectory but you need good reason to do so, good reason in the eyes of all of us other observers and onlookers, which then determines whether they will let you interfere with person A (because if you don’t give good reason for your interfering with some random person, the onlookers can’t be sure you won’t randomly interfere with them).

    And good reason usually involves something like “harm,” and 9 times out of 10 “harm” itself is interfering with some other innocent’s trajectory.

    So the point is really about where the burden of justification lies. Other political systems seem to assume that the burden of justification is on the doer, the agent, the owner, to justify their trajectory through life, their use and control of things, that they’re doing no harm. You, the owner-of-stuff, must justify your ownership. But that’s wrong, and moreover impossible to implement, because you can’t prove a negative. The other way round is better: innocent until proven guilty. Do what thou wilt shall be the whole of the Law (and everyone’s in the same boat). The proving-guilty is specific and demonstrable, it has a potential terminus that’s agreeable beforehand, and can form the basis for justification in aggressing or interfering (though of course it can be abused – we just have to be alert to that possibility).

    What falls out of this is “finders keepers” (out of a state of nature), with voluntary exchange therafter. This is uncannily analogous to TIT for TAT. Who knew that such fundamentally benign (but non-pushover) rules-of-thumb would be baked into reality itself, almost like a signature? Here we feel in the position of the guy in the engraving, poking his head through to the machinery of the Spheres. Why, it’s almost enough to make one religious …

    • Thegnskald says:

      The NAP doesn’t obviously apply to property, though.

      Some libertarian used to comment here on this, to the gist, IIRC, that if we had no notion of land rights, and everybody subsisted on going out and foraging for food, the idea of planting something that is exclusively yours would be alien and weird. We can extend the idea further by supposing foragers regularly plant seeds as they forage, which makes the weirdness more apparent; what you are doing, in the context of that culture, is threatening people with violence for doing something.

      It takes a shared cultural agreement – a social technology – to arrive at land ownership as a meaningful concept, to arrive at the idea that planting and watering and fertilizing a plant makes it’s product yours.

      And even if it is a useful technology, it is still a technology, and has no basis in nature.

      • P. George Stewart says:

        Yes, but that social agreement can be understood as a discovery – a discovery of a better way than commons, etc.

        Out there, somewhere in possibility space, is the set of social rules that make for the good life collectively (and for each individual within that collective, individually), given the nature of the world and the relatively stable nature (over time) of human beings (and taking into account changing circumstances as a result of technology, etc.).

        That being the case, the creation of social rules isn’t arbitrary or subjective, it’s a discovery process of an objective structure of possibilities. The fact that it’s a discovery process is really what undergirds the idea of classical liberalism – the fact that none of us have a hotline to Reality to know beforehand what the good life is, and that it must be discovered, therefore society must be structured, to some extent, as an experiment (this is the social level of the Enlightenment echoing the Baconian scientific level).

        In the case of property, the utility of the rule is that it allows for “controlled explosion” in the case of making wrong decisions (it restricts as much as possible, the feedback from bad decisions, to the individual) and it provides incentive to make the right decisions (because you reap the rewards). Dividing up the costs/benefits from actions between the smallest possible functional units (individuals) was a great discovery, and it’s based on a rule so simple that even a child can understand it: leave people in control of whatever they control, until and unless they cede control voluntarily, or are doing harm.

        That was a great discovery – and the fact that primitives hadn’t discovered it was the reason they were primitives. And the fact that the ramifications of the simple rule still aren’t understood by large numbers of people is the thing that’s holding us back from having flying cars 🙂

        • Thegnskald says:

          “Discovery”. Alright.

          Tomorrow we discover a new social structure, in which freedom is dramatically curtailed, but in which humanity flourishes.

          Do you stop being a libertarian because something better has been discovered?

          What if that discovery happened eighty years ago, and the current system is legitimately the best, because libertarianism is inherently unstable, and even though it is in the short-term better, the process of maintaining it (blood of patriots and tyrants) is both costly and error-prone, leading to horrific dictatorships as often as a functional government? Suppose that the current system is inferior over any increment of time, but pays for itself with long-term stability, since libertarianism inevitably just turns into a welfare state.

          Does liberty stop mattering? If somebody could prove it, would you change political systems?

          • Salem says:

            Tomorrow we discover a new social structure, in which freedom is dramatically curtailed, but in which humanity flourishes.

            Do you stop being a libertarian because something better has been discovered?

            Short answer: Yes, obviously.

            Longer answer: “Flourishing” is a value-laden term. I think some measure of individual freedom is baked into what it means for a society to be flourishing – I would not judge as flourishing a society of material comfort but no choice or self-expression. As such, liberty will never “stop mattering.” I don’t deny that a society where “freedom is dramatically curtailed” has a longer road to hoe to get to flourishing than a freer one. But liberty is not the only value, and if (contrary to my present view) it turned out to be important to reduce liberty to obtain other important goals, then I would update accordingly.

          • P. George Stewart says:

            Salem answered you in part, but I don’t think negative liberty is negotiable.

            Positive liberty (self-mastery, power, etc.) yes, you can see that being a trade-off in various ways, but negative liberty can’t possibly be a trade-off because it’s the very precondition for social order: it’s the best possible rule about who gets to control what when.

            Not sure what your reference to 1938 is all about 🙂

  46. Guy in TN says:

    Point taken.

    But there was a time, just a few years ago, when you couldn’t pass a corner without finding a libertarian talking about how all of morality is Natural Rights derived from the Self-Ownership Axiom. In the communities that I was a part of, this Rothbard/Hoppe style of discourse was an epidemic. Maybe in these more sophisticated circles it never caught on. But it did to the general public.

    If 90% of all socialists hinged their arguments on the Labor Theory of Value, and you wanted to do a sly parody demonstrating why this is insufficient, would it be weak-manning necessarily, to ignore the 10% of socialists who were more sophisticated and nuanced? If you insist on only addressing the strongest arguments conceivable, at some point you are no longer even addressing the beliefs held by the run of the mill Facebook-Youtube political-opinionator.

    Its easy, in the SSC bubble, to begin to think that the central-libertarian is as nuanced, sophisticated, and intelligent as David Friedman. How much better off we would all be, if that were the case.

    • Thegnskald says:

      Even in the context of Rothbardian libertarianism, the arguments made aren’t exactly killer. They basically come down to “How does your society deal with this bizarre edge-case problem that our society also doesn’t deal with”.

      I dealt with a lot of hypotheticals back in my libertarian days, and they largely revolve around trying to argue that libertarianism can’t solve every possible problem. Which it can’t. But that isn’t its failure mode; its failure mode is that you can’t implement it except on a blank slate, and it slowly turns into something else. You can’t even roll back to it, as incremental changes are path-dependent (which is to say, law exhibits interdependency, such that removing one law will cause another law to stop working as designed, or to start exhibiting side effect behavior). More, society is law-dependent; laws cause permanent change. Fine parents for delivering children late, and their behavior doesn’t revert if you revert the law. Likewise, the institutions we would have had in a libertarian society don’t exist in ours; a private FDA won’t crop up overnight, nor will the dozens of competitors who would help keep each other honest.

      • Cliff says:

        “You can’t even roll back to it, as incremental changes are path-dependent (which is to say, law exhibits interdependency, such that removing one law will cause another law to stop working as designed, or to start exhibiting side effect behavior).”

        We have had substantial deregulation since the ’70’s with quite some success

        • Thegnskald says:

          Sort of.

          I mean, sure, we deregulated the airlines, but ended up regulating the passengers instead.

          Likewise, we deregulated the power grid, but regulated the shit out of consumer goods that use power, such that, leaving computers aside, our power consumption today looks like the 1930s or 1940s.

          It looks more like the burden of regulation has been shifted from industry to consumers, than that we have actually rolled it back at all.

          • Cliff says:

            There are many examples of deregulation all over the world, including Thatcher’s UK, recent experience in Canada, etc. The idea that thee airlines are not significantly less regulated than they were in the ’70s is in my opinion not reasonable. I don’t think you have the evidence to support an argument that regulation only goes in one direction and can never be rolled back. Trump is cutting a lot of regulations actually.

          • Thegnskald says:

            Cliff –

            You are missing my point. Let us focus on one thing: Alcohol.

            Theoretically, alcohol is way, way less regulated today than it was fifty years ago.

            In practice, while we have rolled back regulation on the production of alcohol, we have vastly increased regulation on the consumption of alcohol.

            We didn’t stop trying to solve the same social problems with regulations – instead we moved the burden of regulation from the producers to the consumers. So now, instead of not being able to buy a beer at a brewery, I get stopped at a drunk driving checkpoint leaving the brewery.

            I am not so sanguine as you that this is a positive development. On the one hand, it increases our freedom to consume alcohol – on the other, it increases the police and nanny state.

            And it negatively impacts those who consume no alcohol, who get stopped at checkpoints anyways.

    • Guy in TN says:

      I didn’t have the patience to read the Current Affairs article. My conception of Nathan Robinson is “naive, big-idea-dreamer with his heart in the right place”, and therefore find more interesting his articles where he is advancing bold and unusual ideas, rather than those where he attempts an ill-conceived intellectual take-down.

      But I will have to come to the defense of a well-placed hypothetical question. If someone is proposing what they believe to be a water-tight system, a searing hypothetical could be the slash that causes the whole construct to crumble. If these hypothetical questions cause people to shift towards making more nuanced arguments, and allowing for ethical gray in their worldview, then we are the better for it.

      • Toby Bartels says:

        I didn’t have the patience to read the Current Affairs article.

        If you read Scott's post, then you read the Current Affairs article. (There's only a small bit at the end that he cut out, I think.)

  47. daneelssoul says:

    So 2,3, and 4 seem like pretty terrible arguments. But starving because nobody else is willing to trade food to you seems pretty close to the types of problems that poor people might face in places with poor social safety nets. Now granted the conspiracy of everyone else explicitly agreeing to make you starve is a bit ridiculous, but it could more charitably be interpreted as exaggeration for the purposes of philosophical argument.

    Also for what it’s worth, I have met multiple non-aggression-as-a-fundamental-moral-principle-libertarians. They might not be as small a minority as you think. Then again, you’ve probably talked to more libertarians than I have.

    • Mark V Anderson says:

      The reason the initial hypothetical case is so bad (besides the fact that every other political theory such as democracy, communism, etc. would be even worse in this situation) is that it postulates an extremely isolated community with a near unanimous point of view that we all see as bad. I don’t know that any political system would be better than libertarianism in such a community, but it isn’t anything like almost all communities in modern society. We don’t have isolated communities that one can’t escape from, and few communities have such broad agreement. How many communities have 100% agreement that some person in their community should not be allowed to work, AND that person also can’t go to any other community? Even the first part is very rare. In real life, everyone will find somewhere to work, as long as the coercive government doesn’t have laws to stop you. Is there any kind of nation-wide network of 100% agreement that certain kinds of people shouldn’t work? In any country in the world? That’s why the government’s coercion is considered the worst, because it is blanket over everyone.

      I myself am not a strong libertarian, only lean in that direction, so I do think government welfare is a good idea. But the idea that libertarians allowing some kind of non-aggression as effectively being worse than government coercion is kind of nonsensical.

      • daneelssoul says:

        I think the charitable view of the argument is that people can be effectively murdered in societies with non-aggression laws and strong anti-theft laws by people being unwilling to trade you food. The idea that there might be a literal conspiracy of 100% of the rest of society working together to guarantee that you starve is far-fetched, it is a clear illustration of what is potentially wrong with not providing some guarantee of being able to obtain basic necessities. And while the literal conspiracy of literally everyone else is not remotely realistic, there is a reasonably accurate real-world analogue:
        Most grocers have agreed to not trade food to people without money.

        • The idea that there might be a literal conspiracy of 100% of the rest of society working together to guarantee that you starve is far-fetched, it is a clear illustration of what is potentially wrong with not providing some guarantee of being able to obtain basic necessities.

          How do you provide a guarantee that will hold against unanimous opposition? How do your write a guarantee into law that will hold against even 90% opposition–defend someone who 90% of the population want to get?

          The U.S. constitution has a bunch of guarantees in it. They did not prevent the Nisei, American citizens who had committed no crime and were accused of no crime, from being imprisoned and their property effectively stolen. They did not prevent the Trail of Tears.

          90% of the population is likely to include a majority of the Supreme Court.

          To starve someone under laissez-faire requires something close to a hundred percent coordination.

          • daneelssoul says:

            How do you provide a guarantee that will hold against unanimous opposition? How do your write a guarantee into law that will hold against even 90% opposition–defend someone who 90% of the population want to get?

            I think you misunderstand my objection. My objection is with this:

            To starve someone under laissez-faire requires something close to a hundred percent coordination.

            This is false. It doesn’t require anything close to full coordination. It merely requires someone with mental or physical disabilities or just a lack of useful skills during a bad economic downturn. If you cannot find employment and there is a gap in the coverage of available charities, you do actually risk death by malnutrition or exposure. There is already a wide ranging conspiracy to not trade things to people without money.

          • quanta413 says:

            This is false. It doesn’t require anything close to full coordination. It merely requires someone with mental or physical disabilities or just a lack of useful skills during a bad economic downturn. If you cannot find employment and there is a gap in the coverage of available charities, you do actually risk death by malnutrition or exposure. There is already a wide ranging conspiracy to not trade things to people without money.

            Is there good evidence that strong central governments are a more effective bulwark against starvation than the Mormon Church or the Catholic Church?

            The change compared to the past or compared to less developed countries looks almost totally due to technological progress to me.

          • daneelssoul says:

            Is there good evidence that strong central governments are a more effective bulwark against starvation than the Mormon Church or the Catholic Church?

            Not sure. I suspect that this would be the case (especially if you are LGBT or atheist or something), but I don’t really have any hard data.

            The change compared to the past or compared to less developed countries looks almost totally due to technological progress to me.

            Though even though the world does have enough money to end hunger, aid organizations from the first world haven’t stepped up to solve the issue yet.

            —————————————————————

            But this is a bit beside my original point. My primary thesis here is just that the argument from part 1 of the post could be seen as an unrealistic, extreme hypothetical to argue against extreme versions of the libertarian moral system, and that it flows somewhat naturally into a more realistic argument against more realistic version of libertarian philosophy.

            My claim isn’t so much that this extended argument is correct so much as it is that this argument isn’t immediately unreasonable, and that at the very least there’s a charitable version of argument 1 that is being ignored.

          • quanta413 says:

            Not sure. I suspect that this would be the case (especially if you are LGBT or atheist or something), but I don’t really have any hard data.

            As far as I’m aware, Catholics have been pretty good about not tying their material aid to a requirement for conversion or a change of behavior or anything like that. At least for the last couple hundred years.

            Catholics have also been doing the charity thing for millenia now. Their overhead isn’t zero, but I’d bet it’s a lot lower than the cost of government. Tithing is only 10% and a lot of people fail to even make that contribution.

            I’d bet on the Catholics, although their reach is obviously very different than a single government. It may be greater overall than most countries, but it’s probably smaller than a first world country’s within that country. There’s almost certainly some displacement effects here.

            Though even though the world does have enough money to end hunger, aid organizations from the first world haven’t stepped up to solve the issue yet.

            Sure, but cross country aid is a totally different issue than form of government. It seems rather naive to chalk the problem off to lack of will or not trying either. A lot of famines are due to wartime conditions and blockades. How exactly will more money from another country get aid past this short of invading? Bribing the government appears to be a poor and unreliable strategy.

            My claim isn’t so much that this extended argument is correct so much as it is that this argument isn’t immediately unreasonable, and that at the very least there’s a charitable version of argument 1 that is being ignored.

            Yeah, but there’s charitable, and then there’s rewriting things down to the very premises. I think Scott should probably just not have responded at all, but if he was going to respond I don’t think there were any good responses available.

          • daneelssoul says:

            How do you provide a guarantee that will hold against unanimous opposition? How do your write a guarantee into law that will hold against even 90% opposition–defend someone who 90% of the population want to get?

            90% of the population is likely to include a majority of the Supreme Court.

            To starve someone under laissez-faire requires something close to a hundred percent coordination.

            I don’t think that this is a fair comparison. You are comparing the United States government, a real-world system whose rules must be adjudicated by real people to some sort of idealized laissez-faire libertarianism whose rules are enforced by magic.

            Any actual laissez-faire system would need to be run by people, and even ignoring weird edge cases like conspiracy-starvation-murder, it should take well under 90% of them working together to kill you.

          • 1soru1 says:

            To starve someone under laissez-faire requires something close to a hundred percent coordination.

            The question then becomes, if so, how did it historically happen so often before governments abandoned laissez-faire in food?

            Plenty of things in society require close to a hundred percent coordination, for example driving on one side or other of the road.

            If you are in a famine situation, being the one shopkeeper handing out all your food for free to those who cannot afford it _will_ see you and your family starve to death with greater probability than merely heading down the highway in the wrong direction.

          • To starve someone under laissez-faire requires something close to a hundred percent coordination.

            The question then becomes, if so, how did it historically happen so often before governments abandoned laissez-faire in food?

            I should have made it clear that I was talking about the sort of “starve someone” being discussed–lots of food out there but nobody will provide you any. Obviously, under any system, people can starve because there isn’t enough food being produced, and that can happen under laissez-faire.

            The cases I know of where the food was being produced yet large numbers starved were cases where governments had abandoned laissez-faire in food–the Ukraine famine and the famine during the Great Leap Forward. In both cases, the country where the famine occurred was on net exporting food.

            The Irish famine is a bit tricky on those terms. Ireland, as I understand it, was not on net exporting food–it was exporting expensive food (cattle) in order to import inexpensive food (maize). On the other hand, Ireland plus Britain was, I think, producing enough food, or at least could have afforded to import enough food, to prevent the famine. That’s the closest case I can think of to large scale famine under laissez-faire not due to the food not being available.

          • 1soru1 says:

            Very few famines involve their not being physically enough food to feed everyone; i.e. if the food was shared out optimally by rationing, people would die of starvation until the share of the dead people, split out evenly, was enough to live on.

            All other famines happen because the political system in the relevant country allocates food preferentially to one group over another, typically with group A getting as much as they want, and B only getting what is left over.

            This model fits the Holodomor, the Irish and Indian famines, Mao’s famines, and the mid-20C African ones, etc.

            Now, you may be sufficiently committed to a leader or system to say ‘the difference is, this political system was making a legitimate choice when it starved those people to death’.

            These days, defenses of Stalinism and Maoism on those grounds are relatively rare, but other political systems do have more enthusiastic supporters.

          • Very few famines involve their not being physically enough food to feed everyone; i.e. if the food was shared out optimally by rationing, people would die of starvation until the share of the dead people, split out evenly, was enough to live on.

            In the case of both the famines I mentioned, the country was exporting substantial amounts of food. I don’t know if there was enough left to feed everyone, but given how poor China was my guess in that case is not.

        • Ketil says:

          Most grocers have agreed to not trade food to people without money.

          Most is different from all, and all was a requirement for you starving. Also, this is not any agreement, just grocers being self-serving. I’m sure many would be willing to trade food for other things than money. Also, grocers don’t hold a monopoly on food, your friend could buy food for you, or you could trade directly with a farmer.

  48. melboiko says:

    I think this is the most uncharitable thing I’ve ever read from Scott’s pen, even counting the antisjwist posts. I’m used to this kind of toxicity from the blog commenters, but not from the blogmaster. I was left with just a kind of existential sadness or disappointment. All I can do is hope that it’s not a trend.

    • LadyJane says:

      Do you think the Current Affairs article deserves charity? Do you think the author was being charitable to libertarian ideas, and portraying the ideology in an honest way, and presenting convincing and well-reasoned arguments against libertarianism?

      Because from my point of view, it seemed like he was relying entirely on 1.) fighting against a nonsensical bastardized version of libertarianism that no actual libertarians believe in, and 2.) coming up with absurdly extreme edge cases where that version of libertarianism wouldn’t work, because he couldn’t even make real arguments against his own pathetic strawman version of libertarian philosophy. It’s not just jousting against a strawman, it’s jousting against a strawman with a rocket launcher. From a helicopter.

      If someone’s argument boils down to “X would never work because this utterly ridiculous scenario that would never happen in real life shows that it wouldn’t work,” then saying “X is utterly ridiculous and would never happen in real life and you should feel bad for using such a stupid example as proof of anything” is a perfectly valid response. (It’s worth noting that Scott isn’t even a libertarian himself, and his response was not meant to be an active endorsement of libertarianism. He wasn’t trying to prove libertarian ideology is right, just that these particular really stupid arguments against it were very very wrong.)

      • melboiko says:

        I believe “but mom, she did it first” is not only a flimsy pretext in general, but in the case of sarcasm it’s part of the set of rationalizations-against-the-outgroup that brought us to this sorry state of affairs in the first place.

        Let’s be Kantians. Imagine a world where Scott regularly writes like this: deliberately missing the points, dripping with venom, etc. Outgroup authors then quote Scott’s blog and reduce it to absurd, with equal snarkiness. (You did notice that the formal structure of this post is exactly that of a Tumblr toxoplasma, right?). Everyone does that to each other’s blogs. The mandatory two minutes of hate grow into hours; it becomes, in all senses of the word, normal. Moloch. Moloch.

        Do I think the original article was deserving of charity? Do I think nazis deserve human rights? Do I think Breivik deserves a safe and comfortable prison? Yes. Yes. Yes, I actually think morals do real, Goddess help my poor soul and guide me in this valley of snarks . Now let me reach for the gin.

        • Cliff says:

          “deliberately missing the points, dripping with venom”

          Did we read the same post?

          • melboiko says:

            The deliberateness of the missing of obvious, even trivial points, and the sheer intensity of the toxins in the snark, are so pervasive that the only way I can imagine someone not noticing them is out of a feeling of vindication against Them, of attachment to the delight of the two-minutes o’hate. I could go over the obvious points and rephrase all of them in a bulleted list, but what’s the point? It feels like senselessly wallowing in my disappointment, and will only worsen the bitter aftertaste of this whole affair. It’s not as if the points lacked clarity in the first place. I could cite one by one all the venomous snarky remarks, like when he says——

            Nope. Not doing this. I’m not building my own little bristly toxoplasma. I’ve learned a long ago what’s the only winning move in the game of sarcasm: not to play. The correct response to snark is an empty string. The proper way to deal with an infection is to stop it. And do you know what’s great at disinfecting? Alcohol! So let’s see, what’s next: grapefruit mimosa or licor 43 con leche?

          • Cliff says:

            Agree to disagree, I guess

          • Nornagest says:

            Don’t see the venom either, but: definitely grapefruit mimosa.

        • moscanarius says:

          Let’s be Kantians.

          Then let’s be. Imagine a world where Scott regularly goes at length to be extra charitable even to the most air-headed criticism. Outgroup authors then quote Scott’s blog and reduce it to absurd (what kind of thinker spends so much time on so trivial criticism?), all while mocking him for being an autist and trolling him with even more deliberately stupid stuff.

          Scott could avoid this by not respinding to bad criticism and letting it fester among the public, but let’s imagine this world too: outgroup authors would then mock Scott for being afraid of tackling the opposition (“ha! Not even Scott could answer it!”), all while also quotemining Scott’s blog to reduce it to absurd, regardless of what he wrote. Or while fishing the worst comments in the comment session to parade as a representative sample of Scott’s ideas, cuz why not?

          If your concern with this post is that it will make the outgroup angry, be rested: they are already angry, they will always be angry. Being more, uh, respectful of the bad arguments among them will not make them any less angry. Maybe the opposite, really.

          Do I think the original article was deserving of charity?

          So do I, but I think it got the amount charity it deserved. “Applying charity” does not mean one shouldn’t call a strawman a strawman when one sees it.

          Do I think Breivik deserves a safe and comfortable prison?

          So do I (mostly; depends on what “confortable” means). But I’m sure you agree that if the prision guard is a bit rude with him he is not commiting some sort of mortal sin, right?

      • Protagoras says:

        Do you think the Current Affairs article deserves charity?

        I think Prince Hamlet had the right of that one. “Use every man after his desert, and who should ‘scape whipping? Use them after your own honor and dignity. The less they deserve, the more merit is in your bounty.”

      • daneelssoul says:

        1) I have actually met self described libertarians who claim that their moral system consists basically entirely of you can do whatever you want so long as you do not directly physically harm another person or their property. This may be a weak man, but it is not a weak man that nobody subscribes to.

        2) There is a philosophical technique where you begin by undermining the general principle by picking a very clear-cut if totally implausible hypothetical, and only after relating it to more realistic scenarios.

        For example, you argue against someone’s principle that it is NEVER morally permissible to murder someone, by saying “well what if aliens were threatening to destroy the world unless you shot Hitler”, and only after having established that there are some theoretical circumstances where it might be permissible, do you move on to more realistic scenarios like “well what if there’s a mass shooter on the lose and the only way to stop them before they kill 10 more people is by shooting them first” and maybe after to “well what if the Nazis are going to enslave everybody unless you send soldiers to fight them”.

        Similarly, if you are arguing against an extreme “taxation is theft, which is never permissible” maybe you start with “well what if society as a whole decided to murder someone by forming a conspiracy to never trade them any food” and then when they agree that there might be weird theoretical circumstances where your choice not to let someone else trade with you could amount to murder, do you point out that this scenario has some important structural similarities to the situation that poor people might actually face in countries with insufficient social safety nets.

        • quanta413 says:

          Similarly, if you are arguing against an extreme “taxation is theft, which is never permissible” maybe you start with “well what if society as a whole decided to murder someone by forming a conspiracy to never trade them any food” and then when they agree that there might be weird theoretical circumstances where your choice not to let someone else trade with you could amount to murder, do you point out that this scenario has some important structural similarities to the situation that poor people might actually face in countries with insufficient social safety nets.

          Weird note. I’m vaguely libertarian but a Caplan argument comparing taxation to slavery actually convinced me of the opposite of his point. I now accept there may be almost impossible occasions where slavery is justifiable. Like ” Evil scientist X is deeply afraid of dying, but has made a plague which will kill everyone on earth but them in 5 years. You need to enslave the evil scientist X who made the plague and force him to aid in the development of the cure for it”. I don’t expect to ever actually see such a situation or anything even vaguely close to it, but I now think it’s not 100% ruled out. A less extreme example is a contract that makes you temporarily like a slave, forced conscription. Although there are almost no cases of forced conscription I’d want to defend.

          Also maybe it’s just me but a “well what if a whole society… conspires” scenario doesn’t seem like a useful question to me. It doesn’t matter what hypothetical society you’re part of if everyone is conspiring against you, you’re screwed. There is no good rule that is a guarantee against this possibility but even worse, it’s not desirable to have a society where it’s impossible for society to deeply harm people. Punishment of defectors is hard to make work if cooperators can’t effectively conspire against defectors.

          • daneelssoul says:

            The issue isn’t that society shouldn’t be allowed to deeply harm someone if it has justification. Nor is the complaint that libertarianism doesn’t perfectly defend you in practice against conspiracies of literally everyone else trying to do you harm.

            The issue being brought up as I see it, is that the libertarian moral philosophy (the one Scott thinks is a weak-man) says that this conspiracy’s murder of an innocent person is *morally acceptable*.

          • quanta413 says:

            It is morally acceptable to libertarians who think that all of morality is the NAP; these people are hopelessly confused on the distinction between political principles and moral principles. I have never met such a person, but I could likely find them online the same way I can find communists who advocate for shooting capitalists.

            Proper comparisons should be crazy fringe to crazy fringe. The fact that strawman libertarians at least aren’t going to shoot you seems like a plus in my book compared to most crazies. The fact that it would apparently take 90-100% of them agreeing on starving me to kill me just makes the hypothetical even more hilarious. Libertarians are not famed for their uh… organizational ability. Getting that to happen would be like herding cats.

          • Thegnskald says:

            I have never met a libertarian who thought morality and law were even in the same domain.

            It seems like it is a central, largely unspoken, tenet of libertarianism: Ethics and law are separate magisteria.

            So all this talk of the NAP as a core ethical principle seems like a Swahili translation of Japanese into English, to me. I feel like something is getting lost in translation.

          • daneelssoul says:

            I have met NAP libertarians. In person. They do exist. Calling them weak-men is probably fair, but they are not made-up internet strawmen.

            And again, the problem isn’t that you expect that NAP libertarians will actually conspiracy-starve-murder you, but that they consider it morally acceptable for people to be killed so long as the murder weapon isn’t direct violence. Or perhaps more relevantly, that they consider it to be morally acceptable for poor people to be forced to live miserable lives even when there are reasonable government interventions to prevent it (note: I am not actually claiming here that socialism in practice does a better job of dealing with these issues than libertarianism, just that there’s a plausible story that it does).

          • quanta413 says:

            @daneelssoul

            I have met NAP libertarians. In person. They do exist. Calling them weak-men is probably fair, but they are not made-up internet strawmen.

            Sure, and the communists who say they want to shoot capitalists also exist. There are also communists who actually did that. Strawmen aren’t positions that are never held. People believe all sorts of crazy things. Strawmen are beliefs that are a much weaker version of a more common belief and then the attack on the weak belief is used as an attack on the stronger belief.

            The current affairs article wasn’t titled “some puzzles for rigid deontological libertarians who make no distinction between law and morality”.

            Or perhaps more relevantly, that they consider it to be morally acceptable for poor people to be forced to live miserable lives even when there are reasonable government interventions to prevent it

            I had a longer response typed, but then I realized if I wanted to argue about the actual subject this isn’t really the place. And I’m not 100% on board with either side of this claim either.

            Obviously the real argument is harder to make, so if the Current Affairs people wanted to make it in the article, they should have. I have read good articles on related topics. But I’m not obligated to take bad articles about something and transmute them into good ones. No point in steelmanning when there are already better sources to engage with. Just like I’m not going to transmute rigid deontological libertarians who just go “communists are evil —> therefore government is. Checkmate Statists!” into David Friedman.

          • Toby Bartels says:

            The current affairs article wasn’t titled “some puzzles for rigid deontological libertarians who make no distinction between law and morality”.

            If only it had been! Change the title, change the ending (the part that Scott didn't quote) analogously (replace ‘psychotic’ with ‘psychopathic’ while you're at it), maybe add an introductory paragraph establishing the existence of such people, and now it's turning into something reasonable! Hell, present that to me out of context, tell me that it was written by another libertarian, and I'll believe it! (Kevin Carson has written worse, and on much the same lines.)

        • LadyJane says:

          The problem is, these hypotheticals aren’t just ridiculous, they’re also unsuccessful, since they don’t even prove the point they’re supposed to prove in any meaningful way. “Libertarianism doesn’t work in a hellish worst-of-all-possible-worlds nightmare scenario” doesn’t actually prove anything about libertarianism, because realistically, no system works in a hellish worst-of-all-possible-worlds nightmare scenario. With the first scenario in particular, you could swap out libertarianism for literally any other political system and you’d have the exact same results, or even worse results (in a dictatorship, the chosen victim would just summarily executed by the state as soon as the Supreme Leader commanded it; in a direct democracy, he would be lynched by the rest of the populace as soon as 51% of citizens voted for it). It’s not a dystopia because of its strict adherence to libertarian ideas, it’s a dystopia because every single person in the world is completely and totally devoted to practicing ritual human sacrifice and cannibalism, and the fact that they’re also devout libertarians is orthogonal to that.

          • daneelssoul says:

            I think you are missing the point that they are supposed to prove. The complaint isn’t that libertarian political philosophy allows this to happen (you are correct that no practical political system prevents conspiracies of literally everyone else from killing you), but that libertarian moral philosophy (the kind that believes in the non-aggression principle and property rights and literally nothing else) believes that such a situation is morally permissible. The US Constitution may allow a small slim majority to murder you and then judge the offenders to have committed no crime at their trial, but at least the supporting moral philosophy of natural rights still deems such acts to be morally wrong.

            Actually, I think that the whole Current Affairs piece makes a lot more sense when viewed from this angle. It explains why they keep asking where the non-aggression principle is violated in various scenarios, and why they bring up fully self consistent philosophies (which is one of the defenses I have heard of this libertarian moral system).

          • LadyJane says:

            I’m not missing the point, I’m choosing not to accept it because the point is stupid.

            Yes, there’s a sense in which this situation would be considered ‘acceptable’ under libertarian philosophy. But it’s the same sense in which this situation would be considered ‘acceptable’ under modern democratic philosophy, so long as a supermajority of the legislature agreed to pass a law allowing murder for this specific circumstance, and a majority of the judiciary agreed that it was constitutionally valid.

        • Hyzenthlay says:

          2) There is a philosophical technique where you begin by undermining the general principle by picking a very clear-cut if totally implausible hypothetical, and only after relating it to more realistic scenarios.

          The problem with this technique is that there’s virtually no principle that can’t be undermined in this way.

          Example:

          “Rape is never okay.”

          “What if a woman has a bomb that is about to destroy the world hidden in her cervix, and the only way that bomb can be deactivated is with semen, and only after repeatedly thrusting against it, and you are locked in a room naked with her and the bomb cannot be removed by other means without killing her and the bomb will go off in ten minutes and DESTROY THE ENTIRE WORLD if you don’t have sex with her but she still won’t consent to sex?”

          I mean, how often is this approach going to lead to useful dialogue?

    • gbdub says:

      This is precisely an isolated demand for charity. CA constructed a series of strawmen, Scott merely imagined what a world populated by those strawmen would look like.

      If “quoting long sections of an article and writing little vignettes taking every one of the articles arguments as true” is uncharitable, then nothing is charitable. And at a minimum, satire is dead, which would give me far more existential sadness than Scott occasionally getting snarky.

    • xXxanonxXx says:

      I’m curious how you distinguish between toxicity and gentle mockery.

    • moscanarius says:

      “toxicity”

      “dripping with venom”

      “venomous snarky remarks”

      Either hypersensitive, or trolling, or Inigo Montoya.

      Hard to be any more generous to your (uncharitable?) characterization of this post.

  49. Conrad Honcho says:

    I had a lot of fun reading this. Thank you Scott.

  50. Tenacious D says:

    “Maybe, some of them. I think in general they believe there are moral values other than non-aggression – after all, many of them are Christian, and believe in all sorts of moral values – but they’re skeptical of the government enforcing them. Remember, it’s not always correct to insta-convert ethics into law. I think in general they believe both that it’s important for a society to be virtuous, and that the government compelling people to exhibit more virtue than they possess can only go terribly wrong.”

    The way I phrase this is that not every sin is a crime. (And vice versa).

  51. theredsheep says:

    Anybody ever read Jared Diamond’s The World Until Yesterday? He describes the typical features of life in non-state (AKA “tribal” or “primitive” in less delicate terms) societies. The one feature that struck me most is that such societies are (for the most part) horrifically violent. Their per capita war death rate is fantastic, higher than Germany’s or Russia’s during WWII, higher even than the Aztecs’. Since there’s no higher party interested in compelling peace, and no mechanism to enforce behavior on the tribe, the only way peace can be maintained between neighboring groups is by constant diplomacy coupled with the fear of reprisal. Both break down with distressing frequency, and when they do there are no rules of engagement and no distinction between civilians and combatants. Massacres are routine. The cycle of raids and counter-raids can continue for decades before dying down.

    Everyone in such societies knows it happens, but they’re powerless to stop it, because it only takes one incident to initiate a positive feedback loop of escalating murders. You can have fifty sensible, restrained people in your tribe and one hotheaded punk who didn’t get enough love growing up. That one punk will do something stupid, like steal a pig from the neighbor tribe, and his equivalent on that side will retaliate by starting a fight that leaves one of them dead. Prompt diplomacy may wind up with payment of a blood price to ease things over, but that doesn’t always work. The only enforcement option left is MAD. It’s all downhill from there.

    I think that’s the strongest objection I have to the libertarian POV; libertarian schemes depend on people acting out of rational self-interest, with elaborate self-correcting mechanisms based on chosen associations, reputation maintenance, etc. But those in turn depend on a very fragile state of basic civility and assumptions of good faith that are only maintained in the presence of a robust state. It only takes one jerk to ruin it for everyone. And the real-life jerk ratio is way higher than 1:50.

    I’m not a libertarian, or a socialist. If anything, I have vague sympathies for distributism. But I take it as a given that somebody will wind up on top in any given system, for the same reason I don’t believe T. rex was a scavenger. When there’s all that lovely meat walking around, something’s going to show up to eat it. I’d rather have rules for the ‘rex.

    • SEE says:

      By your definition of the “libertarian POV”, Ayn Rand wasn’t a libertarian*. Which she would agree with, mind, but rather makes me wonder exactly who you think libertarians are.

      *To quote Rand:

      There is only one basic principle to which an individual must consent if he wishes to live in a free, civilized society: the principle of of renouncing the use of physical force and delegating to the government his right of physical self-defense, for the purpose of an orderly, objective, legally defined enforcement.

      • theredsheep says:

        I know there’s a distinction between libertarians and anarchists. My point wasn’t “libertarians want no government at all and this would turn into murderland,” but “the basic norms of civil order libertarians depend upon/assume are the products of state intervention in any given sphere.” And the strength of those norms is roughly proportionate to the strength of state intervention. The tribal murderland was meant as an extreme example. Looking back, this wasn’t at all clear. I was going for brevity and leaving too much implied. Sorry.

        Just for example, the free market libertarians love is itself an artifice preserved by continual state intervention. Companies don’t like to compete; it forces them to sacrifice money by either cutting prices or sinking cash into R & D. It would be much simpler to just buy out the competition and stop playing the annoying game. The only reason they don’t is because the state stops them, and it doesn’t do so as much as I’d like. To be fair, in our current system they can do things like buy special subsidies or parasitic contracts, and this is a favored example of how we’d be better off with a slimmer government. I could do without the rent-seeking myself, but I’m very glad that we have, say, food and drug inspectors, imperfect as they are. I get the feeling libertarians take the pleasant norm under continuous state intervention as a given. The present crony-capitalist corruption is nothing compared to what we’d see with a “free” market. Even now, the bigger online companies have a worrisome amount of power.

        • Cliff says:

          “It would be much simpler to just buy out the competition and stop playing the annoying game. The only reason they don’t is because the state stops them, and it doesn’t do so as much as I’d like.”

          So why has this never happened? Do you realize anti-trust began not to protect consumers but to protect producers from competition from the so-called monopolists?

          • theredsheep says:

            Do you mean that protecting smaller businesses for their own sake was the explicit rationale for such laws, or that smaller businesses were the ones asking for them? If the latter, they would be the most immediately injured party and it would make sense for them to push for recourse, whether it made sense for the rest of society or no.

            At any rate, modern companies regularly attempt to merge into omnicorps, and the government regularly reviews and sometimes blocks them. I’d rather they did more. Are you arguing that the reduction of choice to monopoly or de facto monopoly doesn’t happen, or that if it does it doesn’t hurt consumers? For the former, Amazon and Google; for the latter, look at pharmacy benefits managers. Because of the arcane interlinking of contracts, basically all major chain pharmacies are in a race to cut service quality in favor of volume ASAP. That’s just an example I’m intimately familiar with, but it resembles the old horror stories of “trusts” exactly.

    • Mark V Anderson says:

      Pinker makes this point in “Better Angels of Our Nature” that ancient tribal societies had a lot more violence than today’s world, and even a lot greater than societies of say 2000 years ago. He lists several reasons for the decrease of violence, but one of the main ones is that today’s large governments make violence a lot less beneficial to individuals than they are in tribal societies. I didn’t like reading this, because I don’t like large governments, and they certainly decrease everyone’s freedom, but I haven’t been able to make a good argument against it. I would be interested if there are good arguments against this point of view.

      David Friedman, have you read that book? Do you disagree with Pinker’s point, or maybe think that the advantages of no government offset a possible increase in violence? Actually Pinker doesn’t really make many arguments to prove this point in the book, but it sounds right to me.

      • theredsheep says:

        I’d say, off the top of my head, that it’s a result of more centralized government and improvements in technology, and the two are very much related. Most medieval kings, for most of the middle ages, had very little real control over their countries. They could collect taxes over some portion, but the roads were wretched, news traveled mostly by hearsay, and if one of your counts had a decent castle and loyal troops, there were limits to how far you could impose your will over his. Sure, you’d call out your other vassals to punish him if he was outright rebellious, but if he felt like asserting his rights in certain respects, those other vassals might well feel he had a point, if they were being asked to go to the expense and bother of a war to suppress him. Especially since they might feel like asserting their own rights. Similar constraints applied in other societies, like Byzantium or the various Islamic states. Even Rome, to some extent.

        We might talk about despotism now, but the simple fact is that historically it was bloody hard to assure the loyalty of one’s subordinates in troubled times. You had few ways of keeping tabs on everyone and local areas tended to be a lot more self-sufficient than they are today. That meant a lot more opportunities for the old ultra-violence. You’re familiar with the Normans? These Vikings ravaged the north of France so brutally that Le Roi offered to let them keep a big chunk of it if they’d stop smashing the rest and pretend to be Christian. They said yes, and their descendants spent several subsequent generations terrorizing anywhere they could reach that couldn’t defend itself.

        I haven’t read Better Angels yet, though it’s at the library and I intend to get around to it. I vaguely recall Pinker advocating stuff that struck me as creepy, but can’t recall what exactly. My understanding, from what I’ve heard, is that he’s saying economic and technological improvements have made life a good deal safer on average. True, but not exactly what the title implies; it’s more “the currently incentive-aligned angels of our nature.” Still, I should read it, if only to have an informed opinion and all that.

      • I have not read that book. I have read Warfare Before Civilization by Keeley which argues for high death rates in conflicts among primitive peoples.

      • Ketil says:

        I think it is correct, anarchy is inherently unstable, and even brutal ancient kings (and modern dictators) reduce the general violence. Which is why so many of the West’s internventions are misguided, even if Saddam and Ghaddafi and Assad and Taliban are/were horrible dictators, tearing their governments down and expecting some kind of democracy to emerge is….let’s be generous and say “optimistic”. I am really quite apalled at the apparent ignorance by western governments here, and the horrible consequences they inflict by their incompetence and/or lack of concern for human life.

        A counter-example may be DDR, where the country could be merged into an existing, functioning state with good institutions and a shared culture.

  52. aciddc says:

    I was going to post something about how this is weak-manning the Robinson piece as much as that piece weak-mans libertarianism, but I’m going to try not to disappear into the “arguing about arguing” hole.

  53. Jeremiah says:

    Podcast version is up. This is not noteworthy. What is noteworthy is I attempted voices for the various characters…

    I post here so that all might partake of the hilarious results.

  54. convie says:

    It should be noted The Infinitely Rich Man would have just as much power in a society with Government as a Libertarian one. What’s to stop him from bribing the police, courts, politicians..etc? In fact it could be argued he has even more power since he has the ability to influence such a powerful apparatus.

    • Edward Scizorhands says:

      No, because we wrote “DO NOT LET BECOME CORRUPT” on it with bright red marker.

    • daneelssoul says:

      To be fair, I think a lot of communists propose solving this problem by outlawing the existence of The Infinitely Rich Man.

      • Nornagest says:

        And how well did that work out the dozen or so times we’ve tried it?

        • daneelssoul says:

          Not well usually.

          I’m not actually making a claim here about which systems are better. I’m just saying that if the author of the original piece were communist or sufficiently socialist, the response that Infinitely Rich Man would corrupt their favored form of government, might not be valid.

      • Conrad Honcho says:

        But what about the Infinitely Powerful Man?

      • Paul Zrimsek says:

        Thereby denying their own major premise. If the Infinitely Rich Man gets to decide what the government will do, then taking away the Infinitely Rich Man’s money is pretty much the last thing he’ll let it do.

        • daneelssoul says:

          Sorry, I’m not sure what you are getting at here. Are you trying to say that it is impossible to implement a communist society without rich people in the real world because current rich people will use their influence to prevent the government from implementing it?

  55. benwave says:

    Awful lot of subject/meta level confusion in the comments, which is a shame. If the post seems to lack a certain amount of charity I would wager it’s only because it’s presented as a take down of the weak-man vs weak-man argument but doesn’t really propose a solution or alternative. But give the man a break people, sometimes you just want to break loose and write a funny story. That too, is part of Scott’s brand.

    One thing I’ve always wondered about libertarian utopia on a practical level is, creating through contracts systems and institutions to protect and promote one’s flourishing seems like an awful lot of work. I wonder if many people would ever do this for themselves rather than contract out that job to someone else. Does anyone have any information on that?

  56. benwave says:

    Also, I’d forgotten how much I liked Contra Askell on Moral Offsets, so thanks for the timely reminder to re-read that : )

  57. tayfie says:

    My one disappointment was that the identity of the time traveler was never tied into the later story.

  58. Lawrence D'Anna says:

    I dunno man. On one hand, I really appreciate what you’re doing here and. your points are totally valid and good on you for making them.

    On the other hand NAP-libertarians are a thing that exists, and I’m even a libertarian myself and I think NAP-libertarians are the most annoying people in the universe. Come on admit it, you’ve met them. Imagine how annoying they are to actual socialists. I think it may be just too big an ask to expect socialists not to make fun of them.

  59. gloriousg999 says:

    I think this is the genre of “Popular writing says dumb thing. Slightly smarter people mock it.” This seems fairly uninteresting, especially since (as many people point out) A LOT OF LIBERTARIANS PULL RIGID DEONTOLOGICAL POSITIONS. So, if we’re talking about a true Rothbardian, or true Objectivist, or many capital L Libertarians, then they will take rigid ethical stands. In fact, they may feel compelled to, as justifying everything in the world is wrong is a lot easier if one disregards nuance.

    The thing that makes this post somewhat stranger is that Scott’s post is trying to say “But how does it help to focus on this tiny pathological subset of libertarians and desperately try to convince the world that every libertarian is like this?” But….. is Scott actually right on this? So, if we look to the Libertarian Party platform, we see lines like: “We call for the repeal of the income tax, the abolishment of the Internal Revenue Service and all federal programs and services not required under the U.S. Constitution.” And the party itself was partly founded by crazy libertarian Murray Rothbard. When you look to the influential people in mainstream libertarian thought(not academic thought), it’s more frequently extremists on the Austrian school and Ayn Rand, as well as some Milton Friedman, but I think his influence is fading(unfortunately).

    I get the feeling that Scott’s model of a libertarian happens to be David Friedman. David Friedman is a good libertarian to be, and more libertarians(and non-libertarians) ought to emulate David Friedman. Most libertarians are not David Friedman. I feel like I’d like better definitions and more data before shaming people for weakmanning a position when:
    a) The original article made is clear it was addressing rights-libertarians(even if it doesn’t explicitly flag it)
    b) Rights-libertarians have played a very dominant role in libertarian politics over the last 50 years
    c) When the political party in the US expressing this view is clearly similarly extreme in expressed belief

    • pyroseed13 says:

      As a moderate, consequentialist if you will, libertarian, I had a similar reaction to this post. Scott writes hundreds of words with needless examples to make a point that he could have just made in a few paragraphs. A point that I would add doesn’t really address the fundamental critique in Nathan’s piece, which is that deontological libertarian principles taken to their logical conclusion actually yield what most would regard as morally problematic consequences. In fact, it’s strange to see Scott resort to this kind of hand-waving here given that Scott famously criticized this type of rigid thinking here: http://lesswrong.com/lw/e95/the_noncentral_fallacy_the_worst_argument_in_the/

      Having spent quite a bit of time hanging around libertarians at different seminars and conferences, I can confirm that “rights-based libertarianism” is certainly not a non-influential strain of thought in the movement. In fact, it is what chiefly distinguishes the Ron Paul’s from say the Milton Friedman’s. Incidentally, even libertarian academics such as Jeffrey Friedman and Jeffrey Miron have written pieces critiquing those more extreme strains. See Friedman’s “What’s Wrong with Libertarianism?” for example.

      • baconbits9 says:

        A point that I would add doesn’t really address the fundamental critique in Nathan’s piece, which is that deontological libertarian principles taken to their logical conclusion actually yield what most would regard as morally problematic consequences.

        Well there is the issue that they aren’t logical conclusions. What ifs aren’t logical, they are conditional. What if I decide to smother my wife in bed tonight? This is a logical conclusion of getting married right? You spend virtually every night together, totally vulnerable and it only takes one night to smother another, so logically all marriages end in suffocation.

        If you conditionally make it so that people will act in X way and then draw your conclusion that is how you get ridiculous ‘logical’ conclusions.

        • pyroseed13 says:

          This is a fair point, but it’s hard to know where to draw the line. For example, in regards to Scott’s position on torture, he states that you could probably construct a “ticking time bomb” scenario where it would justified, but nonetheless has a strong moral presumption against torture. I have the same view. But if you are willing to reappraise your position on torture based on evidence of its efficacy, then you don’t believe that torture is ipso facto morally wrong. The fact that you could imagine perhaps only 1% of cases where it would be acceptable means that you could not justify your opposition to torture on deontological grounds. So if I can think of some examples, however unlikely, where following the non-aggressive principle led to some very undesirable outcomes, shouldn’t this at least caution against constructing a society solely based on it?

          • Skivverus says:

            But if you are willing to reappraise your position on torture based on evidence of its efficacy, then you don’t believe that torture is ipso facto morally wrong.

            Er. I think you lose me here, at least. I can say I don’t believe torture is infinitely morally wrong, but still say that it’s very morally wrong, even in those edge cases where the known alternatives (including “searching for less wrong alternatives”) are worse.

        • Guy in TN says:

          If you advocate for deontological libertarianism, except for in cases with really serious consequences (and for those cases you switch to utilitarianism to make the call), then I would say that your deontological positions aren’t doing much of the work for your ideological grounding. If you can’t even convince yourself to hold onto them when the going gets tough, then you’re going to have a heck of a time convincing me.

          ——————————————————————————-
          The attitude general attitude here towards Nathan’s piece seems to be “come on, nobody is actually a deontological libertarian, they just use what appears on the surface to be deontological reasoning for their rules-of-thumb”. But no. Deontological libertarians exist, in real life, and are in no small number. The libertarian ideas of “self-ownership”, the “homestead principle”, and a moral distinction between the state and private property, underpin much of classical liberal philosophy.

          When Nathan points out that the tangible difference between a state and an unregulated property owner is essentially nil, and that enforcing ownership requires the threat of initiation of violence, this strikes at the heart of the liberal ethical framework that supports the capitalist system. It’s not just nominal “Libertarians”.

          If the Current Affairs piece was bad, it was because it was poorly worded, and poorly fleshed out, not because it was unnecessary. Heck, Scott devoted a large section of his FAQ to deontological libertarianism a few years ago, much in the same vein of what Nathan attempted to do, but done with 10x more thought and nuance.

          • Toby Bartels says:

            When Nathan points out that the tangible difference between a state and an unregulated property owner is essentially nil, and that enforcing ownership requires the threat of initiation of violence, this strikes at the heart of the liberal ethical framework that supports the capitalist system.

            I agree. But reading it, it never occurred to me that he was pointing that out. (I can see it, now that you mention it, as the unifying theme.) And I suppose that it didn't occur to Scott either, since (as you say) he's made the same point before himself.

          • 1soru1 says:

            What tends to happen, in even in the steeliest of people, is a certain dance between the two perspectives, something like:

            Obviously I agree that not all regulations are bad; I would be an idiot straw man if I did. And I can accept reality; this particular proposal advertised as a deregulation
            did in fact turn out to be a bad idea. Nevertheless, the reason it failed was because of the interaction with these other regulations, and so in an ideal world with no regulations, a perfect free market, it would have worked. So I will continue to default to supporting similar proposals

  60. JPNunez says:

    I am not sure why we need to strawman a strawman of libertarianism, when there is a very specific example right now going on in america where libertarianism is both clearly in the wrong, and also literally killing real children.

    • quanta413 says:

      Libertarians control no branches of government and are total nobodies.

      The only possible claim I can think of is that you’re saying gun control is needed because people shoot children sometimes. You should just say it if that’s what you’re thinking of.

      • JPNunez says:

        They do share the position of absolutely no gun control, though.

        Point is I don’t know why we are looking at ridiculous examples of where libertarianism does not work when one of their positions is clearly at work.

        Besides, if your objection is that libertarians control no branches of government, then why are we discussing them at all anyway.

        • Conrad Honcho says:

          What laws would you like passed to stop children from being killed? Given that the number of children killed with guns in a nation of ~330 million people with ~270 million guns is extremely tiny, can you think of any ways in which that law would lead to more people being killed than the lives saved by your law?

        • quanta413 says:

          Besides, if your objection is that libertarians control no branches of government, then why are we discussing them at all anyway.

          I wish I knew why Current Affairs wanted to discuss that. Maybe Current Affairs is sad at their lack of influence? Maybe they can feel a little better by taking potshots at strawmen with even less influence?

          Sadly, it’s basically one step removed from “for the lulz” and we are all getting into the muck too (including Scott).

    • John Schilling says:

      We’ve had that discussion before, and we are having it right now in parallel threads, and there is absolutely nothing to be gained by having it here as well. Also, beginning the discussion with “My side is obviously right”, disqualifies you from having anything useful to contribute. Please go away.

  61. nestorr says:

    The webcomic Unsounded has the cannibal village scenario as one of the main religions. Twins are sacred so one of ’em gets ‘et when they reach a certain age. It’s been mostly background window dressing so far but it’ll be explored in the current chapter, which takes place in a twin shrine.

    Religion always helps with this sort of thing.

  62. MB says:

    I think the author is making a valid point here — in a perfectly (to the point of caricature) libertarian society, the possibility of coordinated action, for good or for evil, is drastically diminished. Possibly, society breaks up into smaller autarchic independent communities. Maybe there are no more roads, maybe there is no heavy industry, maybe there is no large scale infrastructure.
    Thus, this puts a practical limit to the amount of evil the Infinitely Rich Malicious Billionaire can do.
    Yes, there may be some localized evil — some cannibal community, here and there. But without government support there is only so much they can do.

  63. Sniffnoy says:

    Heh, was going through old Steven Kaas tweets today and found this one on the subject of #3:

    All our moral theories are plagued by fundamental conceptual difficulties, so let’s go to the zoo and bite all the zebras!

  64. marvy says:

    disturbance in the Farce

    I get the feeling you’ve been saving this one for the right moment 🙂

    best pun I’ve read since Unsong ended

  65. emlinne says:

    Have I ever told you you’re my hero?

  66. Brian Bleakley says:

    I don’t think libertarians who want to derive a complete system of ethics are a tiny inconsequential minority. There are only three systems that I know of that attempt to derive a ethical system from a single moral axiom: Libertarianism (based on the non-aggression principle), Utilitarianism (the greatest good for the greatest number), and Divine Command Theory (the moral dictates of God are True and Good). Of the three of these libertarianism produces the most coherent, contradiction-free system that drives critics to the most contrived, unbelievable scenarios to discredit it. Utilitarianism and divine command theory can force people into situations that are clearly unethical using situations that actually occur in the real world.

    The desire for a coherent moral system that isn’t based on fiat is very appealing for a lot of people, this is becoming even more true as the liberal world grows more secular; many people are drawn to libertarianism for philisophical reasons. These puzzles and thought experiments actually do serve a purpose, they provide a useful critique of libertarianism as the popular moral philosophy that it is.

    I personally think you have it backwards. Libertarianism roughly succeeds as a limited moral philosophy (it can only distinguish between acts that are permissible and acts that are forbidden- it cannot account for acts that are good or bad-but-not-forbidden) but fails as a political position/system of government because of the preponderance of prisoner’s dilemma-type situations in the real world.

    I did find this piece really funny, I just think you missed the point.

    • Lillian says:

      Utilitarianism […] can force people into situations that are clearly unethical using situations that actually occur in the real world.

      Wait, really? All the criticisms of utilitarianism i’ve seen involve bizarre hypothetical scenarios. What real life situation does utilitarianism fail in?

  67. Two comments:

    1. If you want to see attempts to explore contradictions between NAP theory and the moral intuitions of libertarians, you might want to look through back issues of Liberty. Bill Bradford had some good ones. On the whole, I suspect libertarians are better at that game than non-libertarians.

    To illustrate which …

    2. I buy a strip of land that entirely surrounds the place you are currently standing, which may or may not be on your land. I don’t have the right to kill you for trespassing, so can’t use the threat to do so to starve you to death. But I do have the right to use force to keep you from violating my rights–that’s part of what “right” means. So every time you try to cross my land I and my two big friends push you back.

    Are we violating your rights?

    • sharper13 says:

      Helicopter with winch and pilot rental for the win? 🙂

      In all seriousness, sounds like a good argument to insist on arranging for a deed with access rights with your land and for people you visit to contract for access rights for themselves and their guests for their land.

      • Toby Bartels says:

        Helicopter with winch and pilot rental for the win? 🙂

        English Common Law held that property owners also own the air ‘up to the heavens’ (‘usque ad cœlum’). Obviously, Libertopia need not follow that rule, but you'd better find out what rule they do follow, or you might get shot down.

  68. mjr says:

    Of course, the rich man insurance will now kill the poor wife, as the Infinitely Rich Man has no incentive now to make the offer of health care, so the insurance won’t either.

  69. Humbert McHumbert says:

    Isn’t this the same Current Affairs whose editor wrote a scathing critique of the trolley problem as a meaningless hypothetical?

    • I don’t know if Robinson wrote the piece, but googling around I came across a different piece on libertarianism that he did write.

      “There is a wearying familiarity to The Libertarian Mind; Hayek wrote all of this in The Constitution of Liberty, then Rothbard wrote it again in The Ethics of Liberty, then David Friedman in The Machinery of Freedom. Read one sentence of one libertarian book and you’ve read every sentence of every libertarian book. ”

      ..

      “So there you have it: libertarianism ranges from people who support small governments and free market capitalism to… people who support small governments and free market capitalism. “

      So he apparently knows what’s in my book and Rothbard’s without having actually read either of them.

      • Toby Bartels says:

        So he apparently knows what’s in my book and Rothbard’s without having actually read either of them.

        Hey, he read one sentence of Hayek's book, what more does he need?

        More seriously, one could argue that no government is included within small government. Zero is a very small number.